Addison Johnson_Boards Questions

¡Supera tus tareas y exámenes ahora con Quizwiz!

Dural ectasia is? Thoracic cage deformity An opening in the cranial dura mater A type of scoliosis Enlargement of the dural sac

Enlargement of the dural sac

Epilepsy alone does not increase risk of major congenital malformations: True or False

True AEDs can increase risk of birth defects, but not epilepsy alone (valproic acid is associated with greater incidence of major congenital malformations than other AEDs)

The level of statistical significance of SNP-disease association identified by GWAS does not imply the strength of the SNP-disease association. True or false?

True

The majority of heart defects associated with a CNV is dx with DiGeorge syndrome, a deletion in the q arm of chromosome 22: True or False?

True

True or False: the ACMG recommends that UPD testing be considered when a carrier fetus is identified with a Robertsonian translocation involving chromosomes 14 or 15

True

Isolated hydronephrosis is noted on the prenatal ultrasound of a 30-year-old woman. What mid-pregnancy risk of Down syndrome do you quote to her? a. 1 in 10 b. 1 in 20 c. 1 in 100 d. 1 in 500

c. 1 in 100

Trisomy 18 is NOT the most common aneuploidy associated with which of the following ultrasound findings? a. choroid plexus cysts b. single umbilical artery c. hydronephrosis d. clubfoot

c. hydronephrosis

Prenatal ultrasound in the second trimester detects cystic hygroma. Which of the following is NOT correct? a. cystic hygroma is associated with congenital heart defects, certain genetic syndromes, and teratogens including alcohol b. cystic hygroma can progress into hydrops c. if the cystic hygroma resolves later in pregnancy, there is no longer an increased likelihood of aneuploidy d. chromosome analysis by amniocentesis should be offered

c. if the cystic hygroma resolves later in pregnancy, there is no longer an increased likelihood of aneuploidy

Which of the following family histories warrants assessment for FAMMM? a. man with two melanomas b. man with melanoma; brother with melanoma; father with prostate cancer c. man with melanoma; brother with melanoma; father with pancreatic cancer d. man with melanoma; brother with melanoma; father with renal carcinoma

c. man with melanoma; brother with melanoma; father with pancreatic cancer Clinical dx: Need both of the following - Malignant melanoma in at least one FDR/SDR - High total body nevi count (~ >50), includes some clinically atypical nevi Other tumors: - Pancreatic cancer (Overall, ~17% by age 75, but not well established; Varies significantly by CDKN2A mutation) - Brain tumors

True or False: 22q11.2 deletion involves haplosufficiency of many genes resulting in a multisystem disorder

True

Up to 40% of Warfarin response is due to genetic variation: true or false?

True

Genetic heterogeneity

a phenotype that can be caused by variants of any of several genes

Unaffected parents of a child with 22q deletion have the following recurrence risk

An ill-defined number that is slightly elevated over the general population risk REASONING: If the parents have not been tested for the 22q11.2 deletion but are clinically unaffected, the risk to the sibs of a proband appears to be low. However, sibs of a proband with clinically unaffected parents are still presumed to be at increased risk for 22q11.2DS because of the possibility of reduced penetrance in a heterozygous parent or parental germline mosaicism.

A mutation in which a cytosine is replaced by a thyme is called a A. deletion B. frameshift C. transition D. transversion

C. transition Why? - Transition: pyrimidine (T or C) replaces a pyrimidine or a purine (A or G) replaces a purine - Transversion: a purine and a pyrimidine are interchanged

Individuals with 22q11.2 deletion have the following recurrence risk

0.5

Uniparental isodisomy

Both chromosomes from the one parent are identical

Coping mechanism: escape-avoidance

Hopes for a miracle

AMA increases the risk for genetic disorders arising from?

Nondisjunction

Synthesis of RNA using a DNA template is?

Transcription

Short QT syndrome

- A heritable disorder of the electrical system of the heart - Characterized by short QT interval on EKG (<320ms) and tall, peaked T waves - Increased risk for SCD and atrial fibrillation - Same symptoms as long QT syndrome (clinically indistinguishable from long QT) KCNH2, KCNQ1, KCNJ2 (AD) Detection rate unknown

Facts about genome-wide UPD

- All living cases are mosaic - 23 cases in literature (20 paternal) - All individuals with paternal GWUPD are female (YY line not viable) - Individuals with maternal GWUPD could by mosaic 46,XX/46,XY - Most common mechanism suggested: normal fertilization followed by endoreduplication

End joining (non-homologous repair)

- Does not require a template. - Broken DNA ends are aligned and rejoined by DNA ligase. - Result of repair generally is loss of nucleotides at repair site. - Tolerated well as long as damage not with gene coding or regulatory sequences. Problems: - Could remove nucleotides from an essential gene - Could join two different ends together if multiple breaks are present - Potentially give deletions, inversions, translocations

X-inactivation mechanisms

- Involves altered chromatin structure through differential methylation of DNA - CpG islands in silenced genes = methylated - X-inactivation center at Xq13.2 (Cis for X-inactivation) - XIST (x inactive-specific transcript) is uniquely expressed from the inactive X only

Underlying faulty mechanisms leading to disease phenotype: - Lynch syndrome - MUTYH-associated polyposis - Rb and p53 - Xeroderma pigmentosum, Bloom, and Cockayne - Ataxia-telangiectasia - BRCA1/2

- LS: Mismatch repair - MUTYH: Base excision repair - Rb and p53: CpG dinucleotide hotspots - XP, Bloom, Cockayne: Nucleotide excision repair - AT: Double-strand break repair - BRCA1/2: accessory proteins involved in homologous recombination (repairing double-strand breaks) *Almost all cancer genes are involved in DNA repair

Differences between mitosis and meiosis

- Meiosis produces 4 daughter cells not 2. - Meiosis produces genetically different daughter cells. - Meiosis produces daughter cells that half the amount of DNA compared to the parent cell. - Meiosis involves 2 divisions.

Decision-making styles

- Rational: characterized by a thorough search for and logical evaluation of alternatives - Intuitive: characterized by reliance on hunches and feelings - Avoidant: characterized by attempts to evade decision-making - Spontaneous: characterized by a sense of immediacy and desire to get through the decision-making process ASAP

Methods to detect UPD

- SNP array - Microsatellite testing - Methylation-specific polymerase chain reaction (MS-PCR) - Methylation-specific multiplex ligation-dependent probe amplification (MS-MLPA)

Inbreeding coefficient (F) for various relationships: always draw out the pedigree and count relationship lines, then multiply by 1/2! - Siblings: - Parent child: - Half-siblings: - Uncle-niece: - First cousins: - First cousins once removed: - Second cousins:

- Siblings: 1/4 - Parent child: 1/4 - Half-siblings: 1/8 - Uncle-niece: 1/8 - First cousins: 1/16 - First cousins once removed: 1/32 - Second cousins: 1/64

Molecular testing techniques

- Souther blots - Techniques based on allele specific oligos - MALDI-TOF (mass spec) - Sequencing - Array analysis (SNP or CGH)

The number of genetic conditions associated with developmental disabilities is closest to 10 100 1000 10,000 100,000

1000

What is WES detection rate dependent on?

- patient population - indication - phenotyping - availability of family members for testing estimated to be 25-51%

GWAS design

- start with people with disease = cases - find "similar" people without disease = controls - look at lots of SNPs across genome - see if any of the SNPs are more or less likely to be found in cases v controls

Translation initiation requires?

- the small and large ribosomal subunits - GTP - A specific charged tRNA (initiator tRNA) - Initiation factors *Translation is initiated at an AUG codon in the mRNA, which encodes methionine

Partial hydatidiform moles are usually associated with ___________ fetus and _____________ undergo malignant changes. Classic moles are associated with ______________ fetus with a ___________ karyotype and _______ undergo malignant changes.

- triploid - rarely - absent - diploid (usually diandric diploidy) - often Important to karyotype molar pregnancies because difficult to distinguish complete vs partial

WES pipeline example

- ~300,000 variants called by atlas-SNP and atlas-indel - in human gene mutation database? (collection of known variants responsible for human inherited disease) - minor allele frequency <5% according to the 1000 genomes project - down to 400-700 variants; variants in genes associated with single gene disorders? - disease gene related to patient phenotype? - variant interpretation? (evolutionary conservation; amino acid change; predicted effect on protein [polyphen, SIFT]) - confirmation for proband (usually done by sanger sequencing) - FINAL: diagnostic deleterious mutation identified and confirmed

The exome is what percent of the genome?

1-2%

Southern blot process

1. restriction enzymes cut DNA of sample 2. gel electrophoresis separate DNA fragments based on size 3. DNA fragments transferred to a membrane 4. probed with different single DNA sequences that will bind with fragments in sample if complimentary 5. probes are labelled with radioisotopes so can be distinguished from sample DNA

The patient from the previous question says that she would consider a CVS if indicated. What maternal serum screening should be considered? [select as many as apply] a. combined first trimester screening b. integrated screening c. stepwise sequential screening d. contingency screening

1. A and C. NSGC recommendations also note that of the tests reporting results in the 1st trimester, stepwise sequential screening has the highest detection rate for T21 and T18.

Contracting - Genetic counselors establish a mutually agreed upon GC agenda with the client by doing the following 4 things:

1. Describe the GC process to clients 2. Elicit client expectations, perceptions, knowledge, and concerns regarding the GC encounter and reason for referral 3. Apply client expectations, perceptions, knowledge, and concerns toward the development of a mutually agreed-upon agenda. 4. Modify the GC agenda, as appropriate by continually contracting to address emerging concerns.

Assumptions of Hardy-Weinberg law

1. Random mating 2. No selection 3. No new mutations 4. Population is infinitely large 5. No migration

Mechanisms leading to whole chromosome UPD

1. Trisomy rescue 2. Monosomy rescue 3. Post-fertilization errors (mosaic UPD) 4. Gamete complementation

16p11.2 microdeletion syndrome causes some cases of autism. About 1% of individuals tested by array comparative genome hybridization (CGH) for autism spectrum disorder (ASD) have the common 16p11.2 microdeletion, while only 0.03% of people in the general population carry the microdeletion. According to the CDC, overall estimated ASD prevalence is ~15 out of 1,000 eight-year-old children. So, if CGH is performed on 10,000 8-year-olds, here's what happens: 16p11.2 del / No del - ASD 300 98,200 + ASD 15 1,485 1. What is the sensitivity of the 16p11.2 microdel in detecting cases of ASD? 2. What is the specificity? 3. What is the PPV? 4. What is the NPV?

1. sensitivity = TP / (TP+FN) = 15 / (15+1485) = 1% 2. specificity = TN / (TN+FP) = 98200 / (98200+300) = 99.7% 3. PPV = TP / (TP+FP) = 15 / (15+300) = 4.5% 4. NPV = TN / (TN+FN) = 98200 / (98200+1485) = 98.5%

A pregnant couple are first cousins once removed. The couple shares ______ proportion of their genes. And, their child, on average, would be expected to be homozygous (autozygous) for _____ proportion of the genome.

1/16 (1/2 x 1/2 x 1/2 x 1/2) - because 4th degree relatives so F = R/2 = 1/32

Among children with ASD, the number who have fragile X as the underlying cause is closest to 1/20 1/200 1/2000 1/20,000 1/200,000

1/20 (5%)

You see a Caucasian couple in prenatal clinic because of a family history of CF. Your patient's brother had CF. She has tested positive for being a CF carrier and her partner tested negative on his carrier screening. The test is reported to have 90% detection rate. What is her partner's residual risk of being a carrier for CF?

1/241 or about 0.3%

What % of CPM is noted on a CVS due to true fetal mosaicism?

10%

What is the approximate detection rate with CMA for global developmental delay, ID, and/or MCA?

15-20%

BWS differential diagnosis: 1. infant hypotonia 2. older children (obesity)

1a. Down syndrome or other chromosome rearrangement 1b. SMA type 1 1c. Myotonic dystrophy 1d. Other myopathy/neuropathy 2a. Fragile X 2b. Bardet Biedl 2c. Microdeletion

Cytogenetic abnormalities occur in what % of pregnancies?

2% - 0.6% whole chromosome abnormalities - 1.3% microdeletions and duplications

A child with facial angiofibromas, hypomelanotic macules, cardiac rhabdomyomas, and dental pits was recently diagnosed. What do you tell the parents as the estimated de novo rate of mutations in the TSC1 and TSC2 genes? a. 10-12% b. 35-45% c. 65-75% d. near 100%

2/3 = c. 65-75%

A DNA molecule has 40% G+C content. What percentage does each base contribute?

20% C 20% G 30% A 30% T

Turner syndrome occurs in about _% of all abnormal SABs. Its frequency among live borns is about _.

20%, 1/5000 females

What % of DCM is genetic?

20-50% More than 30 genes identified which explain ~half of familial DCM Most often AD (80-90%), but AR, XL, and mitochondrial reported Mutations in TTN are most common known genetic cause

What is the recurrence risk of hypoplastic left heart syndrome?

22% for some type of heart condition but not exclusively hypoplastic left heart.

What is the most common microdeletion syndrome?

22q Prevalence of 1/1000-1/2000

What percentage of individuals with 22q del develop psychosis?

25% (0.25) schizophrenia in adults

Incidence of Huntington disease in Western European populations is ~5 in 100,000. What is the allele frequency of the disease allele?

2pq = 5 in 100,000 so q = 5 in 200,000

A child from a first cousin mating has what risk above GP for a congenital anomaly?

2~2.5% above GP

The editing or proofreading function of DNA polymerase is due to?

3'-5' exonuclease activity DNA polymerase has a 3'-5' exonuclease activity that proofreads the growing strand: - Will not add the next nucleotide unless there is good base pairing at the new 3' end - If base pairing is not perfect, the incorrectly paired base at the 3' end is removed. - The polymerase activity resumes and incorporates the appropriate base.

What is the recurrence risk of a neural tube defect in a family with one previous affected child?

3-5% 3-5%, if 2 affected siblings the risk increases from 12%

IN the previous scenario, a child with the karyotype: 46,XY,der(14;21)(q10;q10)+21 has a _% chance that one of their parents is a balanced carrier of the translocation?

40%

If 9% of the African population has sickle cell anemia (ss), what percentage will be more resistant to malaria because they have (Ss)?

42% Work: - q^2 = 9% = 0.9 - q = 0.3 - p = 0.7 (because p + q = 1) - 2pq = heterozygotes = 2(0.3)(0.7) = 0.42 Change to fractions to do math and multiply top and bottom by same number so denominator = 100 2 x 3/10 x 7/10 = 3/5 x 7/10 = 21/50 = 42/100

A balanced robertsonian translocation carrier has how many chromosomes?

45

⅔ of babies with a second trimester cystic hygroma have what genetic diagnosis?

45,X

A child presents with a "greek warrior helmet appearance, ID, and hypotonia. Wolf-Hirschorn syndrome is diagnosed by CMA. Most cases are de novo ________________, usually with ________________ origin. In the remaining cases, one parent is a balanced translocation carrier, usually _______________.

4p deletion, paternal, maternal

Kalydeco is the first medication that treats the underlying cause rather than the symptoms of CF. It is most effective in individuals who have class III (such as G551D) and class IV (such as R117H) mutations. What percentage of patients with CF will currently benefit from this treatment? 5% 10% 25% 50%

5%

Surveillance for WT should be pursued if the risk for tumor development is greater than...?

5%

If one strand of a DNA molecule has the following sequence: 5'-AATCGGATTCG-3', then the complementary strand would have what sequence? (keep answer in 5'-x-3' format)

5'-CGAATCCGATT-3'

What % of hypertrophic cardiomyopathy (HCM) is genetic?

60-70% caused by mutations in sarcomere genes (AD) - detection rate not 100% though MYH7 and MYPBC3 make up ~40% of HCM MYH7 associated with younger age of dx, more severe hypertrophy, and nearly complete penetrance, variable survival

What is the coverage of the indicated position in this diagram? Can't insert picture but you would count the amount of reads being compared to the reference genome (2017 review course --> new genomics folder --> WES tutorial --> question 2)

6x

The CFTR gene is located on chromosome #7 and codes for 1480 amino acids. The most common CF mutaiton is F508del. What percentage of patients with CF have this common mutation? 15% 50% 70% 100%

70% *in caucasians 30-80% depending on ethnicity

What percent of human genes use alternative splicing?

75%

What percent of disease causing mutations are found in the exome?

85%

In prenatal clinic, a patient tells you that she has had two basal cell carcinomas. Which of the following additional pieces of information would be enough to refer the patient to be evaluated in the cancer genetics clinic for nevoid basal cell carcinoma syndrome? [select all that apply] a. the patient was diagnosed with her first BCC at age 29 b. the patient was diagnosed with her first BCC at age 39 c. the patient's father has been diagnosed with 2 BCCs d. the patient's father has been diagnosed with 7 BCCs

A - pt < 30 D - father with more than 5 in lifetime 2 of 5 major criteria: · Multiple BCCs - >5 in lifetime or 1 <30y · FDR with NBCSS dx

Which of the following indicates a multifactorial or complex etiology? A concordance rate of 100% among dizygotic twins A concordance rate that is higher for monozygotic than dizygotic twins A concordance rate that is higher for dizygotic than monozygotic twins A concordance rate of 25% among dizygotic twins

A concordance rate that is higher for monozygotic than dizygotic twins

A couple is considering pregnancy. The woman has an AD-form of skeletal dysplasia; the man is unaffected. Pre-conception counseling is suggested for what reason(s)? a. to offer molecular testing for the woman before pregnancy b. to discuss pregnancy and delivery management specific to individuals with skeletal dysplasia c. to offer carrier testing for the man for skeletal dysplasia d. to discuss the risk of homozygosity or compound heterozygosity for skeletal dysplasias.

A and B (A) is good if prenatal diagnosis would be desired. (C) while there are AR and XL forms of skeletal dysplasia, there is no indication that is a concern here. (D) would only be an issue if the man were also affected with skeletal dysplasia.

A 60-year-old woman is concerned about her family history of breast cancer. Which of the following would further increase her risk of breast cancer? [select as many as apply] a. dense breast tissue b. fibroadenoma c. papilloma d. first child at 32

A and D Risk factors o Age o Early menarche (<12) o Late menopause (>52) o Breast density o Nulliparity or first child after age 30y o Estrogen / Progesterone use after menopause o More than 2-3 alcoholic beverages per week

Exceptions to random mating: consanguinity (inbreeding)

A couple share a common ancestor and therefore share more alleles which are identical-by-descent than expected in a random mating population. More likely to share a potentially harmful mutant allele than if they mated with someone from the general population. In rare autosomal recessive conditions, one finds a higher likelihood of consanguinity among the parents of the affected child

Interference with intrinsically normal development and structure is a disruption? A malformation? A deformation?

A deformation such as club foot

Genomic imprinting

A normal expression or non-expression (or the gene's "behavior") of certain genes influenced by the parent of origin, i.e. maternal or paternal inheritance Imprinting = DNA methylation - The addition or removal of methyl groups controls gene activity

At what age are individuals with DICER1 syndrome at the highest risk of developing the serious tumor type mentioned in the previous question? A. Age 0-5 B. Age 6-10 C. Age 11-15 D. Age 16-20

A. Age 0-5 Highest risk is for children <7y

UPD testing should be considered in which of the following situations? [select as many as apply] a. CVS shows trisomy 6 b. amnio shows mosaicism for trisomy 7 c. a pediatric patient with features of Russell Silver syndrome d. a fetus found to carry a de novo Robertsonian translocation involving chr 14 and 15 e. CVS and amnio show confined placental mosaicism of trisomy 13

A, B, C, D. Not E because chr 13 is not one of the critical chromosomes for UPD, but CPM is a significant risk factor if seen for chr 6, 7, 11, 14, 15

For which of the following condition(s) would you expect to see an AR pattern of inheritance? a. Werner syndrome b. Xeroderma pigmentosa c. Dyskeratosis congenita d. Familial Wilms tumor

A, B, and sometimes C (can also be XL-R) Familial WT = AD

A woman is seen for prenatal genetic counseling due to an abnormal maternal serum screen. The counselor notes that the patient is frequently tearful and seems to have trouble concentrating. The patient says that she "Just can't sleep." Which of the following is the BEST initial response from the counselor? A. "Tell me more about how you have been feeling before and during the pregnancy." B. "Would you consider meeting with a psychologist for evaluation of your sadness?" C. "It is normal to feel down or even depressed after hearing about this kind of result." D. "Who have you turned to for emotional support since hearing about the test results?"

A. "Tell me more about how you have been feeling before and during the pregnancy."

A 38y woman with breast cancer comes for genetic counseling. Her mother died of ovarian cancer at age 35. The woman states that she is fearful her 19-year old daughter will develop cancer and wants to know what testing options are available. Which of the following is the genetic counselor's BEST response? A. "To accurately determine your daughter's risk for cancer, you would need to have genetic testing first" B. "To confirm your mother's diagnosis, we need a copy of her pathology report" C. "It's understandable to worry about your daughter. We can test her because she is an adult" D. "My daughter is also at risk for a BRCA2 mutation and I worry about her, too"

A. "To accurately determine your daughter's risk for cancer, you would need to have genetic testing first"

A newborn male with hydrocephalus, microtia, and a ventricular septal defect is referred to the genetics clinic. His mother has been treated for cystic acne, but she reports that she stopped taking all medications when she learned she was pregnanct. When obtaining the pregnancy history, which of the following is MOST important to ask? A. "What medications were you prescribed, and when did you stop taking them?" B. "What medications were your prescribed and how long did you take them prior to conception?" C. "Did you go on a special diet prior to pregnancy?" D. "Did you have fever or flu-like symptoms in early pregnancy?"

A. "What medications were you prescribed, and when did you stop taking them?"

A 28-year old pregnant woman comes for genetic counseling because the father of her female fetus has myoclonic epilepsy with ragged red fibers (MERRF). The risk that the fetus is affected with MERRF is closest to which of the following? A. 0% B. 25% C. 50% D. 100%

A. 0% Mitochondrial inheritance

The risk of non-disjunction for a 40 year old is? A. 0.01 B. 0.05 C. 0.1 D. 0.15 E. 0.25

A. 0.01

Approximately what percentage of individuals with HCM have more than one mutation? A. 0.05 B. 0.1 C. 0.2

A. 0.05 (5%)

Up to ________ of patients have de novo germline mutations in the APC gene A. 0.3 B. 0.2 C. 0.5 D. 0.05

A. 0.3 De novo rates to know for boards: - APC (30%) - TP53 (7-20%) * STK11 (50%)

At CVS, confined placental mosaicism is seen in _____ of cases. Of cases with CPM, ______ are associated with true fetal mosaicism. A. 1-2%; 10% B. 10%; 1-2% C. 10%; 10% D. 10%; 50%

A. 1-2%; 10%

What is the incidence of BRCA mutations among individuals of AJ ancestry? A. 1/40 B. 1/20 C. 1/50 D. 1/10

A. 1/40

Which of the following is the cost commonly reported mutation in CHEK2? A. 1100delC B. 1100delG C. I157T D. 1400delT

A. 1100delC Truncating mutations like 1100delC Missense mutations like I157T

A 50-year old woman tests negative for a familial mutation in the BRCA1 gene. She plans to discuss ovary removal with her gynecologist because she believes her risks are still too high. The genetic counselor should clarify that the woman's lifetime risk of ovarian cancer is closest to" A. 2% B. 10% C. 40% D. 50%

A. 2%

Synthesis of a new DNA strand occurs in the? A. 5' to 3' B. 3' to 5' C. 2' to 3' D. 4' to 5'

A. 5' to 3'

% of pregnant women who take a medication A. 50-90% B. 1-2% C. 10-20% D. 3-5%

A. 50-90% Majority of which are non-teratogenic

What is the rate of de novo mutations in LFS A. 7-20% B. 1-10% C. 50-90% D. 1

A. 7-20%

Approximately what % of Alzheimer disease is caused by AD gene mutations? A. <5% B. 5-10% C. 20-25% D. 20-40% E. none

A. <5%

The detection of IBD in array analysis of patients is useful in helping to identify A. AR conditions B. Contiguous gene deletions C. Dominant-negative syndromes D. Haploinsufficient syndromes E. Triplosensitive syndromes

A. AR conditions

Which of the following cardiomyopathies is generally a genetic disorder, unlikely to be caused by other factors? A. ARVD B. Restrictive cardiomyopathy C. HCM D. DCM

A. ARVD

A 15-year old girl at 21 weeks gestation comes in with her mother to the genetic counselor because the fetus has gastroschisis. The patient's mother insists that the birth defect is incompatible with life and that she should terminate the pregnancy, but the patient says she wants to keep the baby. However, the patient says that she might terminate the pregnancy if the counselor says the chances of the fetus surviving are poor. The mother becomes agitated when the counselor states that the birth defect is often compatible with life. She accuses the counselor of undermining her authority as the mother. Which of the following is the BEST response by the counselor? A. Acknowledge the mother's concern but inform her it is the counselor's obligation to provide accurate information B. Ask the mother to leave so that they counselor can speak with the daughter alone. C. Call the daughter after the session at a time when the mother is not home. D. Leave the room for a short time, and allow private discussion between the mother and daughter.

A. Acknowledge the mother's concern but inform her it is the counselor's obligation to provide accurate information

Which of the following testing methods would be most useful for a condition where most people have the same point mutation (e.g., sickle cell disease) A. Allele specific oligonucleotide hybridization B. CMA C. MLPA D. Massively parallel sequencing

A. Allele specific oligonucleotide hybridization

NSCG practice guidelines exist for which of the following neurodegenerative diseases? A. Alzheimer disease B. Parkinson disease C. ALS D. Both a and b E. Both a and c

A. Alzheimer disease

A "no-call" result with cfDNA screening refers to a situation in which no result is reported, the result is indeterminate, or uninterpretable. Which of the following statements about no-call results is NOT correct? A. Approximately 80-90% of women with initial no-call results will receive an informative result on a repeat screen B. Repeat screening may delay aneuploidy dx C. No-call results are associated with a high risk of aneuploidy D. May occur due to a low fetal fraction

A. Approximately 80-90% of women with initial no-call results will receive an informative result on a repeat screen Only 50-60% of repeat screens will provide a result Up to 23% chance of aneuploidy with no-call

Which of the following Urea Cycle Disorders does NOT typically present with severe, neonatal-onset hyperammonemia? A. Argininemia B. Citrullinemia C. Ornithine Transcarbamylase Deficiency D. Argininosuccinic aciduria

A. Argininemia

A man comes in for genetic counseling with his two daughters, ages 12 and 17. Their mother died of breast cancer at age 39 and had an identified BRCA1 mutation. The father wants his daughters to be tested for the mutation. Which of the following is the BEST next step by the genetic counselor? A. Ask the father about his motivation for requesting genetic testing of his daughters B. Discuss with the father the options for medical management should breast cancer arise in his daughters C. Explain the insurance implications of a positive result. D. Arrange genetic testing for the 17-year old daughter

A. Ask the father about his motivation for requesting genetic testing of his daughters

A 35-year old woman comes for genetic counseling after recently learning her sister has an MSH2 mutation. The woman wants testing for the familial mutation for herself and her three school-age children. Which of the following issues is MOST important to discuss regarding testing the woman's children? A. Autonomy B. Beneficence C. Nonmaleficence D. Justice

A. Autonomy

What is the main cancer risk that a mutation in the gene listed in the previous question confers? A. BCC B. Brain tumor C. Melanoma D. Pheo

A. BCC · Multiple BCCs - >5 in lifetime or 1 <30y - only cancer risk associated with the condition besides childhood medulloblastoma (PNET)

Sudden unexpected nocturnal death syndrome (SUNDS) and ________________ are considered to be the same disorder. A. Brugada syndrome B. Long QT syndrome C. Arrhythmogenic Right Ventricular Dysplasia/Cardiomyopathy (ARVD/C) D. Catecholaminergic Polymorphic Ventricular Tachycardia (CPVT)

A. Brugada syndrome

All of the following birth defects are associated with use of ACE inhibitors in the second and third trimester except? A. CHD B. Renal tubular dysplasia C. Pulmonary hypoplasia D. Potter's sequence E. Hypocalvaria

A. CHD

What is the most frequently observed structural malformation that has been reported in large studies of SSRI use in pregnancy? A. CHD B. Low birth weight C. Persistent pulmonary hypertension of the newborn D. Limb defects E. Neonatal abstinence syndrome

A. CHD Most consistent adverse finding: 2-fold increased in CHD (2% instead of GP of 1%)

What should be offered as follow-up to a first-trimester NT measurement of 3.0 mm? a. CVS or amnio b. NIPS c. fetal echocardiogram d. MSAFP

A. CVS or amnio If the NT is at least 3.5 mm, fetal echo should also be considered.

Unbalanced 14;21 translocations...? select all that apply A. Can cause down syndrome B. Have clinical phenotypes that are more severe than T21 due to nondisjunction C. Are a common type of reciprocal translocation D. Are clinically indistinguishable from nondisjunction type T21

A. Can cause down syndrome D. Are clinically indistinguishable from nondisjunction type T21 Not B. because phenotypes are indistinguishable Not C. because common type of ROBERTSONIAN translocation

All of the following regarding inversions are true except A. Carriers of large pericentric inversions face a lower risk to have unbalanced offspring than carriers of smaller inversions B. Carriers of paracentric inversions face a lower risk to have unbalanced offspring than carriers of pericentric inversions C. Unbalanced products from paracentric inversions include dicentric and acentric fragments D. Unbalanced products from pericentric inversions include duplication/deletion fragments E. None of the above (all are true)

A. Carriers of large pericentric inversions face a lower risk to have unbalanced offspring than carriers of smaller inversions

What is the underlying defect in children with Fanconi Anemia? A. Chromosomal breakage B. Mismatch repair deficiency C. RNA splicing defect D. Shortened telomeres

A. Chromosomal breakage FYI: Telomere biology disorders (dyskeratosis congenita) are caused by shortened telomeres and require telomere length studies DKC Associations: - Classic triad: oral leukoplakia (white spots), reticular skin pigmentation (lacy pigmentation of chest/neck), nail dystrophy (75% w/ at least one, <50% w/ all three) - Familial MDS/AML - Bone marrow changes with cytopenias = bone marrow failure - Pulmonary fibrosis - scarring of tissue which impairs function - Liver cirrhosis - Early onset SCC of the head/neck/anogenital tract

Which of the following risk models does NOT help to identify the prior probability for an individual to have a gene mutation? A. Claus B. Penn II C. BRCAPro D. Tyrer-Cuzick

A. Claus Breast cancer gene mutation prior probability models: - CaGene (BRCAPro) - Penn II - BOADICEA - Tyrer-Cuzick - Hughes Risk App

Patient has an SDHD mutation and a family history of head and neck PGLs. What is the patient's risk of developing a paraganglioma if she inherited the gene mutation from her mother? A. Close to 0 B. 0.2 C. 0.5 D. 0.8

A. Close to 0

A 28-year old female is seen by a genetic counselor during her second pregnancy because she reports that her first pregnancy was diagnosed with thanatophoric dysplasia. No records are available to confirm this. In order to substantiate the diagnosis, it is BEST to ask whether the fetus has which of the following in addition to shortened limbs? A. Cloverleaf Skull B. Fractures C. Ambiguous genitalia D. Polydactyly

A. Cloverleaf Skull

Which of the following is not an aneuploidy screening test with both a first and second trimester blood draw? A. Combined screen B. Sequential screen C. Integrated screen D. Contingent screen

A. Combined screen Combined first trimester screen - Incorporation of NT measurement Sequential - NT measurement at 10-14 weeks - First and second trimester bloods drawn - Results reported after both part 1 and part 2 analyzed Integrated - NT measurement at 10-14 weeks - First and second trimester bloods drawn - Results reported only after part 2 analyzed Contingent - NT measurement at 10-14 weeks - First trimester bloods drawn and results reported - Part 2 blood draw only for patients at intermediate risk (Highest risk patients referred for diagnostic testing; Lowest risk patients do not go on for second blood draw)

Which of the following genetic principles is generally NOT applicable to cardiovacular genetic disorders? A. Complete penetrance B. Allelic heterogeneity C. Limited fam hx may be due to premature death D. Phenotypic heterogeneity

A. Complete penetrance

A 60-year old woman who was diagnosed with breast cancer at age 36 and ovarian cancer at age 50 is being seen for genetic counseling. Her mother was diagnosed with breast cancer at age 55, a maternal aunt was diagnosed with ovarian cancer at age 60; and a maternal first cousin was diagnosed with leukemia at age 20. The proband underwent comprehensive germline sequencing analysis of BRCA1 and BRCA2 at the time of her ovarian cancer diagnosis with negative results. Which of the following additional genetic tests would be MOST appropriate for the proband to have? A. Comprehensive germline large rearrangement analysis of BRCA1 and BRCA2 B. BRCA1 and BRCA2 promoter methylation analysis of either of her tumors C. Comprehensive germline PTEN genetic testing D. p53 promoter methylation analysis of either her tumors

A. Comprehensive germline large rearrangement analysis of BRCA1 and BRCA2 (BART)

For a newborn presenting with distal arthrogryposis, genetic testing should include sequencing of genes that cause: A. Congenital myasthenic syndromes B. DMD C. Fascioscapulohumeral dystrophy D. Type 2 MD

A. Congenital myasthenic syndromes When you see distal arthrogryposis, congenital myasthenic syndromes should be on your differential

A 25-year old pregnant woman comes for genetic counseling and amniocentesis because her father had Becker muscular dystrophy (BMD). Ultrasonography shows a male fetus. Her father's mutation is known. Test results show that neither the woman nor her fetus carries the BMD mutation. The woman is not aware that her father is not her biological father. Which of the following is the BEST course of action for the genetic counselor? A. Consult with the ethics review board regarding the best course of action B. Tell the woman that the results indicate that the fetus is not affected. C. Arrange a session for the woman and her mother to discuss the non-paternity D. Contact the woman's OB with test results and let him inform the patient of the non-paternity and unaffected status of her fetus.

A. Consult with the ethics review board regarding the best course of action

Which amino acid can affect the tertiary structure of a protein by forming a covalent bond with the protein? A. Cysteine B. Glutamine C. Glycine D. Proline

A. Cysteine Disulfide bridges, covalent bonds formed between two cysteine residues, further reinforce the shape of a protein. Disulfide bridges form when the sulfhydryl groups of two cysteine residues come into close contact because of protein folding.

The diagnosis of Meckel-Gruber syndrome is suspected in a stillborn fetus with multiple anomalies. Which of the following feature is MOST significant to establish this diagnosis? A. Cystic renal disease B. Renal agenesis C. Occipital encephalocele D. Postaxial polydactyly

A. Cystic renal disease of the 3 major characteristics: - occipital encephalocele - cystic kidneys - postaxial polydactyly Cystic kidneys are most indicative of Meckel-Gruber

Which of the following is not a key element of pretest counseling for CMA? A. Discussion of future research studies B. Detection rate C. Testing limitations D. Potential results

A. Discussion of future research studies

Which of the following types of epithelial ovarian cancer increases the suspicion for a mutation in one of the mismatch repair genes the most? A. Endometrioid B. High-grade serous C. Low-grade serous D. Mucinous

A. Endometrioid

A couple whose child had a positive NBS for galactosemia is seen for genetic counseling. Follow-up testing showed slightly reduced GALT enzyme activity, gal-1-P and galactitol levels within normal limits and one copy of the Q118R mutation in the GALT gene. Which of the following is MOST important to discuss with the parents? A. Explain that screening tests are not diagnostic B. Recommend the enzyme studies be repeated C. Discuss the management of galactosemia D. Describe the purpose of newborn screening.

A. Explain that screening tests are not diagnostic Heterozygote identified

What is the most prevalence Fanconi Anemia gene? A. FANCA B. FANCC C. FANCD D. FANCD1

A. FANCA (60-70%) - highest proportion by a long shot Cytogenetic loci: 16q24.3

A patient reports a personal hx of "carpet of polyps" in his colon and extra teeth. Which of the following is the most likely dx? A. FAP B. LS C. PJS D. None of the above

A. FAP Other features that can be seen in FAP · CHRPE - congenital hypertrophy of the retinal pigment epithelium · Supernumerary teeth, epidermoid cysts, osteoma (20%)

Patient has a personal hx of lung blebs and a fam hx of chromophobe kidney cancer. Based on this history, which cancer susceptibility gene is most likely? A. FLCN B. MET C. VHL D. NF1

A. FLCN Birt-Hogg-Dube Major features: - Skin lesions (fibrofolliculomas and trichodiscomas) - 90% - Lung cysts/blebs - 90% (Risk for spontaneous pneumothorax - 25-30%) - Renal tumors - 33% (Renal oncocytomas, chromophobe renal cell carcinoma, hybrid tumors)

Mutations in GLA result in A. Fabry disease, which can present with HCM B. Noonan syndrome, which can present with HCM C. Danon disease, which can present with HCM D. Wolff-Parkinson-White (WPW) syndrome, which can present with HCM

A. Fabry disease, which can present with HCM Syndromic inheritance of HCM: - Fabry (XL) - Danon syndrome can also present with HCM but caused by LAMP2 (XL) PVs - Also some mitochondrial and AR genes

Given the characteristics of a screening test, which of the following would NOT be an example of a screening test? A. Fetal echocardiogram B. Maternal age C. First trimester biochemical screening D. Nuchal translucency ultrasound

A. Fetal echocardiogram

Which is not a characteristic trigger of arrhythmia in the most common types of Long QT syndrome? A. Fever B. Exercise C. Emotion D. Rest/sleep

A. Fever

The insertion or deletion of a single nucleotide into the protein coding portion of a gene is a A. Frameshift mutation B. Missense mutation C. Nonsense mutation D. Silent mutation

A. Frameshift mutation

Which factor(s) lead to the founder effect? A. Geographic isolation B. Heterozygote advantage C. Random mating D. All of the above E. Both a and b

A. Geographic isolation Why are certain ethnicities at increased risk for certain AR conditions? - Founder Effect - Centuries of marrying/having children within the same population - Heterozygote advantage (Sickle cell carriers resistant to malaria)

Which of the following conditions would not be diagnosed by WES? A. HD B. Marfan C. Cowden D. Noonan

A. HD

Which of the following is true about an individual who has sickle cell trait? A. He or she would have resistance to malaria B. He or she would have difficulty at high altitude C. He or she would have mild symptoms of sickle cell disease D. All of the above E. None of the above

A. He or she would have resistance to malaria Carriers typically don't show symptoms

A 38-year old African American woman comes for genetic counseling at 10 weeks gestation. She is not accompanied by her partner. She has a low mean corpuscular volume. Which of the following is the BEST next step for the counselor to recommend? A. Hemoglobin electrophoresis, iron and lead studies in the woman B. Complete blood count for the woman C. Complete blood count for the partner D. Hemoglobin electrophoresis, iron and lead studies in the partner

A. Hemoglobin electrophoresis, iron and lead studies in the woman Low mean corpuscular volume id'd on CBC

Which of the following statements about SMA is true? A. In SMA1, onset is before 6 mo B. In SMA2, children develop both sitting and walking C. In SMA 3, the lifespan is usually less than 50y of age D. In SMA 4, onset is in adolescence

A. In SMA1, onset is before 6 mo

Which of the following tests is recommended by ACOG for all pregnant patients who present for care in the first trimester? A. Integrated first and second trimester screening B. FTS C. Second trimester screening D. Nuchal translucency measurement E. All of the above

A. Integrated first and second trimester screening Integrated includes: - NT measurement at 10-14 weeks - First and second trimester bloods drawn - Results reported only after part 2 analyzed - DR (DS = 94-96%; T18 = 91-96%) - Though recommended, rarely occurs due to timing of care, NIPT, etc. ACOG PB #77 (2007): - "All women should be offered aneuploidy screening before 20 weeks of gestation, regardless of maternal age." - "Integrated first- and second-trimester screening is more sensitive with lower false-positive rates than first-trimester screening alone....and should be offered to women who seek prenatal care in the first trimester." - "...information about the detection and false-positive rates, advantages, disadvantages, and limitations, as well as the risks and benefits of diagnostic procedures, should be available to patients so that they can make informed decisions."

What is the role of topoisomerase? A. It relieves torsional strain in the DNA helix B. It removes damaged bases from the DNA helix C. It replaces mismatched base pairs in the DNA helix D. It unwinds the DNA helix

A. It relieves torsional strain in the DNA helix

You are starting a clinical lab that will offer testing for a new syndrome. Which of the following is not necessary in order to receive CLIA licensing? A. Lab must list their CPT codes on their published materials B. Lab must meet regulatory requirements for timeliness C. Lab must validate testing procedures D. Lab reports must include interpretation of results

A. Lab must list their CPT codes on their published materials CLIA requires: - Fulfills regulatory requirements for accuracy, timeliness, reliability - Quality control for reagents, equipment, environment, personnel - Validation of testing procedures (can known mutations be identified) - Regular proficiency testing (CAP exam for cyto labs) - Documentation/communication (Report must include interpretation, what is known/not known about clinical value of test, any comments on patient specific results.)

Which of the following findings are NOT associated with a pregnancy at increased risk for Down syndrome? A. Low hCG B. Low PAPP-A C. Increased nuchal translucency D. Increased maternal age

A. Low hCG High hCG

Which of the following hereditary cancer syndromes are NOT typically offered to children (under age 18)? A. Lynch syndrome and MEN1 B. Hereditary PGL/PCC syndrome and VHL C. FAMMM and MEN2 D. LFS and DICER1

A. Lynch syndrome and MEN1 LS - not typically, screening begins in adulthood (colonoscopy at age 20-25) MEN1 - not typically, although biochemical screening can begin in childhood based on clinical features MEN2 - yes, · very early onset MTC (surgery <1yo) PGL/PCC - yes, screening begins at age 10y VHL - yes hemangioblastoma screening at age 5y (optho exam) and 10y (brain MRI), biochemical screening for PCC/PGL starting at age 5y FAMMM - yes (early onset skin cancer --> recommend derm exam at least 2x per year beginning age 10y) LFS - yes (childhood onset cancers, esp. leukemia) DICER1 - yes (childhood lung tumor onset = pleuropulmonary blastoma)

What group of mitochondrial disease can cause the cell to not make enough mitochondrial DNA? A. Mitochondrial depletion syndromes B. Mitochondrial deletion syndromes C. Mitochondrial myopathy syndromes D. None of the above

A. Mitochondrial depletion syndromes POLG functions in both mtDNA replication and repair (~150 known mutations, ~3% of all primary mitochondrial disease) POLG has three functional regions causing different disease when affected: - Exonuclease (proofreading) --> mistakes can cause mtDNA deletions/point mutations and PEO - Linker (links between polymerase and exonuclease region) --> mistakes cause mtDNA deletions and ataxia - Polyperase (replication) --> mistakes cause mtDNA DEPLETION and Alpers syndrome (seizure, dementia, regression, cortical blindness, liver disease) mtDNA copy number analysis is how you test for mtDNA depetion

Which of the following statements about splicing is true? A. More than half of human mRNAs undergo alternative splicing B. All genes in humans have at least one intron that needs to be removed by splicing C. Splicing can result in epigenetic changes in gene expression D. Efficient splicing requires a polypurine tract just upstream of the 5' splice site

A. More than half of human mRNAs undergo alternative splicing 75%

Which of the following is FALSE about coronary artery disease? A. Most often the result of a single gene mutation B. Most often multifactorial C. Risk factors include sedentary lifestyle, smoking, obesity, and high-fat diet D. Genetic testing is generally not a part of routine care at this time

A. Most often the result of a single gene mutation

Which of the following genes is NOT on the recommended secondary findings list? A. NF1 B. BRCA1 C. TSC1 D. NF2

A. NF1

All of the following are true about neonatal lupus except A. Neonatal lupus is true lupus which occurs perinatally B. Skin rash C. liver problems D. Usually resolves by 3yo E. Low blood cell counts F. All of the following are true

A. Neonatal lupus is true lupus which occurs perinatally It is passed from mother to baby across placenta (not the fetus's lupus, but the mother's)

A 12-year old girl is referred for a genetics evaluation to rule out a syndrome after having been diagnosed with colon polyps. The abnormal results of which of the following would be MOST supportive of a diagnosis of Familial Adenomatous Polyposis (FAP)? A. Ophthalmologic evaluation B. Audiologic testing C. Brain MRI D. Abdominal ultrasound

A. Ophthalmologic evaluation CHRPE

A mutation in which of the following genes increases the chance of having a child with Fanconi Anemia? A. PALB2 B. STK11 C. CDH1 D. ATM

A. PALB2 Biallelic mutations of PALB2, BRIP1 and RAD51C can also lead to FA - Affected individuals are compound heterozygotes

What are the 3 major conditions caused by a common mtDNA deletion? A. Pearson, Kearns-Sayre syndrome (KSS), and Progressive external opthalmoplegia (PEO) B. Maternal diabetes, MELAS, and Leigh syndrome C. Both a and b D. None of the above

A. Pearson, Kearns-Sayre syndrome (KSS), and Progressive external opthalmoplegia (PEO) Pearson, KSS, and PEO all caused by 4.9 kb common deletion MELAS (point mt-DNA tRNA mutations) - Low % heteroplasmy (<30%) causes diabetes mellitus NARP caused by two main point mt-ATP6 mutations (T8993G/C). Either in high amounts lead to Leigh disease

A 32-year old pregnant woman is referred for genetic counseling at 6 weeks gestation because of alcohol exposure. She explains that she drank several alcoholic beverages daily for one week during her honeymoon 4 weeks ago. She expresses concern that this exposure will cause fetal alcohol spectrum disorder (FASD). Which of the following is the BEST response to this patient? A. Reassure her that the risk for FASD is not increased given the timing of the exposure B. Reassure her that the risk for FASD is not increased given the limited duration of the exposure C. Confirm that there is an increased risk for FASD since there is no known safe amount of alcohol consumption during pregnancy D. Confirm that there is an increased risk for FASD and probably other birth defects because organogenesis has begun.

A. Reassure her that the risk for FASD is not increased given the timing of the exposure

An individual with a strong history of late-onset Alzheimer disease contacts you for information about APOE genetic testing. Which of the following is NOT a recommended course of action? A. Recommend DTC labs that provide APOE analysis B. Provide genetic counseling following the Huntington disease predictive testing model C. Obtain a detailed 3 generation fam hx D. Inform the individual that there are no medical or lifestyle interventions to reduce risk of Alzheimer disease

A. Recommend DTC labs that provide APOE analysis APOE analysis NOT CURRENTLY recommended under any circumstances

An ophthalmology exam performed on someone who has a VHL mutation is looking for which of the following A. Retinal angioma B. Retinal hamartoma C. Optic nerve tumor D. Optic melanoma

A. Retinal angioma Hemangioblastoma risk: 60-84% (includes retinal angiomia which is a hemangioblastoma of the eye) Other major cancer risks: - clear cell RCC - Pheos

The clustering of complex disorders within families seen in family studies A. Reveals nothing definitive about the etiology of the condition B. Demonstrates unequivocally that genetics are an important contributor to etiology C. Demonstrates unequivocally that shared environment is an important contributor to etiology D. Both b and c

A. Reveals nothing definitive about the etiology of the condition

Mutations in which gene make up 15-30% of Brugada syndrome (the greatest proportion compared to any other gene) A. SCN5A B. CACNA1C C. SCN1B D. KCNE3

A. SCN5A

According to a recent study comparing biochemical sequential screening to NIPS cell free DNA analysis, which test identified more chromosomal abnormalities? A. Sequential screening identified over 10% more chromosomal abnormalities than did cell free DNA B. Cell free DNA analysis identified over 10% more chromosomal abnormalities than did sequential screen C. The two screens identified the same number of chromosomal abnormalities D. Neither test identified the majority of chromosome abnormalities

A. Sequential screening identified over 10% more chromosomal abnormalities than did cell free DNA Because NIPT detects only T13/18/21 and SCA; whereas, sequential screening can identify rare aneuploidies, large del/dups, etc. (HOWEVER, sequential screening is not validated to do so, just can get an abnormal test (MoM value) when underlying cause is any chromosome abnormality)

The definition of ID includes which of the following features? A. Significant limitations in intellectual functioning B. Onset before 5y C. Deficits in social interaction and communication D. All of the above. E. A and B

A. Significant limitations in intellectual functioning

A 25-year old pregnant woman comes for genetic counseling. Her screening results are positive for an increased risk Smith-Lemli-Opitz (SLO) syndrome. Her results are as follows: AFP: 1.00 MoM HGC: 0.95 MoM UE3: 0.10 MoM Inhibin: 1.10 MoM In addition to counseling the woman about SLO, which of the following should be discussed? A. Steroid sulfatase deficiency B. Trisomy 21 C. Biotinidase deficiency D. Trisomy 18

A. Steroid sulfatase deficiency Due to extremely low uE3 and pattern not concerning for T21 or T18

According to ACMG, known PVs in which of the following genes should be offered as a reported secondary finding in WES? A. TSC2 B. CFTR C. ATM1 D. CDH1

A. TSC2

Most nucleotides in an mRNA chain have a 5'-3' linkage except A. The base at the 5' end of the mature mRNA which has a 5'-5' linkage B. The base at the 5' and 3' exon junction what has a 1'-5' linkage C. The base at the 3' end of the mature mRNA which has a 2'-3' linkage D. The base at the translation start site which has a 3'-3' linkage

A. The base at the 5' end of the mature mRNA which has a 5'-5' linkage Why? The capping process (5' end) is comprised of three enzymes: - Phosphatase which removes one phosphate group from the 5' end of the growing chain. - Guanyl transferase which transfer a GMP through a 5'-5' linkage to the 5' end. - Methyl transferase which adds a methyl group to the 7 position of the guanine cap.

Which of the following is true about CF? A. The disorder is seen in all ethnic backgrounds B. Carrier frequency does not vary between ethnic backgrounds C. Detection rate using mutation analysis is consistent across ethnic backgrounds D. ACOG recommends screening only in Caucasian and AJ populations E. Both B and D are correct

A. The disorder is seen in all ethnic backgrounds

In the initial discussion of a prenatal diagnosis of Down syndrome, which of the following is NOT essential information? a. fetal loss rate for DS is 75% between CVS and term, and 50% between amnio and term b. children with DS are more like other children than they are different c. raising a child with DS may involve more time commitment than typical children d. individuals with DS can live independently or in a group home and can be employed competitively or in a workshop setting

A. The fetal loss rates for DS are 78% of recognized pregnancies; 32% between CVS and term; and 25% between amnio and term.

You are seeing a couple for prenatal GC for a positive CVS result for SMA. Both parents are identified as carriers of SMA (SMN1 = 1) as part of the GP screen (neither has a positive fam hx). The fetal result is reported as SMN1 = 0 and SMN2 = 2. You counsel the couple that... A. The fetus will have SMA but clinical phenotype could be type 1, 2, or 3 B. The fetus will have type 2 SMA since there are two copies of the SMN2 gene C. The fetus will have type 1 SMA since there are zero copies of the SMN1 gene D. The fetus will have type 1 SMA since that is the most common type of SMA

A. The fetus will have SMA but clinical phenotype could be type 1, 2, or 3

Which of the following statements is TRUE? A. The mtDNA genome includes fewer genes than the nuclear genome B. The mtDNA genome does not undergo meiosis, similar to the nuclear genome C. The mtDNA genome has 2 copies per cell, similar to the nuclear genome D. The mtDNA genome demonstrates biparental inheritance, similar to the nuclear genome

A. The mtDNA genome includes fewer genes than the nuclear genome A. Yes, 37 genes in mitochondrial genome compared to >20,000 genes in nuclear genome B. No, No meiosis for mtDNA, but meiosis for nuclear DNA. FYI, mtDNA undergoes continuous replication, not synchronized to cell cycle C. No, Copy number not fixed in mtDNA (2-10 genomes per mitochondria, and many mitochondria per cell), nuclear genome has only 2 copies per cell D. No, Maternal inheritance of mtDNA

Achondroplasia shows AD inheritance with complete penetrance. Matt has achondroplasia, but an otherwise negative fam hx. His wife Mary is of normal stature, but her father had Achondroplasia. Matt and Mary have two unaffected children. Which of the following statements is most accurate? A. The probability that their next child will be affected is 1/2. B. The probability that each of their future children will be affected is less than 1/2 because Matt most likely has a new mutation C. Since Mary has an affected parent, her children are at increased risk to be carriers D. If their third child is unaffected, the the probability that their fourth child will be affected is 1/8

A. The probability that their next child will be affected is 1/2.

RNA interference refers to A. The use of small RNAs bound to a protein complex to downregulate translation B. The use of double stranded RNAs to control imprinting C. The use of tRNAs to block translation initiation D. The use of rRNAs to reduce transcription

A. The use of small RNAs bound to a protein complex to downregulate translation Why? RNA interference is used to downregulate translation, not transcription, and uses small RNAs (miRNA and siRNA) to the jobs

The specific heteroplasmy load for a specific mtDNA mutation in any given tissue tolerated before showing signs of pathogenicity is known as? A. Threshold effect B. Heteroplasmy C. mtDNA copy number variation D. None of the above

A. Threshold effect

Ms. T is G3P0SAB2, LMP 10/7. Ms. T had a hysterosalpingogram (X-ray of uterus and fallopian tubes with radiocontrast) on 10/17. The primary reason that this radiologic procedure does not increase the risk to her pregnancy is due to the following principle A. Timing B. Dose C. Genetic susceptibility D. Tissue access E. Identified pattern of malformations

A. Timing (all or none period)

Which of the following statements are false: A. Trisomy 20, like all aneuploidies, can be easily detectable in fetal blood B. Aneuploidy is the leading genetic cause of ID C. Aneuploidy is the leading known cause of pregnancy loss D. All trisomy 16 causes are caused by maternal origin of aneuploidy

A. Trisomy 20, like all aneuploidies, can be easily detectable in fetal blood *T20 not present in blood

You are consulted on a child in the NICU who is suspected to have a metabolic condition. The resident tells you the patient is a 7-day old male whose parents brought him into the ER after two days of poor feeding, lethargy and a seizure early this morning. He has been very lethargic and his liver panel testing came back showing some liver damage. What other biochemical testing would be important to obtain in diagnosing this child? A. Urine Orotic Acid B. Fasting glucose C. Serum amino acid panel D. Lactate levels

A. Urine Orotic Acid

Muscle weakness is A. a descriptive dx B. a transient problem that can be ignored C. easy to distinguish from hypotonia in children D. indicative of a particular genetic dx

A. a descriptive dx

Gowers' sign is A. a sign of proximal muscle weakness B. a sign of distal muscle weakness C. Dx for limb girdle muscular dystrophy D. Dx for DMD

A. a sign of proximal muscle weakness

Which of the following clusters of terms accurately describes DNA as it is generally viewed to exist in eukaryotic cells? A. double-stranded, antiparallel B. double-stranded, parallel C. single-stranded, parallel D. triple-stranded, antiparallel

A. double-stranded, antiparallel

Which of the following is a study used to record the electrical activity of the heart using electrodes attached to the skin? A. electrocardiogram B. Transthoracic echocardiogram C. Transesophageal echocardiogram D. Color Doppler study

A. electrocardiogram

Which of the following types of conditions are NOT included on the minimum list of secondary findings to report according to ACMG's recent guidelines? A. epileptic encephalopathies B. cancer predisposition syndromes C. cardiac conduction disorders D. connective tissue disorders

A. epileptic encephalopathies

The chance for recurrence of a complex disorder among family members of an affected individual is higher when the affected individual A. experienced earlier than typical age at onset B. is of the more frequently affected sex C. experienced less severe illness D. all of the above

A. experienced earlier than typical age at onset

For an individual with mosaic trisomy 18, which two are the most likely reproductive outcomes? A. healthy child B. child with mosaic T18 C. child with full T18 D. child with T21

A. healthy child followed by C. child with full T18

Neonatal liver failure and encephalopathy (brain dysfunction) are most consistent with which mitochondrial disease? A. mtDNA depletion B. MELAS C. MERRF D. Kearns-Sayre

A. mtDNA depletion

All statements below are true about mitochondrial DNA (mtDNA), except A. mtDNA undergoes meiosis B. mtDNA has a high mutation rate compared to nuclear DNA C. mtDNA contains no introns D. mtDNA is inherited maternally

A. mtDNA undergoes meiosis

Which of the following is sensitivity? A. true positives/(true positives + false negatives) B. true negatives/(true negatives + false positives) C. true positives/(true positives + false positives) D. true negatives/(true negatives + false negatives)

A. true positives/(true positives + false negatives) affected / all affected

The couple in #8-9 ask, given their consanguinity, what is the risk that the baby will have a significant birth defect? a. 3-4% b. 6% c. 10% d. up to 40% risk

A: near population risk. 3rd degree relatives: 6%, 2nd degree relatives: 10%, 1st degree: up to 40%

A genetic disorder affects a great-grandfather, grandfather, mother, and daughter in the same bloodline. What is the most likely pattern of inheritance? AR AD Mito XL-D

AD

Huntington disease

AD fatal, progressive neurological disorder - Age of onset varies: usually in 30s-40s Three main types of symptoms in Huntington disease: ◦ physical symptoms, including involuntary movements and clumsiness ◦ emotional symptoms, including depression and irritability ◦ cognitive symptoms, including loss of ability to recall information, loss of attention and difficulty with decision making.

Deformation MEMORY: Think of deformation and disruption together - Disruption --> destructive, halts development of a body part that would've differentiated normally (amniotic band) - Deformation --> alters shape/structure (body part still present, just deformed, Potter sequence, twin constraint)

ALTERATION in the shape/structure of a body part that has differentiated normally - caused by non-disruptive mechanical forces (Potter sequence, twin constraint, PRS)

An inborn error of metabolism is detected in fraternal twins (one boy and one girl), but does not affect their older brother. There is no fam hx of the condition. What is the most likely pattern of inheritance? AR AD Mito XL-D

AR

Most dystroglycanopathies such as Fukuyama congenital muscular dystrophy is inherited in what pattern?

AR

For the couple in #8, what is the chance that the baby will inherit two recessive alleles for an autosomal recessive locus?

AR = F/2 = 1/64

A child is diagnosed with Fanconi Anemia. What is the most common inheritance pattern? What other inheritance pattern do you need to consider?

AR, although one XL-R (FANCB) Remember chart - first one is FANCA (most common, 60-70%) - next one FANCB - XL

Which of the following clinical features is generally not seen in CF? Chronic respiratory infections Male sterility Obstructive liver disease Atrial septal defects

ASDs

WT surveillance?

Abdominal u/s every 3mo until 5-8y

What feature underlies many of the Rasopathies?

Abnormal lymphatics

Dysplasia

Abnormal organization of cells into tissues as a consequence of generalized defect in differentiation or growth (e.g., achondroplasia involves abnormal signaling during cartilage differentiation)

Polar negatively charged (acidic) amino acids

Acidic amino acids contain a side chain that functions as an acid and is capable of becoming ionized by donating a hydrogen atom to the surrounding environment. Aspartic acid, glutamic acid

Coping behaviors: distancing

Acs as if nothing has happened (e.g., "well I didn't really learn anything new today" after high risk gene identified and discussed)

Selection against AD mutations: Achondroplasia example

Affected individuals have only 1/5 of the number of children than unaffected individuals have. Therefore, f = 0.2 and s = 0.8, and 80% of individuals with achondroplasia are the result of new mutation. *If the fitness of an individual improves through medical improvements, etc., then selection against the mutant allele will decrease, and the frequency of the mutant allele will increase.

Sensitivity

Affected with positive test / all affected 1 - sensitivity = false negative rate

Referral is warranted for assessment for cancer predisposition for which patients? [as many as apply] a. woman diagnosed with a pheochromocytoma and CNS hemangioblastoma b. woman diagnosed with a pheochromocytoma and clear cell RCC c. woman diagnosed with a pheochromocytoma; father with prostate cancer d. woman diagnosed with a pheochromocytoma; mother with pheochromocytoma

All A. yes, concern for VHL B. yes, concern for VHL C. yes, all individuals with pheo should be offered genetic testing D. yes, concern for PGL/PCC predisposition

Marfan can present in Neonatal period Childhood Adulthood All of the above

All of the above

What condition is caused by mutations in FBN1 Gelophysic dysplasia Marfan syndrome Weill Marchesani syndrome All of the above

All of the above

Which of the following is a management recommendation for individuals with Marfan? Use of beta blockers Regular ophthalmology appointments Prophylactic surgery on the aorta All of the above

All of the above

MEN2B has a 50% de novo rate, and includes which of the following features: (select all that apply) a. Mucosal neuromas b. Marfanoid features c. Medullary thyroid cancer d. Medullated corneal nerve fibers

All of the above Also ganglioneurmatosis in the GI tract, Pheos, and large lips

Which of the following in one individual is enough to warrant assessment for cancer predisposition? [select all that apply] a. desmoid tumor b. endolymphatic sac tumor c. gastrinoma d. retinal hemangioblastoma

All of them because of specific associations, although they do not meet clinical dx criteria for each condition on their own A. 10-20% risk for desmoid tumor in FAP B. ELST part of VHL dx criteria but need one other feature to meet dx C.Zollinger-Ellison syndrome (subtype of MEN1) - gastrinomas, pancreatic islet cell tumors, and/or peptic ulcers D. need 2 hemangioblastomas (any location, retinal or CNS) + one other feature for VHL clinical dx

Expanded carrier screening (pan-ethnic)

Also called universal carrier screening, this type of screening allows for a large number of conditions to be screened at the same time, regardless of the patient's reported ethnicity or family history. Some of the included conditions may be rare, and therefore less information is available on them. Additionally, some diseases can have implications for individuals who are carriers of the disease. Typical expanded panel options include: - An expanded panel of ~60 conditions in which there is at least a 1% chance to be a carrier. The conditions on this panel are all AR with a known impact early in life, including intellectual and physical disabilities. - An expanded panel of ~175 or more conditions, many of which are autosomal recessive with impact early in life, while others may have milder impact or later onset. In addition, several XL conditions that may be carried by females and affect males are included.

What is the most common cause of demetia?

Alzheimer's disease

Nonpolar (hydrophobic), neutral (uncharged) amino acids

Amino acids containing uncharged hydrocarbon groups or benzene rings as side chains are non-polar. - Neutral side chains exhibit neither acidic nor basic qualities. - Although the bulk of amino acids are neutral, the ones that are both non-polar and neutral are alanine, glycine, isoleucine, leucine, methionine, phenylalanine, proline and valine. Normally buried inside the protein core I saw (ISO) LEUcy METHodically PRObe and PHEel ALAN and then VAL TRYPped Isoleucine, Leucine, Methionine, Proline, Phenylalanine, Alanine, Valine, Tryptophan

What can cause echogenic bowel other than: GI malformations, genetic conditions, or infections?

Amniotic bleeding. The baby swallows it and it shines bright like a diamond

During which phase of meiosis do the homologous chromosomes separate?

Anaphase I Sister chromatids separate in anaphase II

A genetic disorder affects a great-grandfather, grandfather, mother, and daughter in the same bloodline. The great-grandfather, grandfather, and mother displayed muscle weakness, cataracts, cardiac conduction abnormalities, and difficulty relaxing their grip during day-to-day activities, like gripping a doorknob. At birth, the daughter has severe hypotonia and difficulty breathing. What phenomenon is demonstrated here? Phenocopies Imprinting Skewed x-inactivation Anticipation

Anticipation

Discriminative accuracy/predictive ability

Area under the curve (AUC) for a receiver operator characteristic (ROC) curve Higher AUC = better discrimination = better prediction y-axis is sensitivity x-axis is 1 - specificity

Arrhythmia - Bradycardia - Tachycardia

Arrhythmia - abnormal heart beat • Bradycardia: Excessively slow heartbeat • Tachycardia: Excessively rapid heartbeat

Gene regulation: methylation

Associated with decreased gene expression - DNA methylation in vertebrates is limited to cytosine residues. - The methylated C residues occur in the sequence 5'-CG-3' (often called CpG) - Both C residues are symmetrically methylated. - Methylation does not alter base pairing specificity; instead, methyl group PROTRUDES into major groove *Many genes have CpG "islands" in promoter region

Defense mechanism: identification

Assuming the attitude or behavior of an idealized person or group "my sister says it's foolish to worry about a few ultrasound glitches"

Triplet repeat genetic disorders

At least 30 genetic disorders arise through expansion of specific trinucleotide repeat sequences. These unstable regions of the genome often show genetic anticipation: an increased severity of phenotype in successive generations. Increasing severity typically due to increase in number of repeats. ◦ Can also see earlier age of onset Ex: Fra(X) = unstable CGG in FMR1, most common form of inherited ID when >200 copies

which of the following is NOT a clinical feature of Alport syndrome? Hematuria Renal failure HL Ataxia

Ataxia Major features - Hematuria - Proteinuria leading to end stage renal disease - SNHL begins in late childhood - Ocular: Anterior lenticonus

Defense mechanisms (9) MEMORY: UPRRRIIDD --> like get rid of

Attempts to cope as a way of maintaining some measures of personal control and self-esteem to reduce painful emotions in the face of a threat - Temporary protection - Not all coping behaviors are defense mechanisms - Can be beneficial in the short-term or harmful Undoing Projection Rationalization Regression Repression Identification Intellectualization Denial Displacement

Clinical manifestations of MFS involve all of the following systems except Ocular Pulmonary Auditory CV

Auditory

Defense mechanism: Intellectualization

Avoiding intolerable feelings through abstract, precise thinking with little or no feeling "So, it is a statistical probability that with this type of translocation a fetus would expire"

A couple is seen for history of 2 SABs. What is the chance that the couple with have a chromosome abnormality? A. 0.5% each so 1% as a couple B. 3% each so 6% as a couple C. 6% each so 12% as a couple D. 12% each so 24% as a couple

B. 3% each so 6% as a couple

In prenatal clinic, a patient tells you that her mother was diagnosed with endometrial cancer. Which of the following additional pieces of information would be enough to suggest that the patient's mother be evaluated in the cancer genetics clinic? [select all that apply] a. the mother was diagnosed at age 55 b. the mother was diagnosed at age 55 AND her brother had colon cancer c. the mother was diagnosed at age 55 AND her brother had melanoma d. the mother was diagnosed at age 55 AND her brother had pancreatic cancer

B and D Now, not true because need to by dx <50, not <55 Pancreatic and CRC are LS-related cancers

What percentage of children with T21 have a robertsonian translocation? A. Less than 1% B. 3-4% C. 10-11% D. 24-25% E. >25%

B. 3-4%

Screening for neural tube defects can be done between __ and __ weeks' gestation. a. 10 and 13 b. 15 and 20 c. 16 and 18 d. 18 and 20

B. 15-20 w is the ACMG recommended window but they do say that 16-18 is optimal

In prenatal clinic, a patient tells you that she has had colon polyps. Which of the following additional pieces of information would be enough to refer the patient to be evaluated in the cancer genetics clinic? [select all that apply] a. she had 5 adenomatous colon polyps. b. she and her brother have each had 5 adenomatous colon polyps. c. she had 3 juvenile polyps d. she had 1 Peutz-Jeghers type polyp.

B, C A - NO. Need at least 10 in personal/fam hx B - YES. personal or fam hx of 10 or more adenomatous polyps is a testing indication for polyposis C - YES. One dx criteria for JPS = 3-5 juvenile polyps of colon D - NO. Need 2 or more PJS polyps (specific hamartomatous type) without fam hx

Under what circumstance(s) is paternal carrier screening recommended? [select as many as apply] a. concurrently with pre-conception maternal carrier screening b. concurrently with maternal carrier screening occurring at 15w gestation c. after maternal carrier screening shows carrier status for an AR condition d. pre-conception if there is significant paternal family history that screening could address

B, C, D. For pre-conception, maternal testing is recommended; then paternal testing can follow if needed. During pregnancy, after 14w, concurrent maternal and paternal testing is recommended to allow time for decision making.

A lab GC is writing result report materials for an expanded carrier screening panel. What should be included? [select as many as apply] a. opt-out for adult-onset disorders b. residual risk for each negative result c. description of the calculation of the residual risk, including the population carrier frequency and the proportion of disease-causing alleles detected on the specific testing platform d. calculation of the fetal risk if the reproductive partner declines testing

B, C, D. Pre-test should include (A) opt-out for adult-onset disorders

Signs of macular degeneration occur in ~30% of individuals over 75. Concordance in monozygotic twins is 37%, and concordance in dizygotic twins is 19%. First-degree relatives of patients are at a 4.2-fold greater risk for disease compared with the general population. Select all that apply: a. heritability is 37% b. there is a prominent genetic contribution to the disorder c. third degree relatives are at close to population risk d. risk for first-degree relatives depends on the severity of the disease in the affected relative

B, C, and D

A 25-year-old woman at low-risk for fetal aneuploidy presents for prenatal care. She is at 15w6d and has had no prenatal care yet. She has no concerning personal or family history. What should be offered for screening/testing for chromosome aneuploidy? [select as many as apply] a. triple screen b. quad screen c. penta screen d. CVS e. amnio

B, C, and E

What is the approximate penetrance of pulmonary arterial hypertension with a BMPR2 mutation? A. Less than 10% B. 0.2 C. 0.5 D. Greater than 90%

B. 0.2 (20%)

BRCA1/2 account for what % of hereditary breast cancer? A. 0.1 B. 0.5 C. 0.9 D. 0.8

B. 0.5 ~50% of hereditary breast cancer, ~90% of HBOC

A couple has a son with an abnormal karyotype: 46,XY,i(21)(q10). Woman is 39 and man is 41. Which of the following is closest to the RR for the condition? A. 0% B. 1% C. 50% D. 100%

B. 1%

A non-consanguineous Caucasian couple is referred for genetic counseling. The woman's brother died of cystic fibrosis. The risk for the couple to have a child with cystic fibrosis is closest to: A. 1/100 B. 1/150 C. 1/250 D. 1/400

B. 1/150

The frequency of DiGeorge/VCF syndrome is? A. 1/1000 B. 1/2000 C. 1/5000 D. 1/10000 E. 1/50000

B. 1/2000

What % of 22q del are inherited from a parent that may or may not be symptomatic? A. 5% B. 10% C. 30% D. 50%

B. 10%

A couple was counseled because their first child was born with a ventricular septal defect. There is no family history of any congenital heart disease and the parents have normal cardiac examinations. They should be counseled that the risk that a subsequent child will have similar or other cardiac defects is closest to: A. <1% B. 3-5% C. 8-10% D. 25%

B. 3-5% - Full sib of isolated case: 2-3% - Offspring of affected father (2-3%) - Offspring of affected mother (5-6%)

Which three post-transcriptional modifications are often seen in the maturation of mRNA in eukaryotes? A. 3'-capping, 5'-poly(A) tail addition, splicing B. 5'-capping, 3'-poly(A) tail addition, splicing C. Heteroduplex formation, base modification, capping D. Removal of exons, insertion of introns, capping

B. 5'-capping, 3'-poly(A) tail addition, splicing

The amount of radiation exposure above which a prenatal exposure is of concern is A. 1 millirad B. 5-10 rads C. None D. 25 rads E. 10 millirads

B. 5-10 rads

The empiric risk of a female carrier of a 14/21 robertsonian translocation having a child with T21 is estimated to be? A. <5% B. 5-10% C. 15-20% D. 25-30% E. ~33%

B. 5-10% But really its 10-15% so answer B is closest without over estimating

A newborn presents with microcephaly, hypotonia, and a high-pitched cry. Which of the following is most likely to be identified on CMA? A. 4p deletion B. 5p deletion C. 5q deletion D. 8p deletion E. 11p deletion

B. 5p deletion Cat's cry cri du chat = 5 words Test-taking pointer: p because all except one answer have p, not q

Which of the following statements about SMN, the gene involved in SMA, is true? A. There are two copies of the gene, one on chromosome 8 (SMN1) and one on chromosome 12 (SMN2) B. 95% of affected individuals have a homozygous deletion of SMN1 C. 10% of SMN2 undergoes alternative splicing to give a truncated protein D. A more severe phenotype is typically seen when there are multiple copies of SMN2

B. 95% of affected individuals have a homozygous deletion of SMN1

Discussion of artificial insemination with donor sperm is MOST appropriate for a couple with: A. A daughter with cleft lip and palate B. A daughter with limb girdle muscular dystrophy C. Son with Becker Muscular Dystrophy D. Son with Kearns-Sayre Syndrome

B. A daughter with limb girdle muscular dystrophy A. No because multifactorial most often B. Yes because AD or AR and most likely to be inherited from dad C. No because XL and therefore inherited from mom D. No because (see below) Kearns-Sayre syndrome: Generally not inherited but arises from mutations in the body's cells that occur after conception (somatic mutation). Rarely, this condition is inherited in a mitochondrial pattern, which is also known as maternal inheritance. Because egg cells, but not sperm cells, contribute mitochondria to the developing embryo, children can only inherit disorders resulting from mtDNA mutations from their mother.

SMA is most commonly characterized by A. A homozygous duplication of SMN1 B. A homozygous deletion of SMN1 C. A homozygous duplication of SMN2 D. A homozygous deletion of SMN2 E. Deletions in both the SMN1 and SMN2 genes

B. A homozygous deletion of SMN1

FTD has illustrated which of the following inheritance patterns? A. AR B. AD C. XL D. Both a and b are correct E. all of the above are correct

B. AD - Parkinson's disease and ALS is all of the above (PD, mixed; ALS mostly AD, but rare AR/XL) - Huntington's, FTD, and Early-Onset Familial Alzheimer disease (EOFAD) are AD or AD reduced pen

Dilated cardiomyopathy most often displays which inheritance pattern? A. AR B. AD C. XL D. Mitochondrial

B. AD All cardiomyopathies display majority AD inheritance

RYR1 gene mutations can cause A. AR malignant hyperthermia B. AD malignant hyperthermia C. AD myofibrillar myopathy D. AR adult onset central core disease

B. AD malignant hyperthermia

Homozygotes of which of the following genes have a condition called Ataxia Telangiectasia? A. PALB2 B. ATM C. CHEK2 D. BRCA1

B. ATM

Which of the following is true about Prader-willi syndrome? A. It is seen when the maternal genes in 15q11-15q13 are absent or non-functional B. About 75% of cases are caused by paternal deletions of 15q11-q13 C. The same genes imprinted in PWS are imprinted in Angelman syndrome D. PWS shows anticipation

B. About 75% of cases are caused by paternal deletions of 15q11-q13 Etiology of PSW: Several genes present in region that are only expressed from paternal chromosome - 65-75% have a deletion of this region on the paternal chromosome - 20-30% have maternal UPD - <3% have an imprinting defect (0.5% with deletion in imprinting center)

A couple comes for preconception genetic counseling because the woman had bilateral retinoblastoma. The husband asks about his wife's risks for developing the non-ocular cancers associated with this disease. The counselor invites the woman to answer in an attempt to assess her level of understanding and perception of risk. She responds in anger, "Do you mind not talking about this!?!" Which of the following is the MOST appropriate response by the genetic counselor? A. Explain that her reaction may be a healthy coping mechanism B. Acknowledge her anger and ask her to elaborate on her reaction C. Discuss information regarding her increased risk of additional cancers. D. Offer to talk to the husband later in private and redirect the conversation

B. Acknowledge her anger and ask her to elaborate on her reaction

A child is born with several birth defects and undergoes genetic testing. The baby is found to carry an unbalanced rearrangement. What is the most likely segregation pattern that occurred in this pregnancy? A. Alternate segregation B. Adjacent 1 C. Adjacent 2 D. 3;1

B. Adjacent 1 Why? - Alternate produces balanced gametes (normal chrom to one gamete and translocated chrom to other) - Adjacent 1 = unbalanced and most frequent mode of malsegregation - Adjacent 2 = unbalanced and UNCOMMON mode of segregation - 3;1 = 3 chrom to one gamete and 1 to the other

Which of the following is an electrical device that is placed to monitor for and correct cardiac arrhythmia by delivering a shock? A. Holter monitor B. Automatic implantable cardioverter defibrillator C. Pacemaker D. Cardiac stress test

B. Automatic implantable cardioverter defibrillator

Patients who have been diagnosed with adenocarcinoma of the pancreas and also have 2 or more relatives with pancreatic cancer should be offered panel testing that includes which of the following genes? A. APC, BAP1, CDKN2A B. BRCA2, CDKN2A, MSH2 C. BAP1, BRCA2, MEN1 D. CDNK2A, CDKN2B, MEN1

B. BRCA2, CDKN2A, MSH2 Risk for pancreas cancer by syndrome (genes): o HBOC (BRCA1/2): ~3% o FAMMM (p16/CDKN2A): up to 17% o HP (PRSS1, SPINK1, CFTR): up to 40% o PJS (STK11): 36% o Lynch (MMR): 3-9% o FAP (APC): ~3% o MEN1: pancreatic neuroendocrine/islet cell carcinoma, NOT adenocarcinoma BAP1 - cutaneous and uveal melanoma, BCC, and malignant mesothelioma

A 45-year old man is referred for genetic counseling because he has pancreatic cancer. Review of his medical record indicates that he also had a melanoma. He is MOST likely to have a mutation in which of the following genes? A. BRCA1 B. CDKN2A C. PTEN D. TSC2

B. CDKN2A

A pediatric patient presents with apparently nonsyndromic DD and ID. Which of the following should be considered as a first-tier test? A. Methylation testing B. CMA C. WES D. Karyotype E. B and C

B. CMA

A 9yo child presents with coexistence of joint laxity in the hands, contractures at the elbows and ankles, and mild weakness. CK levels are normal. Which of the following is the most likely dx? A. CMT disease, type 4C B. COL6 related myopathy (Ulrich/Bethlem myopathy) C. Juvenile type 1 MD D. Merosin deficient congenital MD

B. COL6 related myopathy (Bethlem/Ulrich myopathy), BECAUSE CK can be normal to slightly elevated and phenotype of joint laxity with contractures fits

Which of the following cancer is not a major criteria for Cowden syndrome? A. Breast B. CRC C. Thyroid D. Endometrial

B. CRC

Which of the following is NOT true regarding Fragile X intermediate allele carriers? A. Family members are at increased risk to be premutation carriers B. Carrier is at increased risk to have a child with a premutation C. Carrier is at increased risk to have a child with a full mutation D. Prenatal dx is available if desired by the patient E. All of the above are true

B. Carrier is at increased risk to have a child with a premutation

Population stratification must be considered in the design of GWAS studies because A. The same OR has to be obtained when the GWAS is conducted in two different populations. B. Cases and controls must be from similar populations to account for baseline allele frequency C. you must account for the frequency of the disease in the general population D. It ensures clinical utility E. none of the above

B. Cases and controls must be from similar populations to account for baseline allele frequency Why? Population stratification: - Cases and controls need to be from similar populations (ethnicity) to ensure baseline allele frequencies don't differ

Maternal conditions that do NOT increase the risk of miscarriage: A. Obesity B. Chronic HTN C. Diabetes D. Alcohol use E. Epilepsy F. All of the above increase SAB risk

B. Chronic HTN Is associated with risk of maternal morbidity if pre-eclampsia develops

Which is the best testing method to determine mtDNA heteroplasmy? A. Sanger sequencing B. NGS C. Mitochondrial disease panel D. None of the above

B. NGS Can detect heteroplasmy as low as 1.2% Sanger only as low as 50%

Which of the following statements about cfDNA is not correct? A. Fetal cell free DNA is primarily of placental origin - from the placental trophoblast cells B. Circulating cfDNA of fetal origin can be identified in maternal plasma for many years after delivery C. Circulating cfDNA of fetal origin comprises ~3-15% of total cfDNA after 10 weeks gestation D. cfDNA of fetal origin was first discovered in maternal plasma in 1997 E. The first prenatal cfDNA screen was clinically available in 2011

B. Circulating cfDNA of fetal origin can be identified in maternal plasma for many years after delivery After delivery, cf placental DNA is rapidly cleared from the maternal circulation, making it specific to that pregnancy.

Movement of a protein to the lysosome requires A. ATP dependent transport out of the nucleus B. Cotranslational transport into the ER C. Passive transport across the nucleolus D. Vesicular transport through the mitochondrial matrix

B. Cotranslational transport into the ER Co-translational synthesis definition: - A signal sequence on a growing protein will bind with a signal recognition particle (SRP). This slows protein synthesis. The SRP then binds to a location on the surface of the nearby ER. Then, the SRP is released, and the protein-ribosome complex is at the correct location for movement of the protein through a translocation channel. Any proteins that are destined for a lysosome are delivered to the lysosome interior when the vesicle that carries them fuses with the lysosomal membrane and joins its contents.

Which of the following enzymes needs a 3' end in order to act? A. Primase B. DNA polymerase C. Helicase D. Topoisomerase

B. DNA polymerase DNA polymerase has 5′-3′ activity and cannot start a strand de novo. All known DNA replication systems require a free 3′ hydroxyl group (OH) before synthesis can be initiated ◦ Short RNA primers are synthesized by primase. ◦ Primase does not need a 3' end, only needs a template strand (note: the DNA template is read in 3′ to 5′ direction whereas a new strand is synthesized in the 5′ to 3′ direction—this is often confused

During interphase of the cell cycle A. DNA recombines B. DNA replicates C. RNA replicates D. sister chromatids move to opposite poles

B. DNA replicates

Self-destructive behavior and sleep disorder is reported in your patient with dysmorphic facial features. Which of the following is the most likely dx? A. Duplication on the p arm of chromosome 17 B. Deletion on the p arm of chromosome 17 C. Deletion of the q arm of chromosome 17 D. Deletion of the q arm of chromosome 22 E. None of the above

B. Deletion on the p arm of chromosome 17 Smith-magenis

The most common chromosomal microdeletion syndrome is? A. Angelman B. DiGeorge/VCF C. Miller-Dieker D. Prader-Willi E. Williams

B. DiGeorge/VCF

Ms. H is G3P0SAB2, LMP 10/7. Ms. T had a hysterosalpingogram (X-ray of uterus and fallopian tubes with radiocontrast) on 11/12. The primary reason that this radiologic procedure does not increase the risk to her pregnancy is due to the following principle A. Timing B. Dose C. Genetic susceptibility D. Tissue access E. Identified pattern of malformations

B. Dose

The most common symptom of PAH is A. Syncope B. Dyspnea C. Reynaude's phenomenon D. Fatigue

B. Dyspnea

A 40-year old pregnant woman and her husband come for genetic counseling to discuss amniocentesis. A previous pregnancy ended in the miscarriage of a fetus with trisomy 18. The woman is reluctant to undergo amniocentesis. Her husband is in favor of the procedure; however, he states that it is her decision because she is carrying the pregnancy. Which of the following is the BEST next step? A. Give the couple time alone to decide what they want to do. B. Facilitate a discussion between the couple about the decision C. Initiate the informed consent process D. Offer to review the risks and benefits of amniocentesis a second time.

B. Facilitate a discussion between the couple about the decision

Fetal brain development is dependent upon maternally-derived thyroid hormone after 10weeks gestational age: True or False

B. False *Before 10 weeks

Mutations in the LDL receptor gene are associated with which of the following? A. Familial hypertension B. Familial hypercholesterolemia C. Familial orthostatic hypotension D. Hereditary hemorrhagic telangiectasia

B. Familial hypercholesterolemia

A consult is requested for a 12-year old girl in the ER who presents with liver dysfunction, hyperammonemia, heart arrhythmia, and lethargy after she developed flu-like symptoms almost 3 days prior. Her mother reports that she has had a difficult time eating during her illness but the more severe symptoms did not present until this morning. Her pertinent health history consists of 2 febrile seizures at 1 year of age. Which category of inherited metabolic disease would be MOST appropriate to investigate in this patient? A. Amnio Acidopathies B. Fatty acid oxidation disorders C. Mitochondrial disease D. Glycogen storage disorder

B. Fatty acid oxidation disorders

You are consulted on a child in the NICU who is suspected to have a metabolic condition. The resident tells you the patient is a 3-day old female who was admitted to the NICU approximately 12 hours after birth due to poor feeding, failure to thrive, hemolytic anemia, congential cataracts. She has been very lethargic and developed sepsis caused by E. coli. In reviewing the newborn screen, which result should you pay most attention to? A. Amino acidopathies B. Galactosemia C. Organic Acidemias D. Fatty acid oxidation disorders

B. Galactosemia Due to congenital cataracts

Which genes are found on the short arm of acrocentric chromosomes? A. Genes for 5S RNA B. Genes for 45S RNA C. Genes for RNA polymerase III D. Genes for tRNA

B. Genes for 45S RNA Why? - 45S rDNA is found in clusters on the five acrocentric chromosomes: 13, 14, 15, 21, 22

Which one of the following statements about genomic imprinting is true? A. Genomic imprinting affects a large fraction of the genes in mammalian genomes B. Genomic imprinting involves the silencing of one allele of a gene C. Genomic imprinting is necessary for X-inactivation D. Genomic imprinting occurs at multiple life stages

B. Genomic imprinting involves the silencing of one allele of a gene

Male children with Denys-Drash syndrome or Frasier syndrome have a high likelihood of having which of the following anomalies? A. Duplicated renal collecting system B. Gonadal anomalies C. Thoracic/rib anomalies D. Thumb or upper limb anomalies

B. Gonadal anomalies Both conditions can have ambiguous or female genitalia in 46,XY male and risk for gonadoblastoma

Unlike RNA, DNA A. Has a ribose sugar in the backbone B. Has thymine instead of uracil C. Is single stranded D. Has equal amounts of A and G

B. Has thymine instead of uracil

Which of the following tests cannot differentiate between an individual with sickle cell trait and an individual with sickle cell disease, and therefore should not be used as a carrier screening test? A. Hemoglobin electrophoresis B. Hemoglobin solubility C. Hemoglobin fractionation D. Beta-globin gene sequencing E. None of the above

B. Hemoglobin solubility

Having more than one type of mitochondrial DNA (mtDNA) in a cell is known as? A. Homoplasmy B. Heteroplasmy C. Mosaicism D. None of the above

B. Heteroplasmy

Which of the following results would be expected in a second trimester pregnancy at highest risk for T21? A. High hGC, high inhibin A, high AFP, low estriol B. High hGC, high inhibin A, low AFP, low estriol C. High hGC, high inhibin A, low AFP, low PAPP-A D. Low hGC, low inhibin A, low AFP, low estriol E. Low hGC, high inhibin A, high AFP, low estriol

B. High hGC, high inhibin A, low AFP, low estriol

The same patient changes her mind and says that CVS would not be an option. Now what maternal serum screening should be considered? [select as many as apply] a. combined first trimester screening b. integrated screening c. stepwise sequential screening d. contingency screening

B. Highest detection rates (but does not report results until 2nd trimester)

Cystic fibrosis shows AR inheritance with an incidence of 1/2500. Paul and Ann are referred to genetic counseling because Paul's sister Kate has CF. Assume that the population is in HW equilibrium. Which of the following statements is accurate? A. The probability that Paul is a carrier is 1/2 B. If Paul has a child with Anna the probability that the child will be affected is 1/150 C. If Paul has a child with his first cousin the probability that the child will be affected is 1/12 D. If Kate has a child with her first cousin the probability that the child will be affected is 1/4

B. If Paul has a child with Anna the probability that the child will be affected is 1/150 A. prob is 2/3 B. Correct, 1/25 (chance Anna is carrier) x 1/2 (chance she passes on mutant allele) x 2/3 (chance Paul is carrier) x 1/2 (chance Paul passes on mutation allele) = 1/150 C. First cousins share 1/8 of DNA x 2/3 x 1/2 = 1/24 already just for Paul's prob so wrong D. 1/25 x 1/25 x 1/2 x 1/2 is much more than 1/4

A 38-year old woman come for genetic counseling because of an abnormal first trimester screen with an increased risk for the fetus to be affected with Down syndrome She listens to the counselor explain the additional prenatal screening and testing options, and at the end, tells the counselor, "I don't know what I should do." Which of the following is the BEST ways for the counselor to approach the situation? A. Restate the testing and screening options in simpler terms B. Inquire about how the woman has made difficult decisions in the past C. Convince the woman that the testing will clarify the risk D. Explain why the choice for testing is entirely up to the patient.

B. Inquire about how the woman has made difficult decisions in the past

Which of the following symptoms would lead you to more highly suspect a metabolic condition in a patient who is referred to your genetics clinic due to "Marfanoid habitus"? A. Pectus deformity B. Intellectual disabilities C. Family history of thoracic aortic aneurysm D. Personal history of lens dislocation

B. Intellectual disabilities

During a genetic counseling session, a 27-year old woman is informed that she has a BRCA1 gene mutation. The woman does not seem distressed. She asked detailed questions about the molecular nature of the mutation, the cancer risks associated with the BRCA1 mutation, and the medical management options. She uses correct genetic and medical terminology such as "autosomal dominant Mendelian inheritance," "protein truncation," and "stop codon." This patient appears to be coping with the genetic test results using which of the following defense mechanisms? A. Denial B. Intellectualization C. Projection D. Reaction formation

B. Intellectualization

Which of the following is not a population with increased risk for thalassemia? A. African American B. Japanese C. Hispanic D. Itailian E. Fillipino

B. Japanese HBB (B-thal and SCD): AA: 1 in 11 AJ: 1 in 67 Eastern Asia (Japan): 1 in 51 Finland: 1 in 80 French Canadian/Cajun: 1 in 67 HISPANIC: 1 in 25 Middle East: 1 in 23 Native American: 1 in 67 Northwestern Europe: 1 in 32 South Asia: 1 in 23 SOUTHEAST ASIA (Phillipines): 1 in 23 SOUTHERN EUROPE (Italy): 1 in 14

Which breast cancer pathology is associated with mutations in CDH1? A. Invasive ductal breast cancer B. Lobular breast cancer C. DCIS D. None of the above

B. Lobular breast cancer

A 44-year old man with colon cancer comes for genetics counseling. He has no family history of cancer. His pathology report indicates that his tumor is MSI-High with the following immunohistochemistry (IHC) results: MLH1 - present MSH2 - present MSH6 - present PMS2 - present During the pre-authorization process, the genetic counselor learns that the patient will have to pay for follow-up molecular testing out of pocket and can afford testing for only two genes. In addition to MLH1, which of the following genes should be tested to identify the causative mutation? A. EPCAM B. MSH2 C. MSH6 D. PMS2

B. MSH2 Next most likely gene

Which of the following hereditary CRC syndromes has an AR inheritance? A. FAP B. MUTYH-Associated Polyposis B. AFAP D. PJS

B. MUTYH-Associated Polyposis

Mutations in which genes each make up about 40% of HCM? A. TNNT2 and TNNI3 B. MYH7 and MYBPC3 C. MYL2 and MYL3 D. ACTC1 and ACTN2

B. MYH7 and MYBPC3

A pregnant woman whose son has ornithine transcarbamylase (OTC) deficiency comes for genetic counseling. A detailed ultrasound at 18 weeks shows a normal female fetus. Which of the following is the MOST important information to collect to assess risk in the current pregnancy? A. Results of her partner's carrier testing B. Maternal symptoms of hyperammonemia C. Orotic acid and ammonia levels in her son D. Maternal diet history

B. Maternal symptoms of hyperammonemia OTC deficiency is XL, but carrier females can show symptoms due to skewed x-inactivation. If female is affected, mom should show hyperammonemia In males and females suspected of having partial OTC deficiency who have normal molecular testing and normal or borderline urinary orotic acid concentration under normal conditions, an allopurinol challenge test should be performed. A markedly abnormal increase of orotic acid excretion after administering allopurinol is diagnostic

A 4yo girl presented to clinic with delayed motor milestones (e.g., walked at 22 mo, ran at 3y). She had very elevated CK (=4500) and non-specific white matter changes on brain MRI, but normal cognitive development. Which of the following is the most likely dx? A. Walker-Warburg syndrome B. Merosin deficient congenital MD C. MD type 2 D. Bethlem myopathy

B. Merosin deficient congenital MD (AR) A mild presentation of what you typically imagine for congenital MD

A patient has a personal hx of CRC and a sebaceous carcinoma and is found to have a hereditary cancer syndrome. Which of the following is the most likely dx? A. FAP B. Muir-Torre syndrome C. Gardner syndrome D. MAP

B. Muir-Torre syndrome Gardner syndrome is for FAP with CHRPE, osteomas, and supernumerary teeth Turcot can be used to id Lynch or FAP dependent on brain tumor - Lynch (glioblastoma) - FAP (medulloblastoma) MEMORY: FAP has an M in it and Lynch does not

Which of the following is generally NOT helpful in making a diagnosis of Brugada syndrome? A. EKG B. Myocardial biopsy C. Review of fam hx D. Review of clinical hx

B. Myocardial biopsy

Regarding Rh alloimmunization, the following is true: A. Occurs in Rh+ mother carrying an Rh- fetus B. Occurs in Rh- mother carrying an Rh+ fetus C. RhoGAM (anti-D immune globulin) is given to Rh sensitized women at 28 weeks GA D. Most commonly occurs in the antepartum period

B. Occurs in Rh- mother carrying an Rh+ fetus C. Not true because already sensitized *RhoGAM also given at delivery D. most commonly occurs at delivery (only 1-2% antepartum)

Which of the following is NOT true about carriers of a premutation in the Fragile X (FMR1) gene? A. Females are at increased risk for premature ovarian failure B. Only males are at increased risk for Fragile X ataxia tremor syndrome (FXTAS) C. Females are at increased risk of having a child with a full mutation D. Males are at increased risk of having a grandchild with a full mutation E. All of the above are true

B. Only males are at increased risk for Fragile X ataxia tremor syndrome (FXTAS)

People with Xeroderma Pigmentosum have high risks of developing cutaneous skin cancer as well as high risks of developing cancers of the? A. Brain and pancreas B. Oral mucosa and tongue C. Pancreas and thyroid D. Thyroid and tongue

B. Oral mucosa and tongue Also risk of brain tumors and lung cancer if smoker, but not HIGH like it is for cutaneous, oral mucosa, and tongue skin cancers

Which of the below nuclear genes is one of the most common cause for primary mitochondrial disease and related to the following symptoms: epilepsy, Leigh syndrome, Alpers, ataxia, and PEO? A. ND3 B. POLG C. OPA1 D. None of the above

B. POLG

The type of renal cancer associated with the syndrome listed in the previous question is? A. Papillary type 1 B. Papillary type 2 C. Chromophobe D. Clear cell

B. Papillary type 2 HPRC (MET) is associated with papillary type 1

Inversions that do not involve the centromere are A. contiguous B. Paracentric C. Pericentric D. Reciprocal E. Robertsonian

B. Paracentric

All of the following have been identified as possible explanations for false positive results with cfDNA screening except A. Placental mosaicism B. Paternal aneuploidy C. Maternal malignancy D. Vanishing twin E. Maternal aneuploidy

B. Paternal aneuploidy

Which category of carrier screening focuses on common disorders seen in multiple populations? A. Ethnicity-based B. Population-based C. Pan-ethnic D. Aneuploidy E. None of the above

B. Population-based Expanded carrier screening = pan-ethnic

Which of the following is NOT a protective factor for ovarian cancer? A. OCP B. Post-menopausal HRT C. Breast feeding D. Hysterectomy

B. Post-menopausal HRT - risk factor Protective factors: o OCP o Tubal ligation o TAH o Breastfeeding o Parity >2

All of the following are high risk thrombophilias except: A. Antithrombin III deficiency B. Prothrombin G20210A mutation heterozygote C. Factor V Leiden homozygote D. Factor V Leiden and Prothrombin G20210A mutation double heterozygote E. Prothrombin G20210A homozygote

B. Prothrombin G20210A mutation heterozygote Considered low-risk because 0.5% risk compared to others which are usually about 2% or more

Which of the following is not an AD cause of early-onset AD? A. PSEN1 B. SNCA C. APP D. PSEN2

B. SNCA SNCA associated with full penetrance EO parkinson disease

ROH can be detected by? A. oligonucleotide array B. SNP array C. WES D. BAC array

B. SNP array

If you have a patient with clinical features consistent with NF1, what is the best, first approach to molecular testing? A. WES B. Sequencing of NF1 C. MLPA of NF1 D. Methylation studies

B. Sequencing of NF1

Which types of mutations are found with MLPA assays? A. Nonsense mutations B. Small deletions within one gene C. Trinucleotide repeat expansion D. Large deletions covering several genes

B. Small deletions within one gene

A 35-year old healthy woman comes for genetic counseling for cancer risk assessment. Her mother died with breast cancer at age 43. Two maternal aunts and one maternal uncle are all cancer-free. Her maternal grandmother had stomach cancer in her 60s. The consultant's father is healthy at 55. The father's siblings are cancer-free, but his brother's daughter was diagnosed with cervical cancer in her 30s. The consultant's paternal grandfather had colon cancer in his 60s. Pathologic confirmation of which reported cancer will have the largest impact on the risk assessment for this patient? A. Breast cancer B. Stomach cancer C. Cervical cancer D. Colon cancer

B. Stomach cancer

Which one of the following transcription factors contains a subunit that binds to the TATA box? A. TFIIB B. TFIID C. TFIIE D. TFIIH

B. TFIID Roles: - TFIIB: positions RNPII properly at transcription start site by recognizing BRE element - TFIID: subunit = TFB; Recognizes TATA box - TFIIE: Attracts and regulates TFIIH - TFIIH: Unwinds DNA at transcription start site; phosphorylates C terminal domain of RNPII to allow elongation; releases RNPII from promter

A woman who is seen for genetic counseling contacts the genetic counselor after the session. She states that she wants her history of two prior elective abortions removed from her medical record. After acknowledging her request, which of the following is the BEST response by the genetic counselor? A. Alter the record as requested. B. Tell her that the medical record cannot be altered. C. Forward her request to the medical records office. D. Suggest a follow-up session to discuss the matter.

B. Tell her that the medical record cannot be altered.

A patient of maternal/paternal AJ ancestry has a mother with a known founder mutation in BRCA1. Which of the following is the MINIMUM testing that should be offered to this patient? A. Single-site testing for the mother's BRCA1 mutation B. Testing for all 3 AJ founder mutations C. BRCA1/2 sequencing and del/dup analysis D. Multi-gene panel testing

B. Testing for all 3 AJ founder mutations

You work for Company A designing a test for condition B. Your test has a sensitivity and specificity that are each 99%. The literature says that the overall frequency of the condition is 1/1000. You have just found out that in your population, the actual frequency is 1/100,000. What is the biggest consequence of this change? A. The PPV goes up, improving the test's marketability to medical professionals B. The PPV goes down, harming the test's marketability to medical professionals C. The NPV goes up, improving the test's marketability to medical professionals D. The NPV goes down, harming the test's marketability to medical professionals

B. The PPV goes down, harming the test's marketability to medical professionals Lower prevalence, meaning PPV goes down

Which of the following is TRUE about familial TTR amyloidosis? A. Caused by mutations in the BMPR2 gene B. The V122I mutation is common the African American population C. Inheritance pattern is autosomal recessive D. De novo mutations occur in less than 1% of cases

B. The V122I mutation is common the African American population TTR-associated amyloidosis is a group of diseases characterized by slowly progressive peripheral sensorimotor neuropathy and autonomic neuropathy, cardiomyopathy(cardiac amyloidosis), vitreous opacities (eye floaters), and CNS amyloidosis (buildup of amyloid) Onset 3rd - 7th decade Gene: TTR (AD) - 66% de novo - two common mutations: V30M Portugese, Swedish, and Japanese V122I African American

Which of the following is TRUE about genetic testing for familial hypercholesterolemia (FH)? A. There is no genetic testing clinically available because FH is a multifactorial disease B. The benefit of genetic testing for FH is that screening can begin early for elevated cholesterol levels and risk factors can be modified before onset of disease C. Genetic testing for FH has no clinical utility D. FH is an AR disorder

B. The benefit of genetic testing for FH is that screening can begin early for elevated cholesterol levels and risk factors can be modified before onset of disease AD disorder and genetic testing is clinically available/useful

A young man with PKU, who was successfully treated following dx on NBS is planning to start a family with his healthy, unaffected, and unrelated partner, who has no fam hx of PKU. PKU shows AR inheritance with an incidence of 1/10,000. Assume that the population is in HW equilibrium. All of the following statements about risk are false EXCEPT A. The probability that his partner is a carrier is 1/100 B. The probability that the man will transmit a mutant allele to his first child is 1/2 C. The probability that his partner will transmit a mutant allele to their first child is 1/50 D. The probability that their first child will be affected is 1/100

B. The probability that the man will transmit a mutant allele to his first child is 1/2 A. Carrier freq is 1/50, not 1/100 C. Probability is carrier freq (1/50) x prob of passing on mutant allele (1/2) = 1/100 D. Prob is 1/100 x 1/2 = 1/200

~90% of retinoblastoma occur sporadically. In the remaining cases, there is a positive fam hx, AD transmision and ~80% (=0.8) penetrance. A woman with retinoblastoma, whose father and grandfather were also affected, has an unaffected partner with no relevant family history. Considering her children, which of the following statements is most accurate? A. The probability that their first child will be affected is 0.25 B. The probability that their first child will be affected is 0.4 C. The probability that each of their children will be affected is 0.5 D. The probability that each of their children will be affected is 0.4 x 0.5

B. The probability that their first child will be affected is 0.4 0.5 (prob of inheriting Rb allele) x 0.8 (penetrance) = 0.4

The smallest amount of exposure to the developing baby is likely to occur with this route of administration A. Oral B. Topical C. IV D. Rectal E. Inhalation

B. Topical

Which of the following is a non-invasive cardiac test? A. Transesophageal echo B. Transthoracic echo C. Intracardiac pressure measurement D. Heart catheterization

B. Transthoracic echo

The relative contributions of genes and environment to a condition can be quantified by using A. Family studies B. Twin studies C. Linkage studies D. Association studies

B. Twin studies

Which of the following statements is incorrect? A. Cytogenetic abnormalities occur in 2% of pregnancies B. Whole chromosome abnormalities (aneuploidies) are phenotypically more severe than microduplications/deletions C. Diagnostic/invasive testing is associated with a miscarriage risk of 1/500-700 D. All of the above statements are incorrect

B. Whole chromosome abnormalities (aneuploidies) are phenotypically more severe than microduplications/deletions UT SAB rate: - 1/100 CVS - 1/300 amnio Baylor: 1/900 across the board Everywhere is different! Blah

Which of the below findings would NOT be considered a secondary finding in the following WES example: WES is performed on a proband that is a 1yo male with ID and dysmorphic features? A. a suspected PV in BRCA1 B. a VUS in RAI1 associated with smith-magenis C. a heterozygous mutation in HEXA associated with Tay Sachs D. a mutation in MEN1 associated with multiple endocrine neoplasia, type 1

B. a VUS in RAI1 associated with smith-magenis Why? - A and D are on ACMG's cancer predisposition list - C is a carrier status for an AR disease

Pharmacogenomic testing for the HLAB*5701 allele prior to treatment of HIV with abacavir A. is used for determining accurate dosing B. can reduce immunologically-confirmed hypersensitivity reactions by almost 100% C. is most important for African Americans D. is highly cost effective E. has an NPV of 48%

B. can reduce immunologically-confirmed hypersensitivity reactions by almost 100%

Post-test counseling for carrier testing for SMA should discuss ___. [select as many as apply] a. for a negative result: NPV is high due to the test's 99% detection rate b. for a negative result: there is residual risk due to a 2% de novo rate c. for a positive result: genotype-phenotype information for the identified mutation d. for a positive result for one partner: recommendation to test the other partner, and preliminary calculation of fetal risk based on a population carrier frequency of 1/400 - 1/600

B. for a negative result: there is residual risk due to a 2% de novo rate Not (A): the test's detection rate is 90% (only tests for SMN1 exon 7 deletions, since 95% of affected individuals are homozygous for that deletion), so there is significant residual risk for carrier status. Not (C): genotype-phenotype correlation is poor: homozygotes for the common deletion can show a whole range of severity. Not (D): yes, the partner should get tested, but the carrier frequency is 1/40 - 1/60.

Which of the following first-trimester screening patterns is most suggestive of trisomy 21? A. high free beta hCG, high PAPP-A, nuchal translucency greater than 3mm B. high free beta hCG, low PAPP-A, nuchal translucency greater than 3mm C. low free beta hCG, high PAPP-A, nuchal translucency less than 3mm D. low free beta hCG, low PAPP-A, nuchal translucency less than 3mm

B. high free beta hCG, low PAPP-A, nuchal translucency greater than 3mm

An individual who has high plasma concentrations of active drug when given a normal dose is likely to have genetic variants that A. induce rapid metabolism B. increase drug absorption C. enhance drug excretion D. increase the drug's water solubility E. improve the drug's effectiveness

B. increase drug absorption

All of the following are ways that chromosomal de novo translocations can lead to fetuses with phenotypic abnormalities except? A. breakage within a gene B. involvement of low-copy segmental repeats C. possible position effect D. subtle deletion/ duplication at the breakpoint E. UPD

B. involvement of low-copy segmental repeats

Cytochrome P450-2D6 A. influences pharmacodynamics B. is involved in the metabolism of 25% of commonly prescribed drugs C. has few polymorphisms D. must be genotyped when antidepressants are prescribed E. is primarily important for drug excretion

B. is involved in the metabolism of 25% of commonly prescribed drugs - Antidepressants, antipsychotics, analgesics, cardiovascular drugs - CYP2D6 is highly polymorphic (>100 polymorphisms) - Frequency of poor metabolizing polymorphisms varies by ethnicity

Which of the following is true regarding the use of OVA1 to determine the likelihood that an ovarian mass is malignant? A. most women who test "positive" will have an ovarian malignancy B. its use improves sensitivity but decreases specificity C. the test will result in fewer women undergoing invasive procedures D. the test will be useful in screening for BRCA carriers E. the test is not covered by Medicare

B. its use improves sensitivity but decreases specificity OVA1 = 5-protein signature for ovarian mass malignancy

Which of the following is not typically used to filter the variants found through WES? A. phenotype B. knock-out mice testing C. in silico analyses D. evolutionary conservation

B. knock-out mice testing

On a CMA, what would cause a normalized ratio of 1.5 for chromosome 21? A. mosaic T21 B. mosaic monosomy 21 C. full trisomy 21 D. full monosomy 21

B. mosaic monosomy 21

A prenatal genetic counselor is giving negative NIPS results to a patient. What should be discussed? a. the possibility of inaccurate results due to a vanishing twin b. residual risk for chromosomal abnormalities c. NIPS is not diagnostic, so confirmation by CVS or amnio is recommended d. unbalanced translocations, deletions, and duplications are not detected by NIPS

B. residual risk for chromosomal abnormalities D is a topic for pre-test counseling. A and C are for post-test counseling for a positive result.

Which of the following may not be an advisable support person for an individual undergoing predictive genetic testing? A. spouse B. sibling C. friend D. social worker

B. sibling at risk to be affected and needs their own counseling if wanting predictive testing

CMA on a child with autism detects inv(9)(p11q12). What do you counsel the parents? A. parental studies are needed B. this variant is not likely to have caused the child's autism C. this variant is not likely to have caused the child's autism, but if this individual has children there could be a risk for unbalanced offspring D. this variant likely caused the child's autism

B. this variant is not likely to have caused the child's autism

The empiric risk of a reciprocal translocation carrier having a child with a chromosomal imbalance is? A. <5% B. ~11% C. ~20% D. ~32% E. ~50%

B. ~11% Overall risk of unbalanced segregants: ~11-12% risk

Which of the following statements regarding CNVs is false? A. Most microdeletion/duplication syndromes are caused by CNVs that are >500kb B. ~45-50% of CNVs that are <500kb are benign C. Parental testing can help to determine the pathogenicity of CNVs of unclear clinical significance D. CNVs are a common type of genetic variation in the general population

B. ~45-50% of CNVs that are <500kb are benign Why? 90-95% of CNVs <500kb are benign

BRCA1/2 roles

BRCA1 and BRCA2 play essential roles in homologous recombination repair Both are also involved in regulating other proteins involved in DNA repair, cell cycle, and apoptosis

Which syndromes require Wilms tumor surveillance?

BWS (mat del 15 or pat UPD 15) WAGR (microdel 11p) SGBS (XL)

Hemoglobinopathies: molecular or biochemical studies?

Biochemical - CBC, Hemoglobin Electrophoresis, RBC indices on all patients (NOT hemoglobin solubility testing) Advantages - Inexpensive screening tests - Fast TAT - Detects majority of patients at increased risk Disadvantages - Screening test, requires follow up testing to identify true mutation carriers Molecular - PCR, mutation analysis for individual disorders Advantages - Identifies mutation carriers directly Disadvantages - More costly - Test for only a few mutations - Sequencing required for B-thal - Could miss HGB variants (C, D, E)

Defense mechanism: projection

Blaming other people or situations for difficulties the patient experiences "I know you think I'm a fool to continue this pregnancy" (really, the patient feels foolish)

What is the estimated diagnostic yield of genetic testing for channelopathies/cardiomyopathis for SIDS or and/or Sudden Unexpected Death Syndrome (SUDS)?

Both: 54% "It is estimated that 25% to 35% of SUD cases from age 1 to 35 years and up to 15% of SIDS cases may be due to mutation(s) in one of the known channelopathy-associated genes."

Which FDA teratogen category is given for a drug whose risk cannot be ruled out due to lack of studies. A, B, C, D, or X?

C

Which of the following statements about Leber Hereditary Optic Neuropathy is FALSE? A. 90% of affected individuals have one of three common point mutations in mtDNA B. 50% of males will develop vision loss C. 70% of females will develop vision loss D. Other features may include cardiac arrhythmias, myopathy, and tremors

C. 70% of females will develop vision loss Only 10% of females develop vision loss

What % of balanced reciprocal translocation carriers inherited their translocation from a carrier parent? A. 20% B. 50% C. 70% D. 90%

C. 70%

What % of Patau syndrome (T13) is due to a trisomy versus a translocation. A. 40, 60 B. 90, 10 C. 80, 20 D. 50, 50

C. 80, 20

What is the estimated de novo mutation rate of the NF2 gene? A. close to 0 B. 0.25 C. 0.5 D. 0.8

C. 0.5

BRCA1/2 account for what % of HBOC? A. 0.1 B. 0.5 C. 0.9 D. 0.8

C. 0.9

Incidence of CHD in the non-exposed or general population is A. 1/10,000 B. 1/1,000 C. 1/100 D. 1/750 E. 1/10

C. 1/100

The combined prevalence of all neuromuscular disorders is at least A. 1/30,000 B. 1/10,000 C. 1/3,000 D. 1/1,000

C. 1/3,000

A couple who are first cousins inquire about the chance of having a child affected with a genetic disorder. Their common maternal uncle has galactosemia. The chance that they will have a child with galactosemia is: A. 1/8 B. 1/16 C. 1/36 D. 1/64

C. 1/36

How may genes are present in mammalian mitochondrial DNA (mtDNA)? A. 5 B. 20 C. 37 D. 13

C. 37

What percent of first trimester pregnancy losses are chromosomally abnormal? A. ~10% B. ~25% C. ~50% D. ~75% E. ~90%

C. 50%

What is the minimum number of cafe au lair macules that is clinically diagnostic of NF1? A. 2 B. 4 C. 6 D. 10

C. 6 6 or more café-au-lait macules over 5 mm in diameter in prepubertal and over 15mm in post-pubertal

What is the risk of a CNV in a pregnancy with a structural anomaly identified by ultrasound and a normal karyotype? A. 0.5% B. 1% C. 6% D. 20%

C. 6% Wapner paper

Translation is terminated when A. The stop tRNAs bind to the large ribosomal subunit B. PolyA binding protein interacts with the small ribosomal subunit C. A stop codon is in the A site and no tRNA binds D. The initiation factors dissociate from the ribosomes

C. A stop codon is in the A site and no tRNA binds

Which statement best represents practice guideline recommendations regarding prenatal cfDNA screening A. ACOG and ACMG both recommend that prenatal cfDNA screening should be used as the first line in aneuploidy screen for all women in pregnancy B. ACOG recommends that cfDNA screening be used as the first line screen for aneuploidy while ACMG recommends that it only be used for women who have an increased risk for aneuploidy C. ACOG does not recommend cfDNA screening as a first line screen for most women; ACMG recommends that cfDNA screening be made available to all women D. Both ACOG and ACMG recommend using cfDNA screening outside of high risk populations E. None of the above

C. ACOG does not recommend cfDNA screening as a first line screen for most women; ACMG recommends that cfDNA screening be made available to all women

Which of the following analytes is used to identify a pregnancy at risk for ONTD? A. uE3 B. hCG C. AFP D. DIA E. none of the above

C. AFP

The majority of pediatric mitochondrial disease are due to what mode of inheritance? A. Sporadic B. Maternal C. AR D. None of the above

C. AR

Which of the following best describes the connection between GBA and Parkinson disease? A. An AD cause of PD with high penetrance B. An AR cause of PD C. An AD risk factor for PD with low penetrance D. A protective factor that reduces lifetime risk for PD

C. An AD risk factor for PD with low penetrance GBA: Increased risk low; ~ 5-fold increased lifetime risk for PD. Compared to SNCA (full penetrance) and LRRK2 (30-74% penetrance)

A 17-year old pregnant woman and her partner come for genetic counseling at 10 weeks gestation because she has a clinical diagnosis of Myotonic Dystrophy (DM). She has not had any genetic testing. The patient had difficulty in school and dropped out after 9th grade. After reviewing the range of severity of DM, the counselor offers genetic testing to determine her chance of having a child with congenital DM, which she declines. She states that she does not like needles and would not mind having a child who is like her. Which of the following is the BEST response by the counselor to the woman's decision to decline testing? A. Explore the partner's feelings about the woman having genetic testing for DM. B. Support her decision to decline testing, and conclude the session C. Ask her to describe her understanding of congenital DM D. Encourage her not to base her decision for testing on her fear of needles.

C. Ask her to describe her understanding of congenital DM

Mutations in which gene make up the majority of pulmonary arterial hypertension? A. ENG B. ACVRL1 C. BMPR2 D. MYBPC3

C. BMPR2 ENG and ACVRL1 (Hereditary Hemorrhagic Telangiectasia) MYBPC3 (HCM and LVNC among others)

While a genetic counselor is taking a family history, the patient reports that of her sister's four children, two have polydactyly, cognitive impairments, obesity and vision loss. Which of the following syndromes is the MOST likely explanation of their features? A. Meckel-Gruber syndrome B. Pallister-Hall syndrome C. Bardet-Biedl syndrome D. Smith-Lemli-Opitz syndrome

C. Bardet-Biedl syndrome AR condition Beetles crawl on crypts and commit crimes → CRPTCRM (primary features)- Cognitive impairment- Rod-cone dystrophy- Postaxial polydactyly- Truncal obesity- Complex GU anomalies in females- Renal abnormalities- Male hypogonadism Also Meckel is often lethal

A 6-month old child is seen in your inherited metabolic disease (IMD) clinic for a suspected IMD. The child has significant developmental delays, microcephaly, and is said to smell "mousey" by her pediatrician. Other than PKU, which other metabolic condition should be considered for this child? A. Cobalamin defect B. Biotin deficiency C. Biopterin deficiency D. Primary Carnitine deficiency

C. Biopterin deficiency 2% of infants with high phe have biopterin-related disease

Which genetic condition is characterized by a butterfly shaped rash on the face and immune deficiency? A. Ataxia-telangiectasia B. Fanconi anemia C. Bloom syndrome D. Nijmegen breakage syndrome

C. Bloom syndrome Major features - Extreme sensitivity to sun exposure - General increased risk of cancer - Genomic instability Dysmorphic features - Short adult stature, low weight - Dilated blood vessels and reddening of skin - Skin hypopigmentation and hyperpigmentation - Distinctive "butterfly" pattern of telangiectasia on face - Long narrow face; prominent nose and ears - Mild-mod immunodeficiency with recurrent infections

Which of the following is the most frequent cancer in women with LFS? A. Adrenocortical tumors B. Brain cancer C. Breast cancer D. Leukemia

C. Breast cancer

Individuals who have a personal and fam hx of melanoma and atypical moles have a high likelihood of having a mutation in which of the following genes? A. BRCA2 B. CDC73 C. CDKN2A D. PTCH1

C. CDKN2A (FAMMM) CDC73 (Familial isolated hyperparathyproidism (PHPT) or Hyperparathyroidism-Jaw Tumor syndrome) PTCH1 (neviod basal cell carcinoma syndrome (NBCCS), previously Gorlin syndrome)

Which of the following features is NOT associated with Turner syndrome? A. webbed neck B. cardiac anomalies C. CLP D. kidney anomalies E. All of the above are features of Turner syndrome

C. CLP

PSEN1 G206A is a founder mutation for which ancestry A. Mediterranean B. Eastern European Jewish C. Caribbean Hispanic D. Native American E. South American

C. Caribbean Hispanic Why? EO Alzheimer (PSEN1) has Puerto Rican (e.g., Caribbean Hispanic) founder mutations B. No, GBA and LRRK2 (Parkinsons) mutations more common in EEJ ancestry

Which of the following is FALSE regarding PARP and DNA repair? A. PARP is an enzyme in DNA repair B. Cells with BRCA1/2 mutations are deficient in homologous recombination DNA repair, but still have base-excision repair C. Cells with BRCA1/2 mutations are deficient in base-excision repair, but still have homologous recombination of DNA repair D. All of the above

C. Cells with BRCA1/2 mutations are deficient in base-excision repair, but still have homologous recombination of DN PARP inhibitors (block PARP) eliminate base-excision repair in cancer cells deficient in BRCA1/2 and spare normal cells

Which of the following is FALSE with regard to Jervell and Lange-Nielsen syndrome? A. Inheritance pattern is AR B. Characterized by LQTS and SNHL C. Characterized by LQTS and blindness D. Caused by mutations in KLQT1 (KCNQ1) and KCNE1

C. Characterized by LQTS and blindness

True protein restriction is an effective treatment for which of the following conditions? A. Phenylketonuria B. Galactosemia C. Citrullinemia D. Maple Syrup Urine Disease

C. Citrullinemia

Your patient is a 35yo woman seeking predictive genetic counseling and testing for early-onset AD. Her mother was recently dx with AD at age 58, and there is mat hx of EO dementia. She is unaware of any previous genetic testing in the family. Which of the following is the best course of action? A. Provide her with genetic counseling and testing following the HD predictive testing model B. Provide education on the limitations of APOE genotyping C. Counsel her on the importance of first pursuing genetic testing in a relative with Alzheimer disease D. Provide info on GINA

C. Counsel her on the importance of first pursuing genetic testing in a relative with Alzheimer disease

A patient presents with severe ID, CHD, and a "greek warrior helmet" craniofacial appearance. Which of these chromosomal abnormalities is most likely? A. Deletion of long arm of chromosome 4 B. Trisomy of chrom 16 C. Deletion of short arm of chrom 4 D. Deletion of short arm of chrom 5 E. None of the above

C. Deletion of short arm of chrom 4 Need shield 4 protection = 4p

A 37-year old woman is referred to the adult genetics clinic to be evaluated for Ehlers-Danlos syndrome (EDS) because she has a personal history of hypermobile joints, fibromyalgia, and inflammatory bowel disease. When the counselor is taking the family history, the woman says, "All these questions aren't important! What does this have to do with me?" Which of the following is the genetic counselor's BEST next step? A. Redirect to the goals of the session. B. Review the importance of obtaining the family history. C. Discuss her emotional response. D. Acknowledge that patients with EDS are usually anxious

C. Discuss her emotional response.

After the formation of a peptide bond, the uncharged tRNA leaves the ribosome via the A. T site B. P site C. E site D. A site

C. E site Ribosomal tRNA binding sites and peptide bond formation: - When the two ribosomal subunits combine, the P (peptidyl) site and A (aminoacyl) site tRNA binding sites are formed - The large subunit contains a third tRNA binding site, the E (exit site). - After initiation, the tRNA-met is bound at the P site - Incoming charged tRNAs load onto the ribosome at the A site (tRNA only accepted if its anticodon is complementary to the codon) - The A and P sites are close together to guarantee that continuous bases of RNA are deciphered (no base skipping) - As elongation continues, additional amino acids are added until a termination codon is reached - Uncharged tRNA is always moved to E site after peptide bond formed - SUMMARY: charged tRNA enters at A site --> Peptide bond forms at P site --> uncharged tRNA moves to E site

Using the Schwartz criteria, a dx of Long QT syndrome can be made based on A. Fam hx and genetic testing B. Heart biopsy C. EKG findings, clinical hx, and fam hx D. Cardiac MRI, exercise stress test, and clinical hx

C. EKG findings, clinical hx, and fam hx

Generally what phenotype is most likely if an individual has more than one mutation that is associated with hypertrophic cardiomyopathy? A. Embryonic lethal B. Severe dysmorphic features C. Earlier age of onset of HCM D. No difference from HCM with one mutation

C. Earlier age of onset of HCM

Which of these is the most helpful clinical feature in selecting a genetic test for a patient with Charcot-Marie-Tooth disease? A. Absence of DTRs B. Distal muscle atrophy C. Electrophysiological evidence of axonal v demyelinating disease process D. High arches and hammer toes

C. Electrophysiological evidence of axonal v demyelinating disease process

A 40-year old woman with Li-Fraumeni syndrome comes for genetic counseling after testing confirms that she has a p53 mutation. Her asymptomatic 18-year old son is also found to have the same mutation. The mother repeatedly suggests that gene therapy should be available soon to repair the gene for her son and that perhaps his test should be repeated in the case there was a mix-up at the laboratory. The mother is exhibiting which of the following coping styles? A. Confrontive B. Distancing C. Escape-Avoidance D. Positive reappraisal

C. Escape-Avoidance

The most common class of mutations in BMD is A. Continuous gene deletions B. Deep intronic mutations C. Exonic del/dups D. Missense mutations

C. Exonic del/dups

The C9ORF72 hexanucleotide expansion causes which two diseases? A. Parkinson disease (PD) and Alzheimer disease (AD) B. PD and FTD C. FTD and ALS D. ALS and AD E. None of the above

C. FTD and ALS

A 35-year old Jewish woman of Russian and Polish ancestry is referred for genetic counseling for ethnic-based carrier testing. She is MOST likely to be a carrier of which of the following conditions? A. Canavan disease B. Cystic fibrosis C. Gaucher disease D. Tay-Sachs disease

C. Gaucher disease Most common AJ AR condition

Ultrasonography of a 26-year old woman at 16 weeks gestation shows fetal findings consistent with osteogenesis imperfecta, type II. The woman has a previous fetus affected with the same condition. The woman and her husband are healthy and have no obvious clinical characteristics of osteogenesis imperfecta. Which of the following is the MOST likely explanation for this recurrence? A. AR B. De novo C. Germline mosaicism D. Incomplete penetrance

C. Germline mosaicism

A 32-year old woman comes for genetic counseling at 20 weeks gestation because she recently discovered that a 26 year old paternal cousin has a "metabolic condition". She was told by her mother that her cousin has a very "round, protruding belly and a doll-like face" as well as early onset osteoporosis and gout. The woman is estranged from her paternal family and has not ever had any contact with this cousin. The genetic counselor should look for laboratories that offer carrier testing for which of the following conditions? A. Glycogen storage disease, type IV B. Glutaric Acidemia, type II C. Glycogen storage disease, type I D. Primary Carnitine Deficiency

C. Glycogen storage disease, type I

The trinucleotide repeat in the paternal allele is the one that usually expands in A. Fragile X B. Friedreich ataxia C. HD D. Myotonic dystrophy

C. HD *Only condition in which we see anticipation for the paternally, as opposed to maternally, inherited allele

A 43yo woman seeks genetic counseling because of a recent dx of endometrial cancer. her family hx includes a mat grandma with breast cancer at age 56, father with CRC at age 47, and one pat aunt with ovarian cancer at age 49. Counseling regarding testing for which of the following is most appropriate? A. FAP B. HBOC C. HNPCC D. PTEN

C. HNPCC

Which type of polyp is associated with PJS? A. Adenomatous B. Hyperplastic C. Hamartomatous D. Sessile serated

C. Hamartomatous

Which of the following is true about cell free DNA testing? A. Will identify 100% of cases of T21, 18, and 13 B. Has a comparable FPR to biochemical screening C. Has a higher PPV than biochemical screening D. Has a higher PPV in lower risk patients than high risk patients

C. Has a higher PPV than biochemical screening Will not identify 100% of cases, FPR is much lower in NIPT, and NIPT has a higher PPV in high risk patients

Disease Y occurs in the general population with a frequency of 1/50. An individual undergoes genomic risk profiling for this disease with the following genotype results: SNP1 CC - RR = 1.3 SNP2 AT - RR = 0.8 SNP3 GC - RR = 1.2 You counsel this person that: A. Her relative risk for this disease is 25% B. Her relative risk for this disease is 1.25x that of the general population C. Her absolute risk for this disease is 2.5% D. Her absolute risk for this disease is 25% E. She is at very high risk for this disease

C. Her absolute risk for this disease is 2.5% Why? - To get genomic risk at multiple SNPs, multiply RR at each SNP: 13/10 x 8/10 x 12/10 = 13/10 x 4/5 x 6/5 = 312/250 = 1.25 Answers A and D are too high, Answer C is referring to odds ratio (1.25x that of the general population) C is correct after you multiply be disease frequency in gen pop: - 1/50 x 1.25 = 0.025 = 2.5%

A 55-year old woman with thyroid cancer is referred for genetic counseling. Her medical history includes surgical excision of a growth on her neck and excision of colon polyps at age 50. She was adopted, and her family history is not available. Which of the following would be the MOST compelling additional information to support testing for mutations in the PTEN gene? A. Her first son was born at 36 weeks gestation B. The growth excised from her neck was a sarcoma C. Her head circumference is above the 98th percentile D. The polyps were hamartomas

C. Her head circumference is above the 98th percentile Macrocephaly more specific than hamartomatous polyps in oncology

Gastrointestinal stromal tumors (GISTs) are associated with which hereditary cancer syndrome? A. LFS B. HPRC C. Hereditary PGL/PCC D. HLRCC

C. Hereditary PGL/PCC SDHC most associated with lower risk of PCC/PGL

Which of the following mutations in CHEK2 has been shown to have lower penetrance of cancer risk? A. 1100delC B. 1100delG C. I157T D. 1400delT

C. I157T Truncating mutations like 1100delC Missense mutations like I157T

A 45yo man with CRC comes for genetic counseling. In conjunction with MSI testing, which of the following tests done on his colon tumor would be MOST helpful in deciding about testing for germline Lynch syndrome? A. Comparative genomic hybridization B. Tumor cell karyotyping C. Immunohistochemical staining D. Loss of heterozygosity

C. Immunohistochemical staining

Which of the following is a potential limitation of multi-gene panels for cardiovascular genetic testing vs single gene testing? A. Increased sensitivity B. Decreased sensitivity C. Increased number of VUSs D. Decreased number of VUSs

C. Increased number of VUSs

Which is the greatest risk factor for Alzheimer disease? A. History of TBI B. APOE genotype C. Increasing age D. Diabetes E. Female gender

C. Increasing age

Which of the following chromosome aberrations would be detectable by CMA? A. Balanced translocation B. Pericentric inversion C. Isodicentric chromosome D. Paracentric inversion

C. Isodicentric chromosome

Which of the following is true about telomerase? A. It contains an essential DNA component B. It acts to degrade telomeres C. It prevents loss of essential genes at telomeres D. It is present in all cells

C. It prevents loss of essential genes at telomeres

A woman is referred for genetic counseling. Her son died at the age of 3 weeks with the following findings: holoprosencephaly, cleft lip and palate, and polydactyly. Her doctor has referred her for genetic counseling to discuss implications for future pregnancies. What additional information would be the MOST helpful to determine the risk for recurrence? A. Complete family history B. Quad screen results from the previous pregnancy C. Karyotype results for her son D. Her son's death certificate

C. Karyotype results for her son

Which of the following is NOT a benefit of panel testing? A. Increased chance to detect a clinically significant mutation B. Cost effective C. Lower chance of a VUS D. Medical management is more likely to be affected

C. Lower chance of a VUS Actually higher chance and a limitation of panel testing

A patient is diagnosed with a pheo at age 30y. She is interested in genetic testing. Which of the following genes should be on the panel test you order: A. FH, MEN1 MEN2 B. FH, SDHB, VHL C. MEN2, SDHB, VHL D. MEN1, MEN2, SDHB

C. MEN2, SDHB, VHL Differential dx for pheos: - SDHX, TMEM127, MAX - NF1 - VHL - MEN2A/B

A patient presents with papillary type 1 renal cell carcinoma. What genetic testing should be considered? a. MEN1 gene b. MEN2 gene c. MET gene d. VHL gene

C. MET gene Papillary type 1 --> HPRC --> MET

Which of the following testing methods would be most useful for a condition where there are few common mutations, and most mutations are due to deletions of one exon in a particular gene? A. Allele specific oligonucleotide hybridization B. Matrix assisted laser desorption/ionization-time of flight (MALDI-TOF/mass spec) C. MLPA D. Massively parallel sequencing

C. MLPA

Normal, a 35yo woman, presents fo GC. She has a son, Alex, and a daughter, Maria. Her father, Melvin, has a personal hx of breast cancer. He is adopted and has no information about his family. Norma wants her son tested for HBOC as soon as possible. Who should be testing first? A. Alex B. Maria C. Melvin D. Norma

C. Melvin

Testing for the CAG trinucleotide repeat for HD reveals the following result: 12 repeats; 38 repeats. This result is best classified as A. Normal B. Intermediate/Mutable allele C. Mutation, Reduced penetrance D. Mutation, Full penetrance

C. Mutation, Reduced penetrance 26 or less normal 27-35 intermediate 36-39 reduced pen 40 or more full pen

Which of the following situations does not increase the risk for UPD? A. Trisomy mosaicism on CVS B. Presence of an extra marker or ring chromosome C. No call on NIPT D. Rob trans

C. No call on NIPT

A pregnant woman is undecided about pursuing invasive testing. She asks, "If I were your wife, what would you tell me to do?" The counselor replies, "I don't know what I would do, but I do know what concerns I would think about." Which of the following techniques BEST describes this response? A. Advice giving B. Directive guidance C. Nondirective counseling D. Nonjudgmental reflection

C. Nondirective counseling

Which is FALSE with regard to hypertrophic cardiomyopathy? A. May present with shortness of breath (particularly with exertion), chest pain, palpitations, and syncope B. May be asymptomatic C. Occurs exclusively in adults D. Can lead to sudden cardiac death

C. Occurs exclusively in adults Extremely variable onset, 1-90y, although most 15-35y

What ophthalmologic finding is strongly suggestive of NF1? A. CHRPE and retinal angiomas B. Lisch nodules and retinal hamartoma C. Optic glioma and Lisch nodules D. Optic glioma and retinal angiomas

C. Optic glioma and Lisch nodules - CHRPE (FAP) - Retinal angiomas (VHL) - Retinal hamartomas (TSC major criteria) - Iris hamartomas = Lisch nodules (NF1) NIH dx criteria: 2 or more - 6 or more café-au-lait macules over 5 mm in diameter in prepubertal and over 15mm in post-pubertal - 2 or more neurofibromas of any type or one plexiform neurofibroma- Axillary or inguinal freckling- 2 or more Lisch nodules (iris hamartomas)- Optic glioma- Distinctive osseous lesion (e.g., sphenoid dysplasia or thinning of long bone cortex/ pseudarthrosis). - FDR with NF1 by the above criteria.

Genomic regions that are IBD A. Result in premature protein truncation B. Are caused by microduplications C. Originate from the same ancestor D. Always indicate an AR condition

C. Originate from the same ancestor

Children with Shwachman Diamond syndrome typically have bone marrow failure as well as? A. Dental anomalies B. Coritcal tubers C. Pancreatic insufficiency D. Seizures

C. Pancreatic insufficiency SDS is 2nd most common cause of pancreatic insufficiency after CF

Which of the following is least associated with Cowden syndrome? A. Breast cancer B. Follicular thyroid cancer C. Papillary thyroid cancer D. Endometrial cancer

C. Papillary thyroid cancer Memory: - Cowden (PTEN) --> no P = follicular - FAP --> no F = papillary Which thyroid cancer goes with which syndrome? - Cowden: follicular - FAP: papillary - MEN2: medullary

Zellweger syndrome spectrum is an example of which category of inherited metabolic diseases? A. Lysosomal storage disease B. Mitochondrial disease C. Peroxisomal disease D. Fatty acid oxidation disorders

C. Peroxisomal disease

Mutations in SCN5A are associated with Long QT and Brugada. This is an example of A. Locus heterogeneity B. Allelic heterogeneity C. Phenotypic heterogeneity D. Reduced penetrance

C. Phenotypic heterogeneity

Kuvan© (BH4 supplementation) is a treatment which can improve symptoms for which of the following metabolic conditions? A. Kearns-Sayre syndrome B. Krabbe Disease C. Phenylketonuria D. Galactosemia

C. Phenylketonuria

Which of the following is TRUE about psychosocial issues in cardiovascular genetics? A. There are no psychosocial issues in CV genetics because the conditions can be managed with medication and/or implantable cardioverter defibrillators B. The psychosocial issues in CV genetics are very unique to these diseases and have no overlap with other universal counseling issues C. Pyschosocial issues in CV genetics may include issues surrounding uncertainty, changes in perception of health, and guilt or blame D. The most significant psychosocial issue in CV genetics is denial

C. Pyschosocial issues in CV genetics may include issues surrounding uncertainty, changes in perception of health, and guilt or blame

What is a hallmark feature of mitochondrial disease on muscle pathology? A. Small mitochondria B. No mitochondria. present C. Ragged red fibers D. None of the above

C. Ragged red fibers Skeletal muscle biopsy: Morphological analysis - Mitochondrial proliferation - Ragged red fibers - Subsarcolemmal accumulation of mitochondria - COX-deficient fibers Electron microscopy - Increased number of mitochondria - Structurally abnormal mitochondria

Which of the following increases the suspicion for a NARP diagnosis? A. Symptoms present at birth B. A mutation in Complex I subunit genes C. Retinitis pigmentosa D. Hearing loss

C. Retinitis pigmentosa NARP = Neurogenic Ataxia RP A. No, adult onset disease B. No, complex V C. Yes D. No, HL is not a symptom

Which of the following is not an element of informed consent in genetic counseling? A. Competence B. Amount and accuracy of information C. Risk assessment D. Patient understanding E. Authorization

C. Risk assessment 5 elements of informed consent in genetic counseling: 1. Competence 2. Amount and accuracy of information 3. Patient understanding 4. Voluntariness 5. Authorization

Which of the following is NOT one of the NHGRI criteria for a genetic screening program? A. High frequency of carriers in the screened population B. Screening is cost effective C. Screening is mandatory D. Consent is obtained E. The disorder impairs the health in affected individuals

C. Screening is mandatory NHGRI proposed criteria for a genetic screening program: - The disorder impairs health in the homozygous affected offspring. - There is a high frequency of carriers in the screened population. - Technically and clinically valid screening methods are available and cost effective to all. - IVF, prenatal diagnosis, and termination are reproductive options. - Consent (informed and voluntary participation) is obtained. - Potential benefits and risks of carrier testing are communicated before and after the test. - Privacy is protected. - Stigmatization of the carrier by the community is minimized. - Experienced professional resources are available

According to current professional guidelines, all of the following should be discussed as part of pretest counseling for expanded carrier screening, except A. Some conditions have less well-defined phenotypes B. Because many conditions being screened are rare, disease prevalence, mutation frequencies, and detection may be rather imprecise and residual risk estimates may not be reliable C. Screening panels may change over time, and because there may be differences between laboratories in the conditions screened, updated carrier screening should be considered with each pregnancy D. Screen-negative results reduce the likelihood of the carrier state for the conditions, but a residual risk of being a carrier always remains

C. Screening panels may change over time, and because there may be differences between laboratories in the conditions screened, updated carrier screening should be considered with each pregnancy Elements of pretest-counseling for expanded carrier screening: - Some conditions have less well-defined phenotypes - Because many conditions being screened are rare, disease prevalence, mutation frequencies, and detection rather may be imprecise and residual risk estimates may not be reliable - Screen-negative results reduce the likelihood of the carrier state for the conditions, but a residual risk of being a carrier always remains - Screening panels may change over time, and there may be differences between laboratories in the conditions screened. Despite this, carrier res-creening is typically not offered or recommended.

Which of the following is NOT a principle behind the design of a screening test? A. Screening test must be reliable B. Disorder must be relatively common C. Start with a population at high risk for the disorder in question D. Benefits of screening outweighs the cost of screening E. Disorder must be impairing or fatal

C. Start with a population at high risk for the disorder in question • Healthy population • Reliable • Relatively frequent disease • Impairing or fatal disease • Beneficial intervention may be possible • Prompt testing and follow up • Benefits outweigh costs • Voluntary and educational

A newborn is identified to have polydactyly and holoprosencephaly. Which of the following is most likely? A. Aniridia-Wilms tumor association B. Miller-Dieker C. T13 D. T21 E. T18

C. T13

One example of RNA editing in humans is? A. The insertion of U's into the huntingtin mRNA B. The deletion of U's from CFTR mRNA C. The conversion of a C to a U in the apolipoprotein B mRNA D. The conversion of A to I in the a-tropomyosin mRNA

C. The conversion of a C to a U in the apolipoprotein B mRNA

der(13;14)(q10;q10) is A. The most lethal rob translocation B. Balanced carriers do not have an increased risk of infertility C. The most common rob translocation D. Balanced carriers cannot be detected in leukocytes

C. The most common rob translocation Makes up 85% of robertsonians

Which one of the following statements about the genetic code is FALSE? A. Each amino acid in a polypeptide is specified by a 3 nucleotide codon in the DNA B. Most organisms, from bacteria to humans, have the same genetic code C. The number of tRNAs equals the number of possible codons for amino acids D. There is only one start codon, but multiple possible stop codons

C. The number of tRNAs equals the number of possible codons for amino acids Why? - 61 tRNA types are required to provide one-to-one correspondence between tRNA molecules and codons that specify amino acids, AS THERE ARE 61 SENSE CODONS (3 stop/nonsense codons excluded) of the standard genetic code. - The interaction between tRNA and mRNA depends on complementary base pairing. - Each tRNA has a 3 bp sequence that will base pair with a mRNA codon (tRNA sequence is called the anti-codon)

Which of the following illustrates the genetic principle of anticipation? A. The first symptoms of HD change with each generation B. The first symptoms of HD remain constant between generations C. The onset age for HD decreases with each generation D. The onset age for HD increases with each generation E. The symptoms and onset age of HD vary with each generation

C. The onset age for HD decreases with each generation

Harold is red-green color blind and has hemophilia A. Both of these conditions are inherited in an XL-R manner. Assume Harold's partner is NOT a carrier of either of these conditions. Considering Harold's children, which of the following statements is TRUE? A. The probability that each of his daughters will be a carrier of hemophilia A is 1/2 B. The probability that each of his daughters will be a carrier of color blindness is 1/2 C. The probability that each of his daughters will be a carrier of both conditions is 1 D. The probability that each of his sons will be affected with hemophilia A is 1/2

C. The probability that each of his daughters will be a carrier of both conditions is 1 All daughters carriers, sons unaffected because they inherit his Y

Neurofibromatosis type 1 is one of the most common AD disorders. A woman with NF1 has an unaffected partner. All of the following statements regarding their children is false EXCEPT? A. The probability that each of their children will be affected is 1/4 B. The probability that their second child will be affected if their first child is affected is 1/4 C. The probability that their third child will be affected if their first two children are affected is 1/2 D. If their first child is affected, then their second child will not be affected

C. The probability that their third child will be affected if their first two children are affected is 1/2

Which of the following statements is FALSE? A. The risk for male breast cancer is greater with a BRCA2 mutation than a BRCA1 mutation B. The risk for melanoma is greater with a BRCA2 mutation than a BRCA1 mutation C. The risk for melanoma is greater with a BRCA1 mutation than a BRCA2 mutation D. The risk for ovarian cancer is greater with a BRCA1 than a BRCA2 mutation

C. The risk for melanoma is greater with a BRCA1 mutation than a BRCA2 mutation Male breast cancer: - BRCA1 (1-5%) - BRCA2 (5-10%) Melanoma: - BRCA1 (1%) - BRCA2 (3-5%) Ovarian cancer: - BRCA1 (30-45%) - BRCA2 (10-20%)

A second-trimester ultrasound shows long bone measurements <5th percentile. The head circumference is at the 80th percentile. The chest-to-abdominal circumference ratio is 0.5. The maternal and paternal family history is non-contributory. What should the couple be counseled? a. Based on the long bone measurements, there is an increased risk for Down syndrome (LR ~4). NIPS and/or amniocentesis is recommended to test for chromosomal abnormalities. b. This combination of measurements suggests skeletal dysplasia. Prenatal radiologic evaluation is indicated to confirm the diagnosis, and amniocentesis is recommended for molecular testing to identify the form of skeletal dysplasia. c. These findings strongly suggest a perinatal lethal disorder, and referral to a center with expertise in skeletal dysplasia is appropriate. d. These findings are suggestive of either Down syndrome or skeletal dysplasia. Compared to the 2nd trimester, 3rd trimester US measurements are more accurate at differentiating these conditions. A repeat evaluation should be planned.

C. These findings strongly suggest a perinatal lethal disorder, and referral to a center with expertise in skeletal dysplasia is appropriate. (A) and (D) are true just based on the long bone measurements. But, (C) is the best answer because not only do the long bone and head measurements suggest skeletal dysplasia, the chest-abdominal ratio <0.6 suggests a perinatal lethal skeletal dysplasia. Femur length to abdominal circumference of 0.16 also suggests a perinatal lethal skeletal dysplasia. As for (B), post-natal radiologic exam is used for diagnosis, and amnio for molecular testing is controversial, especially in a case of likely lethality.

Which of the following is PPV? A. True positives/(true positives + false negatives) B. True negatives/(true negatives + false positives) C. True positives/(true positives + false positives) D. True negatives/(true negatives + false negatives)

C. True positives/(true positives + false positives) Affected with positive test / All people (affected and not) with positive test PPV has to do with test results (denominator) whereas sensitivity/specificity have to do with affected v unaffected (de nominator

A risk prediction model based on 18 SNPs revealed an Area Under the Curve (AUC) of 0.6. This means that? A. It is 60% better than flipping a coin B. The combined relative risk is less than 1 C. You will be "correct" 60% of the time D. The sensitivity of the model is much greater than the specificity E. Adding the SNPs to an existing clinical risk model should substantially improve clinical utility

C. You will be "correct" 60% of the time *in your risk prediction

CMA on child with suspected turner syndrome comes back with the following result: i(X)(q10). What does it mean? A. idiopathic X: partial monosomy Xp and partial trisomy Xq B. idiopathic X: partial monosomy Xq and partial trisomy Xp C. isochromosome X: partial monosomy Xp and partial trisomy Xq D. isochromosome X: partial monosomy Xq and partial trisomy Xp

C. isochromosome X: partial monosomy Xp and partial trisomy Xq

Which pathogenic mutation accounts for 98% of MEN2B cases? A. p.Val819Arg B. p.Thr819Met C. p.Met918Thr D. p.Arg918Val

C. p.Met918Thr MEMORY: MEN2B looks like Met918 and throw it away

Oligonucleotide mediated exon skipping therapy A. allows read-through of a premature stop codon B. is the cause of DMD in 20% of cases C. restores the reading frame to give a partially functional protein D. results in upregualtion of utrophin

C. restores the reading frame to give a partially functional protein

Which is NOT a common presenting phenotype in DMD? A. abnormal gait B. delayed motor milestones C. seizures D. speech delay and learning disability

C. seizures

Which of the following accurately describes the replication of DNA in all organisms? A. conservative B. dispersive C. semi-conservative D. nonlinear

C. semi-conservative

According to ACMG, which is not an indication for CMA as a first-tier test? A. MCA B. non-syndromic DD C. speech delay D. ASD

C. speech delay

Which of the following is an example of tertiary structure? A. The primary amino acid sequence of polyA binding protein B. The alpha helix in myoglobin C. the 3D structure of a single TATA binding protein molecule D. The two alpha-globin and two beta-globin molecules assembled into hemoglobin

C. the 3D structure of a single TATA binding protein molecule

Which of the following is part of the COMMON phenotype for all congenital myopathies with classic presentation? A. onset in adulthood with hypotonia B. normal motor milestones followed by loss after puberty C. typically normal CK D. short stature with long face

C. typically normal CK

~ how many variants are detected through the raw sequencing data of WES? A. ~1,000,000 B. ~750,000 C. ~300,000 D. ~1,000

C. ~300,000

Coping behaviors

CDS PPEAS (sounds like CD's please... idk) Confrontative Distancing Self-controlling Plan Positive reappraisal Escape-avoidance Accept responsibility Seek social support

About 75% of children with a 22q11.2 deletion have what following clinical features?

CHD Palatal anomalies Immunodeficiency

What is the classic Di George triad which originally defined the condition (22q)?

CHD, Hypocalcemia, Immune deficiency (CHI)

10% of babies exposed to this in utero have a rash, hearing loss, hepatosplenomegaly, and microcephaly. Which teratogen is this?

CMV

Defense mechanism: undoing

Canceling out a distressing experience through a reverse action An obsessive need for prenatal testing after having a child with anencephaly

Carriers of dystrophin gene mutations are at increased risk for?

Cardiomyopathy

Female carriers of Duchenne have risks of what features of the condition?

Cardiomyopathy (up to 20%) Mild muscle weakness (~5%)

The APOE gene is also a risk factor for

Cardiovascular disease

Father of person-centered counseling

Carl Rogers

Prenatal dx for CF would be least informative for which couple listed below? Those who had a previous child with CF Woman dx with CF who is married to a male F508del carrier Carrier female who has caucasian male partner negative for 100 mutation panel Ultrasound dx for meconium ileus

Carrier female who has caucasian male partner negative for 100 mutation panel

People of Eastern European Jewish ancestry (also called Ashkenazi Jewish) should be offered...?

Carrier screening for Tay-Sachs disease, Canavan disease, and familial dysautonomia in addition to CF and SMA (ACOG). The American College of Medical Genetics also suggests screening for Gaucher disease type I, Bloom syndrome, Niemann-Pick type A, Mu-colipidois IV, and Fanconi anemia group C.

Individuals with Cajun or French Canadian ancestry should be offered...?

Carrier screening for Tay-Sachs disease.

People of African, Mediterranean, and Southeast Asian ancestry should be offered...?

Carrier screening for the thalassemias and sickle cell disease.

Imperforate anus is a characteristic you'll likely only see in which two conditions?

Cat Eye: triad (CAT) - Coloboma, Anal anom, Preauricular pits/tags (22p microdup resulting in partial tetrasomy) VACTERL(S) - vertebral, anal atresia, CHD, TE-fistula, Renal, Limb (SUA)

nonsense mediated decay

Cellular mechanism to decrease production of truncated proteins (due to premature termination codon/PTC) - In first round of translation, ribosome will displace exon junction complexes (EJC) - Most of the time, normal termination codon is in last exon - All splicing is upstream of termination codon If PTC is present before last exon, PTC will appear upstream of EJC - Proteins that associate with ribosome will contact EJC to trigger cleavage of cap followed by mRNA degradation ◦ Loss of mRNA means little production of truncated protein ***NMD doesn't happen if PTC is in last exon or within 50 nts of the last splice junction

Exceptions to random mating: assortative mating

Choosing a mate who possesses the same (positive) or opposite (negative) trait or genotype. For example, individuals who are deaf often seek a mate who is also deaf; similarly for those with achondroplasia; mates with similar family histories for a trait may meet at support groups

CLIA

Clinical Laboratory Improvement Amendments ◦ Part of Centers for Medicare and Medicaid Services (CMS) ◦ CMS regulates all non-research lab testing in US

Walker Warburg is a congenital muscular dystrophy with hypoplastic cerebellum and brainstem, hydrocephalus, and what other feature seen on brain imaging?

Cobblestone lissencephaly Patients also have absent psychomotor development, severe eye problems (congenital glaucoma), and severe ID.

How is the level of consanguinity measured?

Coefficient of inbreeding (F), the probability that a person who is homozygous at a particular locus inherited both alleles from a single ancestor, or the proportion of loci at which a person is homozygous by descent.

Ullrich congenital muscular dystrophy and Bethlem myopathy are a continuum of what overarching condition?

Collagen Type VI related disorders Bethlem myopathy: proximal muscle weakness and variable contractures, affecting most frequently the long finger flexors, elbows, and ankles. The onset of Bethlem myopathy ranges from prenatal to mid-adulthood. Prenatal onset is characterized by decreased fetal movements; neonatal onset by hypotonia or torticollis; early childhood onset by delayed motor milestones, muscle weakness, and contractures; and adult onset (4th to 6th decades) by proximal weakness and Achilles tendon or long finger flexor contractures. Bethlem myopathy is inherited AD. Ullrich congenital muscular dystrophy usually AR

Advanced empathy

Communicates an understanding of underlying implicit aspects of patient experience. You are additive and go beyond surface-level patient expressions. Variously known as additive empathy, reframing, and interpretation

Primary empathy

Communicates initial understanding of what a patient is experiencing. You use your own words to concisely convey an understanding of surface, fairly explicit patient experiences

ACMG/ACOG commentary on expanded carrier screening

Conditions being screened should be associated with one or more of the following - Cognitive disability - Need for surgical or medical intervention - Effect on quality of life Consider excluding: - Conditions with adult onset phenotype - Variants with high allele frequency but low penetrance (MTHFR) Variants should have a clear have a well understood associated phenotype - VUS results should not be reported Residual risk estimates should be provided in report and limitation of post-test risk estimates should be provided

X-linked centronuclear myopathy has similar prenatal features to what autosomal dominant neuromuscular condition?

Congenital myotonic dystrophy X-linked centronuclear myopathy has classic features of polyhydramnios, reduced fetal movements, and newborn hypotonia.

Which one term best defines a "syndrome"? Multiple causally independent features seen in one individual Constellation of findings repeatedly seen in unrelated individuals A common individual finding seen in multiple unrelated individuals A striking constellation of findings seen in a single individual

Constellation of findings repeatedly seen in unrelated individuals

In which of the following syndromes is synophrys an associated feature? Treacher collins Kabuki SLOS Cornelia de Lange

Cornelia de Lange

You meet with a patient with a personal hx of breast cancer. Which of the following clinical features would increase your suspicion of Cowden syndrome the MOST? A. Hamartomatous polyps B. Sebaceous cyst C. Macrocephaly D. Lipomas

Correct answer: C. Macrocephaly (only cancer syndrome with macrocephaly?) I still think it should be... A. Hamartomatous polyps - because >90% with hamartomatous polyps - 3 or more GI hamartomatous polyps = major criteria Macrocephaly = major criteria, but not as common as polyps Lipomas = minor criteria Sebaceous cyst not in criteria

Which of the following is NOT a common finding in 22q11.2 microdeletion syndrome? Bulbous nose Cutaneous syndactyly Hooded eyelids Small ears with thickened helices

Cutaneous syndactyly

You see a couple in prenatal clinic. The man reports that his brother has a son who has chronic lung infections and receives pancreatic enzyme replacement. What condition do you suspect for the nephew?

Cystic fibrosis

All individuals should be offered what for carrier screening? (population-based)

Cystic fibrosis (CF) and spinal muscular atrophy (SMA)

A woman in your prenatal clinic is experiencing virilization symptoms such as voice deepening, acne, and hirsutism. What genetic condition can cause these features?

Cytochrome P450 Oxidoreductase Deficiency or Antley Bixler syndrome ABS phenotype: Ambiguous genitalia, enlarged cystic ovaries, poor masculinization in males, maternal virilization during pregnancy with an affected fetus. Craniosynostosis, choanal stenosis or atresia, stenotic external auditory canals, hydrocephalus. Neonatal fractures, bowing of the long bones, joint contracture, renal malformations

In humans, Lynch syndrome is due to mutations in genes that encode proteins involved in?

Mismatch repair

Of infants exposed to thalidomide during the period of limb formation, what % had limb defects? A. 0.1 B. 1 C. 0.8 D. 0.2 E. 0.01

D. 0.2 (20%)

Patient has an SDHD mutation and a family history of head and neck PGLs. What is the patient's risk of developing a paraganglioma if she inherited the gene mutation from her father? A. Close to 0 B. 0.2 C. 0.5 D. 0.8

D. 0.5

For patients with an NF2 mutation, what is the estimated % of risk that they will develop bilateral vestibular schwannoma? A. 0.25 B. 0.5 C. 0.75 D. 1

D. 1 Bilateral vestibular schwannomas by age 30 (~100%)

Which of the following is NOT an indication to order WES according to ACMG? A. 2yo male with MCA B. 6yo female with ASD C. 12yo male with epilepsy and hx of normal genetic tests D. 1 day old male with jaundice

D. 1 day old male with jaundice

7. A woman comes for genetic counseling to discuss the prenatal diagnosis of Trisomy 21 by amniocentesis. She says she feels "very alone" with this diagnosis. Which of the following is the BEST next response for the counselor? A. "Who has been supporting you during this pregnancy?" B. "Have you spoken with a trisomy 21 support group?" C. "What has your partner said about the diagnosis?" D. "How has this situation been isolating for you?"

D. "How has this situation been isolating for you?"

The sum of the size of the runs of homozygosity divided by the total autosomal genomic length approximately equals: A. % likelihood of a consanguineous union B. % of UPD C. % likelihood of an AR condition D. % identical by descent

D. % identical by descent

Albright hereditary osteodystrophy is an inherited genetic condition whose phenotype can vary based on if the UPD is maternal or paternal. What chromosome is involved? A. 7 B. 11 C. 15 D. 20

D. 20 Imprinting conditions - 7: mat UPD 7 causes RSS - 11: BWS/Russell silver - 14: Kagami-Ogata Syndrome/ Temple syndrome - 15: PWS/ Angelman 20: Albright hereditary osteodystrophy *In AHO, if the father transmits the mutant allele, the phenotype is more mild. If mother, more severe

Which of the following chromosome aberrations would be detectable by CMA? A. 46,XX,inv(4)(p14q21.3) B. 46,XX,t(8;13)(q24.1;q14.1) C. 46,XX,t(3;5)(p21;q31) D. 46,XY,del(5)(q33q44)

D. 46,XY,del(5)(q33q44)

Which of the following laboratory studies is MOST likely to confirm the diagnosis of Smith-Lemli-Opitz syndrome? A. 15q11-13 methylation studies B. Isoelectric focusing of transferrin C. Chromosomal microarray D. 7-dehydrocholesterol analysis

D. 7-dehydrocholesterol analysis

A mutation in which of the following genes warrants a conversation about risk for Fanconi Anemia in offspring? A. BRCA2 B. RAD51C C. APC D. A and B

D. A and B BRCA2 = FANCD1 Biallelic mutations of PALB2, BRIP1 and RAD51C can also lead to FA - Affected individuals are compound heterozygotes

Nonsense mutations can result in? A. The synthesis of a truncated protein B. Replacement of an amino acid with a different one C. Degradation of mutant mRNA by nonsense mediated decay D. A and C E. B and C

D. A and C

In which of the following diagnoses do symptoms typically appear in childhood? A. Pearson and Kearns-Sayre syndrome B. Progressive external opthalmplegia C. MELAS D. A and C

D. A and C B. No, occurs in adulthood adulthood

Which of the following best describe FTD? A. A type of Alzheimer disease (AD) that primary affects language and personality B. A type of AD caused by mutations in the C9ORF72 gene C. A non-Alzheimer dementia that is entirely due to AD inheritance D. A non-Alzheimer dementia that may present with a broad range of symptoms, including language difficulties, changes in personality, and/or movement disorders

D. A non-Alzheimer dementia that may present with a broad range of symptoms, including language difficulties, changes in personality, and/or movement disorders

A significant mode of genetic regulation in eukaryotes is methylation, which refers to? A. Altering RNA polymerase activity by methylation of the enzyme B. Altering translational activity of highly methylated tRNAs C. Addition of a methylG cap to a primary transcript D. Addition of methyl groups to the cytosine of CpG dinucleotides

D. Addition of methyl groups to the cytosine of CpG dinucleotides

At what age do symptoms/signs of NF2 typically manifest? A. Before age 3 B. Age 1-4y C. Age 9-12y D. Age 17-21y E. Age 26-30y

D. Age 17-21y

An individual with mitochondrial disease can present at what age? A. Birth B. Adolescence C. Adulthood D. All of the above

D. All of the above

Annual breast MRI screening is recommended for women in which of the following groups? A. Identified to have a BRCA1/2 mutation B. Fam hx of an FDR with BRCA, but has not personally had genetic testing C. Lifetime risk of >20% to develop breast cancer D. All of the above

D. All of the above

For complex disorders, GWAS typically necessitate the use of A. Very large sample sizes (individuals with and without the phenotype) B. More stringent threshold for statistical signifcance C. Hundreds of thousands of SNPs D. All of the above

D. All of the above

Huntington's disease can present with which of the following types of features A. Movement B. Psychiatric C. Cognitive D. All of the above E. Only a and b

D. All of the above

One gene can give rise to different mRNA products by? A. Alternative promoter use B. Alternative splicing C. Alternative cleavage/polyadenylation site use D. All of the above

D. All of the above

The chance of recurrence of a complex disorder among relatives may be influenced by A. characteristics of the affected individual B. characteristics of the individual for whom chance is being calculated C. characteristics of the fam hx D. All of the above

D. All of the above

Which can be used to diagnose hypertrophic cardiomyopathy? A. Echocardiogram, electrocardiogram, physical exam and history B. Heart tissue sample C. Genetic testing D. All of the above

D. All of the above

Which is true with regard to Short QT syndrome? A. Clinical presentation is indistinguishable from Long QT syndrome B. Variable penetrance C. AD D. All of the above

D. All of the above

Which of the following are issues to consider when choosing an oncology genetic test? A. Patient preference B. Turnaround time C. Sensitivity of the test D. All of the above

D. All of the above

Which of the following diagnoses have been causally linked to 22q11.2 microdeletions? A. Schizophrenia B. ID C. ASD D. All of the above E. A and B

D. All of the above

Which of the following does NOT increase the risk of sudden cardiac arrest/death? A. Coronary artery disease B. Hypertrophic cardiomyopathy C. Long QT syndrome D. All of the above

D. All of the above

Which of the following increases the likelihood of having a BRCA mutation? A. Bilateral breast cancer B. AJ ancestry C. Triple negative breast cancer D. All of the above

D. All of the above

Which of the following should be included in a fam hx when evaluating for cardiovascular disease? A. Hx of accidental death, including circumstances B. Hx of fainting, including circumstances C. Hx of seizures, including information about response to medication D. All of the above

D. All of the above

Which of the following statements about heritability is true? A. It is a statistical measure B. It measures the degree to which the variance in a phenotype is attributable to genetics C. It is a concept that can only be applied at the population level, rather than at the individual level D. All of the above

D. All of the above

Which of the following symptoms can a patient present with who has mitochondrial disease? A. myoclonus, ptosis, and neuropthay B. HL, optic atrophy, and stroke C. exercise intolerance, FTT, and diabetes D. All of the above

D. All of the above

Genetic causes of ALS can have which of the below inheritance patterns? A. AR B. AD C. XL D. All of the above E. Only a and b are correct

D. All of the above - Parkinson's disease and ALS is all of the above (PD, mixed; ALS mostly AD, but rare AR/XL) - Huntington's, FTD, and Early-Onset Familial Alzheimer disease (EOFAD) are AD or AD reduced pen

Mitochondrial disease is caused by which mode of transmission? A. Sporadic B. Maternal C. AR D. All of the above

D. All of the above Also AD and XL (all inheritance patterns observed) AR is most common

Select the answer that best describes the method used for prenatal cfDNA screening A. Massively parallel shotgun sequencing, in which all of the cfDNA that is extracted from a maternal sample is sequenced B. Targeted sequencing of specific regions C. Analysis of single nucleotide polymorphisms to determine the genotype and relative copy number of each chromosome of interest D. All of the above E. None of the above

D. All of the above cfDNA screening employs multiple methods, all relying on NGS technologies and bioinformatic analyses

Which of the following may be used in a clinical dx of Arrhythmogenic Right Ventricular Dysplasia/Cardiomyopathy? A. Tissue biopsy B. Arrhythmias C. Fam hx D. All of the above

D. All of the above Strict clinical dx criteria which involves a combination of noninvasive and invasive dx tests to detect changes in heart structure and rhythm. Can include fam hx, arrhythmias, ECG (depolarization/repolarization anomalies), echo, and biopsy findings.

Which of the following statements correctly states the relationship in a screening test between detection rate and false positive rate? A. As detection rate increases, FPR decreases B. As DR decreases, FPR increases C. DR and FPR are unrelated to each other D. As DR increases, FPR increases E. A and B are correct

D. As DR increases, FPR increases A and B would be true if it was detection rate and false NEGATIVE rate

Which of the following is FALSE with regard to familial hypercholesterolemia? A. Homozygotes have earlier age of onset and more severe disease B. Characterized by significant elevations in total serum cholesterol and LDL cholesterol early in life C. Increases the risk of coronary artery disease and MI D. Associated with no characteristic physical features

D. Associated with no characteristic physical features

Hunter syndrome is a rare form of mucopolysacchararidosis that differs from all other forms in that it shows XL-R inheritance. A woman with two affected brothers is referred for genetic counseling. All of the following statements are FALSE except? A. The probability that she is a carrier is 2/3 B. If she is a carrier, then the probability that each of her sons will be affected is 1/4 C. If she is a carrier, then the probability that each of her daughters will be a carrier is 1/4 D. Before any carrier testing is carried out, the woman can be correctly informed that the probability that her first son will be affected is 1/4

D. Before any carrier testing is carried out, the woman can be correctly informed that the probability that her first son will be affected is 1/4 A. 1/2 chance to be carrier because we KNOW her mom is carrier with 2 affected sons B. If she is a carrier, prob that a known son is affected is 1/2 C. Wrong, 1/2 D. Correct because a known son means that we only factor in chance that she's a carrier and chance that she passes on mutated X (1/2 x 1/2 = 1/4)

Mutations in the DMD gene may result in? A. A predominant cardiac phenotype (DCM) B. A predominant skeletal muscle phenotype (DMD/BMD) C. A predominant conduction system disorder (polymorphic arrhythmias) D. Both A and B

D. Both A and B

Complex disorders A. Are rare B. Are multifactorial in etiology C. Aggregate in families D. Both B and C

D. Both B and C (Multifactorial and aggregate in families)

What types of mtDNA mutations can cause human disease? A. mtDNA deletions B. mtDNA point mutations C. None of the above D. Both a and b

D. Both a and b

Which of the following is NOT a high penetrance gene associated with hereditary breast cancer? A. PALB2 B. PTEN C. CDH1 D. CHEK2

D. CHEK2 PALB2: 35-58% - considered high-pen PTEN: 25-50% - Cowden: breast, thyroid, endometrial, hamartomatous polyps (low CRC risk, 9%) CDH1: 39-52% - Hereditary diffuse gastric cancer (lobular breast cancer) CHEK2: 1.5-3x increased risk (18-36%) - CRC (6-11% dependent on missense v truncating mutations) · Brca: 3.0x increased risk (27-36%)

Which disorder is characterized by cardiac electrical instability (QRS complexes with frequent and rapid changes in morphology) triggered by acute activation of the adrenergic nervous system? A. HCM B. DCM C. ARVD/C D. Catecholaminergic Polymorphic Ventricular Tachycardia (CPVT)

D. CPVT

Which of the following is a limitation of CMA? A. Cannot detect an unbalanced chromosomal translocation B. Cannot detect aneuploidy C. Cannot detect copy number variants (CNVs) D. Cannot detect low level mosaicism

D. Cannot detect low level mosaicism

Which of the following is not a part of the anticonvulsant embryopathy? A. short, upturned nose B. long philtrum C. increased risk NTDs D. Cerebral calcifications E. Fingernail hypoplasia

D. Cerebral calcifications Toxoplasmosis is associated with Intracranial calcifications AED embryopathy: Features: - Hypertelorism - Broad depressed nasal bridge - Short nose with anteverted nares - "Cupid's bow" lip - Fingernail hypoplasia Increased risk for major malformations - Meningomyelocele (NTD) - Oral clefts - CHD - Limb defects DD, most significantly with Valproic acid

Which of the following refers to the chance that results of a genetic test will be able to affect treatment of a patient? A. Analytic validity B. Analytic utility C. Clinical validity D. Clinical utility

D. Clinical utility

Thymine is a base found in A. protein B. glycogen C. RNA D. DNA

D. DNA

Which of the following is not a signal for the cell to stop moving through the cell cycle A. DNA damage B. Incomplete DNA replication C. Lack of spindle attachment D. Degradation of cyclins

D. Degradation of cyclins Reason: Regulation of cell cycle progress is mediated by cyclins and cyclin-dependent kinases (Cdks) - Cyclins are synthesized and destroyed during a cell cycle - Cdks are present throughout but only active when in complex with cyclin (SO, CYCLIN DEGRADATION WOULD HALT THE CELL CYCLE) - Cdks regulate other cell cycle proteins through phosphorylation

A couple comes for genetic counseling because their daughter has congenital adrenal hyperplasia (CAH). Which of the following would be the BEST strategy to minimize the possible effects of CAH in future pregnancies? A. BH4 supplementation B. Biotin therapy C. Cholesterol supplementation D. Dexamethasone therapy

D. Dexamethasone therapy

A 33-year old woman who is 20 weeks pregnant is referred for genetic counseling. Fetal ultrasound showed caudal regression. Which of the following conditions is the woman MOST likely to have? A. Lupus B. Phenylketonuria C. Hypothyroidism D. Diabetes

D. Diabetes

Which of the following disorders is characterized by the muscle that makes up the left ventricle stretching and becoming thinner, eventually spreading to the right ventricle and atria? A. Arrhythmogenic Right Ventricular Dysplasia B. Restrictive cardiomyopathy C. Hypertrophic cardiomyopathy D. Dilated cardiomyopathy

D. Dilated cardiomyopathy

Guidelines published by ACOG recommends the following regarding carrier screening A. Carrier screening should be offered based on ethnic background and fam hx B. Panethnic screening, with a limited panel of conditions, should be offered to all women in pregnancy C. Expanded carrier screening should be offered to all women in pregnancy D. Ethnic-specific, panethnic, and expanded carrier screening are all acceptable strategies for pre-pregnancy and prenatal carrier screening

D. Ethnic-specific, panethnic, and expanded carrier screening are all acceptable strategies for pre-pregnancy and prenatal carrier screening

Which Fanconi Anemia gene is also known as the BRCA2 gene? A. FANCC B. FANNC1 C. FANCD D. FANCD1

D. FANCD1 FA genes: FANCC - yes FANNC1 - no FANCD - no (but FANCD1 and FANCD2 - yes)

What is the most common disorder in the AJ population for which screening is widely available? A. Tay-sachs B. Canavan C. CF D. Gaucher E. Familial Dysautonomia

D. Gaucher

Which of the following is NOT a possible explanation for loss of MLH1/PMS2 protein on IHC staining? A. Sporadic BRAF mutation V600E B. Germline mutation in MLH1 C. Sporadic MLH1 promotor hypermethylation D. Germline mutation in MSH6

D. Germline mutation in MSH6

Which hereditary cancer syndrome is typically associated with the most aggressive form of renal cancer? A. Birt-Hogg-Dube syndrome B. Hereditary Papillary Renal Carcinoma syndrome C. Hereditary PGL/PCC D. Hereditary Leiomyomatosis-Renal Cell cancer syndrome

D. Hereditary Leiomyomatosis-Renal Cell cancer syndrome (papillary type 2)

PAH can occur along with which syndrome, particularly with mutations in ACVRL1 A. Fabry disease B. Timothy syndrome C. Romano-Ward syndrome D. Hereditary hemorrhagic telangiectasia

D. Hereditary hemorrhagic telangiectasia

Limb girdle muscular dystrophy A. Can be dx by decreased CK levels B. Is caused by a triplet repeat expansion in DMPK C. Is mitochondrially inherited D. Is phenotypically similar to DMD and BMD

D. Is phenotypically similar to DMD and BMD

A 25-year old woman comes for genetic counseling at 12 weeks gestation because her sister recently had a baby girl with a "metabolic condition". She was told that her niece has recurrent vomiting, seizures, and smells like "sweaty feet"." The niece was born overseas, medical records cannot be obtained, and the woman is estranged from her sister. The genetic counselor should look for laboratories that offer carrier testing for which of the following conditions? A. MSUD B. Propionic acidemia C. Methylmalonic acidemia D. Isovaleric Acidemia

D. Isovaleric Acidemia Symptoms of organic acidemia: feeding problems, seizures, metabolic acidosis, lethargy MSUD is an amino acid disorder Of Propionic, methylmalonic, and isovaleric, isovaleric is associated with sweaty feet smell

Which of the following characteristics is more likely to be true of a diagnostic test than a screening test? A. It is inexpensive B. It is rapid C. It is fast D. It has some risk

D. It has some risk

A 1 year old girl developed a cough and vomiting. Her 2 brothers recently suffered from the same symptoms, which have since resolved. On the third day, the patient became lethargic with rapid progression to coma. Hypoglycemia was noted, and she made a full and rapid recovery upon resuscitative efforts in an emergency department. Hypotonia, muscle weakness and cardiomyopathy are present, but there is no past history of myopathy. Developmental milestones were normal. Among the following, the most likely diagnosis is: A. MCAD B. Galactosemia C. Pyruvate Dehydrogenase Complex deficiency D. LCHAD

D. LCHAD

Which of the following disorders has the best chance of having the causative mutation detected with exome sequencing? A. Fascioscapulohumeral dystrophy B. MD C. SMA D. Limb girdle muscular dystrophy

D. Limb girdle muscular dystrophy A. No because contracted repeats cause disease B. No because triplet repeat disorder C. No because exon deletions

A 6-year old girl is referred to your genetics clinic due to her observed unilateral cleft lip and palate. As you are taking the family history, her mother reports that the child's paternal grandfather and paternal uncle also had a cleft palate. They all were reported to have typical intelligent and no other major birth defects. What minor defect would be BEST to ask about in paternal relatives? A. Sandal gap B. Pectus excavatum C. Preauricular ear tags D. Lip pits

D. Lip pits

Which cancer susceptibility genes are proto-oncogenes? A. MET and FH B. RET and DICER1 C. RET and FH D. MET and RET

D. MET and RET HLRCC (FH) HPRC (MET) MEN2 (RET)

The lifetime risk to develop CRC is higher for individuals with a mutation in the _______ gene than it is for individuals with a mutation in the _______ gene A. MLH1; MSH2 B. PMS2; MLH1 C. MSH6; MSH2 D. MLH1; PMS2

D. MLH1; PMS2

A woman who is 27 weeks pregnant has a fetal ultrasound that is suggestive of esophageal atresia. This anomaly is an example of: A. Disruption B. Deformation C. Sequence D. Malformation

D. Malformation

A 6yo male with autism and dysmorphic features was found to have a CNV of unclear significance on X chromosome. Which of the following is the best first step? A. Search for AR conditions in the region of the CNV B. Testing of both parents C. Paternal testing D. Maternal testing

D. Maternal testing

Which feature of a pedigree most greatly increase the suspicion for mitochondrial disease? A. Mostly females affected B. Mostly males affected C. Mostly males affected and related through sisters (i.e., two sisters each have affected sons) D. Mostly females affected and related through sisters (i.e., two sisters each have affected daughters)

D. Mostly females affected and related through sisters (i.e., two sisters each have affected daughters) Answer actually said C, but I think not because that could look like XL inheritance

Which of the following genes are the most likely cause of familial ALS? A. GRN and MAPT B. GRN and C9ORF72 C. GRN and SOD1 D. SOD1 and C9ORF72 E. MAPT and SOD1

D. SOD1 and C9ORF72 - SOD1 = most common cause of familial ALS (~20%) - C9ORF72 technically under FTD, but causes FTD-ALS spectrum

Which is FALSE regarding DCM? A. May be caused by mutations associated with sarcomeric proteins or proteins associated with the contractile apparatus B. Mutations in TTN are the most common known genetic cause of DCM C. May be associated with conduction system disease D. Mutations in fewer than 10 genes have been found to be associated with DCM

D. Mutations in fewer than 10 genes have been found to be associated with DCM TTN accounts for 20% of all inherited DCM, >50 genes

Which of the following is NOT a dx test for fetal chromosome abnormalities? A. Amnio B. CVS C. CMA D. NIPS E. Karyotype

D. NIPS

Which of the following genes is associated with Phenylketonuria? A. ARG1 B. PEX1 C. GALT D. PAH

D. PAH

Individuals who have calcification of Falx, jaw keratocysts, and rib anomalies are at increased risk for having a mutation in which of the following genes? A. BRCA2 B. CDC73 C. CDKN2A D. PTCH1

D. PTCH1 (NBCCS)

Array testing identified an abnormality on chromosome 15 in a pediatric patient with hyperphasia and short stature. Which of the following is the most likely diagnosis and chromosomal abnormality? A. Angelman; duplication B. BWS; deletion C. Wolf-Hirschhorn; duplication D. PWS; deletion E. DiGeorge; deletion

D. PWS; deletion

Which of the following are moderate risk genes associated with hereditary ovarian cancer? A. RAD51C, RAD51D, BRCA1 B. RAD51C, RAD51D, BRCA2 C. CDH1, RAD51C, RAD51D D. RAD51C, RAD51D, BRIP1

D. RAD51C, RAD51D, BRIP1

Which of the following is FALSE regarding preventative surgery options for BRCA1/2 mutation carriers? RRM = risk reducing mastectomy A. RRM reduces risk for breast cancer by over 90% B. RR oophorectomy has been shown to reduce breast cancer risk in pre-menopausal women C. RR oophorectomy reduces ovarian cancer risk by 80-90% D. RRM reduces risk for ovarian cancer by 25%

D. RRM reduces risk for ovarian cancer by 25%

A Caucasian genetic counselor meets with a Hispanic woman who is being evaluated for Familial Adenomatous Polyposis (FAP). In an attempt to gain the client's trust and to promote their engagement, the counselor begins the conversation by sharing information about her own family's experience with cancer. This technique is based on: A. Attending language B. Congruence C. Shared language D. Reciprocity

D. Reciprocity

A couple returns for genetic counseling to discuss their genetic test results for cystic fibrosis (CF). Their daughter had an abnormal sweat test and sequencing indentified one delta F508 mutation and one variant of uncertain significance. Parental testing revealed that both the mutation and the VUS were maternally inherited. Which of the following should the genetic counselor do next? A. Tell the family their daughter is a carrier and no further testing is warranted. B. Tell the family that more research is needed to determine the significance of the VUS C. Recommend paternal uniparental disomy testing D. Recommend CFTR deletion/duplication studies

D. Recommend CFTR deletion/duplication studies

A 45-year old woman with premenopausal breast cancer comes for genetic counseling. She says that her mother died of "abdominal cancer" at age 65 years. The patient has been estranged from her family since several years before her mother's death and remembers only vague details about her mother's illness. Which of the following is the MOST appropriate next step by the genetic counselor to establish a diagnosis in the mother? A. Have the patient obtain her mother's death certificate B. Ask the patient to identify a family member who is more familiar with her mother's medical history C. Note the cancer in her record and continue to facilitate recall of other family cancers D. Request the deceased woman's medical records from her treating physician because consent is not required when a provider is the requestor

D. Request the deceased woman's medical records from her treating physician because consent is not required when a provider is the requestor ??? not sure if this is correct

A 41-year old pregnant Vietnamese woman at 15 weeks gestation comes for genetic counseling prior to amniocentesis. She speaks very little English and has brought her husband with her to act as an interpreter. The hospital's Vietnamese interpreter is not currently available. Which of the following is the BEST action by the genetic counselor? A. Call the patient's OB for advice on how to proceed B. Conduct the session with the husband translating C. Proceed with the amniocentesis with counseling afterwards D. Reschedule the appointment when a professional interpreter is available.

D. Reschedule the appointment when a professional interpreter is available.

Switching from transcription initiation to transcription elongation requires A. Recruitment of the U3 snRNP to the initiation site B. TBP binding to the TATA box C. TFIID changing conformations to allow binding of capping factors D. TFIIH adding phosphate groups to the C-terminal domain of RNA polymerase II

D. TFIIH adding phosphate groups to the C-terminal domain of RNA polymerase II

This term is used to describe an excessively rapid heartbeat A. Long QT syndrome B. Sinus rhythm C. Bradycardia D. Tachycardia

D. Tachycardia

A 7yo female with global developmental delay and MCA has a CNV of unclear significance on chromosome 16. Parental testing is performed and normal. The most likely interpretation is A. The parents are likely first or second degree relatives B. This information is not helpful for interpretation C. The CNV is de novo and more likely to be benign D. The CNV is de novo and more likely to be pathogenic

D. The CNV is de novo and more likely to be pathogenic

Assuming HW equilibrium, PKU is an AR metabolic disorder with an incidence of 1/10,000. Which of the following statements is most accurate? A. The frequency of the mutant allele is 1/1000 B. The frequency of the normal (wild-type) allele is 49/50 C. The carrier frequency is 1/100 D. The carrier frequency ~1/50

D. The carrier frequency ~1/50 incidence = q^2 = 1/10,000 q = sq root (1/10,000) = 1/100 p = 1 - q = 99/100 2pq = 2(99/100)(1/100) = ~1/50

Which of the following is NOT a limitation of panel testing? A. Reliable risk information may be limited for some genes B. Large panels have at least a 30% VUS rate requiring follow-up C. There is uncertainty regarding clinical validity and utility for many moderate risk genes D. The chance to detect a clinically significant mutation is lower than single gene testing

D. The chance to detect a clinically significant mutation is lower than single gene testing - Higher chance and a benefit, not limitation A-C true limitations

What does "sensitivity" mean in the context of a prenatal screening test? A. The accuracy of the test B. The likelihood that a fetus is affected when a test is positive C. The likelihood that a fetus is unaffected when a test is negative D. The likelihood that an affected fetus is correctly identified as positive

D. The likelihood that an affected fetus is correctly identified as positive Sensitivity = probability a person with the condition tests positive (affected with positive test / all affected)

What does "specificity" mean, in the context of a prenatal screening test? A. The accuracy of the test B. The likelihood that a fetus is affected when a test is positive C. The likelihood that a fetus is unaffected when a test is negative D. The likelihood that an unaffected fetus is correctly identified as negative

D. The likelihood that an unaffected fetus is correctly identified as negative Specificity = Probability an unaffected person will test negative (unaffected with negative test / all unaffected)

Which of the following is true about chromatin? A. About 250 bp of DNA wrap around a nucleosome core B. DNA is compacted about 100,000x in the interphase nucleus C. Genes are made more accessible by the degradation of histones D. The nucleosome core is made up of two molecules each of H2A, H2B, H3, and H4

D. The nucleosome core is made up of two molecules each of H2A, H2B, H3, and H4 Reasons other answers are false: - About 146 bp of DNA per nucleosome core - Each DNA molecule is packaged into mitotic chromosome that is 10,000-fold shorter than its extended length (During interphase, DNA is in chromatin, much less than 10,000x compacted) - Histones are modified, not degraded, to allow gene expression (acetylation associated with increased activity, methylation associated with decreased activity)

Which of these is NOT part of the mechanism of disease in fascio-scapulo-humeral MD? A. A large number of DNA repeats contracts to a much smaller number of repeats B. A previously silent gene is now expressed C. A polyadenylation signal is added to stabilize the mRNA D. The polypeptide is post-translationally cleaved

D. The polypeptide is post-translationally cleaved

You are asked to see a woman who has two brothers and a maternal uncle with non-specific XL mental retardation. Extensive testing has failed to identify the cause of the mental retardation in her brothers and uncle, therefore there is no testing available to accurately assess the risk that she is a carrier. Based on analysis of the pedigree, which of the statements below is accurate? A. the prior probability that she is a carrier is 1/4 B. The woman already has two unaffected sons. Therefore, the probability that she is a carrier can be calculated using Bayes to be 1/5 C. This means that the probability that her next son will be affected or that her next daughter will be a carrier is 1/5 D. The prior probability that she is a carrier is 2/3

D. The prior probability that she is a carrier is 2/3

Tay Sachs disease shows AR inheritance. Consider a couple with a healthy 2yo daughter, and an affected infant son. They have been referred for genetic counseling. All of the following statements are incorrect EXCEPT A. The probability that their next child will be affected is 1/2 B. The probability that the older unaffected sister of the affected child is a carrier is 1/2 C. The fact that their last child was affected means that their next three children will not be affected D. The probability that each parent is a carrier is 1

D. The probability that each parent is a carrier is 1 A. No, prob is 1/4 B. No, chance carrier is 2/3 because we know she is unaffected C. Common misconception for parents D. Parents are most likely to be obligate carriers

The most serious tumor risk associated with DICER1 syndrome is which of the following? A. Bladder rhabdomyosarcomas B. Nasal mesenchymal hamartomas C. Sertoli Leydig ovarian tumors D. Type 2/3 pleuropulmonary blastomas

D. Type 2/3 pleuropulmonary blastomas A, B, and C can occur but PPB is highest risk (20%, may be 40% for females). Cystic nephromas also highly characteristic

Which of the following conditions is NOT caused by a defect of alpha dystroglycan glycosylation? A. Walker Warburg syndrome B. Muscle Eye Brain disease C. Fukuyama congenital MD D. Ulrich congenital MD

D. Ulrich congenital MD

Organ transplantation is most curative in which category of metabolic disease? A. Mitochondrial disorders B. Fatty Acid Oxidation Disorders C. Organic Acidemias D. Urea Cycle disorders

D. Urea Cycle disorders

A pregnant woman has an ultrasound that identifies a lumbar meningomyelocele in the fetus. Which of the following exposures is most closely associated with this birth defect? A. Tetracycline B. Lithium C. Phenytoin D. Valproic acid

D. Valproic acid

Which of the following statements about ACMG guidelines incidental findings in WES is false? A. Constitutional PVs found in the genes on the minimum list should be offered as an option to be reported by the lab B. Lab may choose to analyze additional variants not included on the minimum list C. Variants should only be reported that are identified on constitutional samples (not tumor tissues) D. Variants in adult onset conditions should not be reported if the proband is a child

D. Variants in adult onset conditions should not be reported if the proband is a child

Which of the following genes, when mutated, causes Denys-Drash syndrome and Frasier syndrome? A. PTCH1 B. SBDS1 C. SMARCA4 D. WT1

D. WT1 Denys-Drash: point mutations Frasier syndrome: splice site mutations PTCH1 - nevoid basal cell carcinoma syndrome (NBCCS) SMARCA4 (2 associations) - Rhabdoid tumor predisposition - Coffin-Siris syndrome: DD, abnormalities of the fifth (pinky) fingers or toes, and characteristic facies (coarse).

Which of the following scenarios do NOT require Bayesian analysis to get the right answer? A. Chance that an individual is a carrier of DMD, given that she has two unaffected sons B. Chance that person inherited HD from affected father, an AD adult onset condition with variable age of onset C. Chance that person inherited Alagille syndrome from father, AD condition with 80% penetrance D. What is the chance that a person is a carrier of a mutation causing hearing loss, given that his mother has an AR form, but has two unaffected sons

D. What is the chance that a person is a carrier of a mutation causing hearing loss, given that his mother has an AR form, but has two unaffected sons We don't need to uncorporate two unaffected sons for an AR condition

Which of the following can cause hypertrophic cardiomyopathy? A. aging B. hypertension C. single gene mutations D. all of the above

D. all of the above

Which of the following is NOT a dystrophinopathy phenotype? A. isolated cardiomyopathy B. exercise induced myalgias (muscle pain) C. DMD D. distal arthrogryposis

D. distal arthrogryposis

The only FDA-approved medication for use of nausea and vomiting in pregnancy is A. ondansetron (Zofran) B. diphenydramine (Benadryl) C. promethazine (Phenergan) D. doxylamine/pyridoxine (Diclegis) E. metacolopromide (Reglan)

D. doxylamine/pyridoxine (Diclegis) FDA recently approved Diclegis (doxylamine-pyridoxine) as only med approved for N&V during pregnancy due to a possible association of ondansetron with 2-fold increase in CHD (background 1% increased to 2%)

Genome-wide association studies? A. employ epidemiological methods used in performing cohort studies B. look for polymorphisms of single nucleotides with a minor allele frequency of less than 1% C. have led to the development of tests which can accurately predict risks for common diseases D. elucidate important information about biochemical pathways E. all of the above

D. elucidate important information about biochemical pathways Why are others false? A. GWAS uses case-control NOT cohort studies B. SNPs used in GWAS are able to tag only common variants that typically have minor allele frequencies >5%. C. We cannot accurately predict common disease

Which of the following is not a function of mitochondria? A. energy production B. calcium homeostasis C. steroid biosynthesis D. glycosylation

D. glycosylation Regulation of cellular functions: - Energy production - Calcium homeostasis - Apoptosis - Radical species generation - Radical species scavenging - Steroid biosynthesis - Orchestrate Metabolism

Congenital myotonic dystrophy A. is an AR, lethal condition B. is inherited from an affected father in the majority of cases C. occurs in ~30% of children of an affected parent D. is always inherited from an affected parent even if the parent has sub-clinical symptoms

D. is always inherited from an affected parent even if the parent has sub-clinical symptoms

Which of the following is the MOST likely mechanism of tumor occurrence in patients who have deleterious mutations in tumor suppressor genes? A. alternative splicing B. errors in DNA mismatch repair C. hypermethylation D. loss of function

D. loss of function

In nonsenses mediated decay, A. Truncated proteins due to nonsense mutations are rapidly degraded by the cell B. PolyA binding protein interacts with the CAP complex to signal the presence of a premature termination codon C. Nonsense mediated decay is most efficient when the premature termination codon is in the last exon of an mRNA D. mRNAs with premature termination codons upstream of the last exon junction are degraded after one round of translation

D. mRNAs with premature termination codons upstream of the last exon junction are degraded after one round of translation

deoxyadenosine triphosphate is a(n)? A. amino acid B. base C. nucleoside D. nucleotide

D. nucleotide Why? - Nucleotides Vs Nucleosides. Building block of DNA v RNA

The classic infantile form of Pompe disease A. typically presents with seizures B. is easily treated with enzyme replacement therapy C. rarely affects the heart D. occurs when there is less than 1% of GAA enzymatic activity

D. occurs when there is less than 1% of GAA enzymatic activity

In the same example, how would you classify the HEXA findings for the proband? A. unimportant finding B. diagnostic finding C. medically actionable finding D. secondary finding

D. secondary finding

The cytochrome P450 2C19*2 allele A. is associated with decreased likelihood of residual platelet aggregation following angioplasty in individuals treated with clopidogrel (Plavix) B. is found most frequently in Caucasians C. only impacts residual platelet aggregation if homozygous D. when present, suggests the antiplatelet drugs other than clopidogrel should be considered E. is the only CYP2C19 allele resulting in reduced enzyme activity

D. when present, suggests the antiplatelet drugs other than clopidogrel should be considered

What percentage of fetuses with Down syndrome SAB during pregnancy? A. ~10% B. ~25% C. ~50% D. ~75% E. >99%

D. ~75%

XL transmission of DCM

DMD TAZ: Barth syndrome (3-Methylglutaconic aciduria type II) - congenital cardiomyopathy, underdeveloped skeletal musculature and muscle weakness, short stature, and neutropenia

In-frame v out-of-frame mutations (del/dups): syndrome example

DMD v BMD DMD is more severe because it more often involves an out-of-frame mutation (del/dup) - Wheelchair bound by 12yo as opposed to slower progression in BMD (reduced gene function rather than absent)

Segmental UPD as a result of somatic (mitotic) recombination (post-zygotic)

Daughter cells which receive one recombinant and one non-recombinant chromosome have segmental UPD Results in segmental paternal and maternal uniparental isodisomy UPD will be mosaic as cell line exists which did not undergo somatic recombination Seen in Beckwith Wiedemann Syndrome

Sudden cardiac death

Death from an abrupt loss of heart function - within 1 hour of onset of cardiac symptoms - natural, rapid, unexpected Symptoms: chest pain, palpitations, dizziness, lightheaded, syncope (fainting) 25-50% have no prior heart medical hx After autopsy, - 95% due to structural disease (coronary artery disease, MI, cardiomyopathy, CHD, arrhythmogenic right ventricular dysplasia/cardiomyopathy) - 5% due to an apparently structurally normal heart (primary heart rhythm abnormalities, including Long QT, Brugada, etc.)

When we see a child with an anomaly due to twin constraint, that is most likely a...? Deformation Disruption Dysplasia Malformation

Deformation

Which of the following names are associated with 22q11.2 deletion syndrome?

DiGeorge syndrome Velocardiofacial syndrome Conotruncal anomaly face syndrome

A person who has heavily thickened long bones, particularly along the shaft, is most likely to have Diaphyseal dysplasia Epiphyseal dysplasia Metaphyseal dysplasia Microphyseal dysplasia

Diaphyseal dysplasia

Case example: infant with kidney mass - A 6mo male infant is found to have a unilateral kidney mass, which could be early stage Wilms' tumor or cystic nephroma. - Infant has no hx of congenital birth defects or DD. - Family hx: Father had a thyroidectomy at age 18 due to thyroid nodules. No history of cancer on either side of the family. What is the differential genetic diagnosis? What genetic tests would you order?

Differential dx: Familial Wilms' tumor ◦ Possible if tumor is Wilms'; also consider BWS; young age at dx makes familial Wilms' more likely; lack of bilaterality or GU anomalies makes it less likely DICER1 syndrome ◦ Very likely if mass is a cystic nephroma; father's thyroid nodules could also be DICER1-related Li-Fraumeni syndrome ◦ Any abdominal tumor dxd <1 yr could be LFS Sporadic kidney mass ◦ May be non-syndromic since it is unilateral; and no fa, hx of kidney tumors/cysts. Conclusion: DICER1 (major features = PBB and cystic nephromas, but can also have Wilms tumor and thyroid tumors)

Case example: child with BMF and AML Female child, age 4, presents with AML; hx of frequent infections & BMF. - Thumbs appear normal, but less reactive muscle tone on palms of both hands. - Family History: ◦ Maternal great-aunt had OV ca, age 65. ◦ Paternal grandmother had BR ca, age 40. ◦ No AJ ancestry. What is the differential diagnosis? What genetic tests would you order?

Differential dx: Fanconi anemia (FA genes, esp. FANCD1 = BRCA2) + chromosome breakage studies - Very suspicious due to fam hx of breast and ovarian cancer Familial AML - RUNX1 and CEBPA (AML predisposition genes) LFS (TP53) - Early onset acute leukemia is feature of LFS Sporadic leukemia - Leukemia is most common childhood cancer, most cases sporadic Conclusion: chromosome breakage showed hypersensitivity to DEB (DNA damaging agent) --> genetic testing revealed homozygous BRCA2 --> FA

Case example - fam hx of pancreatic cancer - patient, age 45, has multiple nevi, including a few small dysplastic nevi - she is of irish ancestry, and has a hx of excessive sun exposure and burns - her father and paternal uncle died from panc ca in 60s. Both had heavy +EtOH - Paternal grandmother had breast cancer at age 60 and paternal grandfather had CRC in 50's Differential dx and conclusion?

Differential dx: Lynch syndrome ◦ 2 generations of GI cancers FAMMM ◦ Pt's hx of dysplastic nevi plus fam hx of PANC ca HBOC ◦ Fam hx of BR ca and PANC cancer Sporadic ◦ No early onset cancers and possible contributing environmental risk factors present in family Conclusion: CDKN2A mutation (FAMMM)

Neuroendocrine case: fam hx pheo. Patient, cancer-free at age 38, reports the following family history. - Her sister had died suddenly of an MI at age 41 after running a marathon. - Autopsy revealed a pheochromocytoma. - Their mother had died at age 45 from complications of a surgical procedure. Give differential dx and conclusion

Differential: MEN2 (RET) - lack of MTC in family makes this dx less likely VHL (VHL) - age at sister's dx fits VHL; variable expression Hereditary PGL/Pheo - Age at sister's dx would fit SDHX mutation Sporadic PCC NF1 - Not syndromic - Mother's death could be unrelated; could be an isolated pheo w/ ~50% chance of negative test Conclusion - SDHB mutation

What is the main cause of death in DMD and BMD?

Dilated cardiomyopathy

Cardiomyopathy

Disease of the heart muscle (myocardium) Can lead to heart failure (swelling of lower extremeties, dyspnea (shortness of breath), risk for arrhythmia, stroke, and sudden cardiac death) Types: - Dilated cardiomyopathy (DCM) - Hypertrophic cardiomyopathy - Restrictive cardiomyopathy (scar tissue replaces normal heart muscle making ventricles stiff and reducing blood flow) - Arrhythmogenic Right Ventricular Dysplasia (ARVD) (muscle tissue in right ventricle dies and is replaced with scar tissue disrupting the heart's electrical signals

When a person has an amniotic band related limb reduction defect, that is most likely a...? Deformation Disruption Dysplasia Malformation

Disruption

FXTAS Does not occur in individuals with non-mosaic FMR1 full mutations Typically has an onset in early adulthood Exclusively affects males with FMR1 premutations Is caused by absence of Fragile X Mental Retardation Protein (FMRP) Non of the above

Does not occur in individuals with non-mosaic FMR1 full mutations Only occurs in premutation carriers: male and female Onset in late adulthood Premutation carriers have FMRP still

The correct term fro increased anterior-posterior length of the head is Brachycephaly Dolichocephaly Macrocephaly Microcephaly

Dolichocephaly

Domain and intensity of emotion

Domain - type of emotion - Feelings are either pleasant (e.g., happy) or unpleasant (e.g., sad) Intensity - the degree or level of emotion - Mild (e.g., irritated) to extreme (e.g., furious)

When a person has achondroplasia, that is a...? Deformation Disruption Dysplasia Malformation

Dysplasia

What is the largest known gene?

Dystrophin - Spans 2.4 Mb - 79 exons encoding the protein - Reason why prone to del/dups in DMD/BMD

If a patient is suspect for Limb Girdle Muscular Dystrophy, what conditions do you first want to rule out by looking to the inheritance pattern?

Dystrophinopathies (DMD/BMD/DCM). They are XL and should be ruled out before a diagnosis of LGMD is made

Current guidelines offering the following carrier screening to all women in pregnancy A. ACOG and ACMG recommend that CF carrier screening be offered to all women in pregnancy B. ACOG and ACMG recommend that SMA carrier screening be offered to all women in pregnancy C. ACMG recommends that carrier screening for SMA be offered to all women in pregnancy; ACOG recommends against universal offering for prenatal SMA carrier screening D. ACOG recommends that carrier screening for SMA be offered to all women in pregnancy; ACMG recommends against universal offering for prenatal SMA carrier screening E. A and B

E. A and B

Factors that influence CGG repeat expansion during maternal transmission of FMR1 include A. The presence of AGG interruptions B. Maternal CGG repeat size C. Fetal gender D. All of the above. E. A and B

E. A and B (the presence of AGG interruptions and maternal CGG repeat size)

First line genetic testing recommendations for the evaluation of children with developmental disabilities currently include A. CMA B. WES C. FMR1 analysis D. All of the above E. A and C

E. A and C (CMA and FMR1)

What are the characteristics of an ideal prenatal screening test? A. Sensitivity must be high B. Specificity must be low C. Screening test must be easy and inexpensive to perform D. A and B are correct E. A and C are correct

E. A and C are correct Ideal prenatal screen: • High sensitivity - Identifies a high percentage of affected individuals • High specificity - Does not alarm a high percentage of unaffected individuals • Positive early enough in gestation to allow maximal options and safety • Easy and inexpensive to perform

If a likely PV is identified in an AD gene through WES, what is the next step? A. parental testing B. sanger (or comparable technology) sequencing C. co-segregation analysis D. A and B E. A, B, and C

E. A, B, and C

Genetic counseling for carrier screening should include which of the following? A. Patient education B. Informed consent C. Review of the limitations of screening D. Reproductive choices including prenatal dx, donor gamete, adoption E. All of the above

E. All of the above

SMA is characterized by A. Symmetric proximal muscle weakness B. Progressive degeneration of lower motor neurons C. Variable expression ranging from infantile to adult onset of symptoms D. Preservation of intellectual development E. All of the above

E. All of the above

The reasons for the change from the Pregnancy Categories classification to the Pregnancy and Lactation Labeling Rule include A. Incorrect assumption of dichotomy risk B. Not updated with new information C. Lack of inclusion of timing and dose D. No information on maternal illness E. All of the above

E. All of the above

What is the value of identifying a CNV prenatally? A. Referral to appropriate specialists B. Offering prenatal dx in subsequent pregnancies C. Enabling appropriate surveillance during the pregnancy D. Providing an accurate recurrence risk E. All of the above

E. All of the above

Which of the following can cause false positive or negative results in NIPS? A. Confined placental mosaicism B. Fetal mosaicism C. Maternal mosaicism D. Vanishing twin pregnancy E. All of the above

E. All of the above

Which of the following disorders would be identified by CMA but not karyotype? A. Williams B. Smith magenis C. Prader willi D. None of the above E. All of the above

E. All of the above

Fetal hydantoin syndrome includes all of the following except: A. Growth restriction B. Microcephaly C. Oro-facial anomalies D. Limb defects E. All of the above may be included

E. All of the above Fetal hydantoin syndrome occurs through exposure to phenytoin (an AED)

The PPV for a screening test is most affected by A. Test sensitivity B. Prevalence of the disease being screened in the population C. Test specificity D. A and B E. B and C

E. B and C Unsure if this is accurate (specificity more than sensitivity?)

How is pan-ethnic carrier screening different from standard ethnicity screening? A. Would have a lower screen positive rate than ethnicity screening B. Typically tests for more disorders C. Panel is not customized by ethnic background D. A and B are correct E. B and C are correct

E. B and C are correct

What is a likelihood ratio? A. The probability of having an affected fetus B. The probability of having an unaffected fetus C. The risk of having an affected fetus given a particular test result D. The risk of having an unaffected fetus given a particular test result E. Both C and D are correct

E. Both C and D are correct

Which gene(s) is/are most relevant when counseling for Parkinson disease in a patient with Eastern European Jewish descent? A. LRRK2 B. GBA C. SNCA D. All of the above E. Both a and b are correct

E. Both a and b are correct

Which disorder(s) have better detection rates using biochemical methodology as compared to molecular testing? A. Tay-Sachs B. SMA C. Sickle cell disease D. Both a and b E. Both a and c

E. Both a and c

All of the following exposures have been associated with a risk of causing orofacial clefts except A. Prednisone B. Lamotrigine (Lamictal) C. Phenytoin D. Cigarettes E. Bupropion (Wellbutrin)

E. Bupropion (Wellbutrin)

The most common adverse outcome from a prenatal CMV infection is A. cerebral calcifications B. IUGR C. CHD D. Petechiae E. Hearing loss

E. Hearing loss Most common: SNHL Other: chronic skin rashes, petechia, epilepsy, hepatosplenomegaly, cerebral palsy, chorioretinitis, optic atrophy, blindness, microcephaly, ID, prematurity, low BW, FTT

Ms. X is a G3P2 woman who delivers a male baby with ToF at 38w gestation. She had recurrent yeast infections in the second trimester that were treated with miconazole (Monistat) and fluconazole (Diflucan). At 30 weeks patient was prescribed zolpidem (Ambien) which she took occassionally for sleep. Which medication caused the heart defect? A. combination of both drugs B. Miconazole C. Fluconazole D. Zolpidem E. None of the above

E. None of the above Heart development occurs between 8-11 weeks gestation (first trimester)

Imprinting plays a role in the phenotype in all but which of the following conditions A. BWS B. Neonatal diabetes C. Triploidy D. Angelman E. T13

E. T13 Chrom 13 not imprinted Major cause of neonatal diabetes is aberrant expression of imprinted genes at chromosome 6q24

Occult trisomies can be suggested by what finding? A. Chromothripsis B. Cryptic deletions C. IBD D. Multiple duplications E. UPD

E. UPD UPD can confer risk for several abnormalities depending on level and affected chromosomes: 1. Imprinted syndromes 2. Recessive disorders (obtaining 2 mutated genes from one parent) 3. Occult trisomy - low level of trisomy mosaicism

Appropriate implementation of genomic medicine should lead to A. earlier disease detection B. reduction in healthcare costs C. increased patient compliance D. improved health outcomes E. all of the above

E. all of the above

Precision oncology utilizes genomic technology to A. predict disease prognosis B. detect cancer in the pre-clinical state C. identify mutations in genes associated with target chemotherapy agents D. detect disease recurrence E. all of the above

E. all of the above

Challenges of tumor NGS include all of the following except? A. low tumor content of the pathology specimen B. tumor heterogeneity C. presence of aneuploidy D. incidental findings E. low coverage with whole tumor exome sequencing

E. low coverage with whole tumor exome sequencing Why? Challenges: - Low tumor content = cancer cells may only be a fraction of the total sample - multiple sub-clones = heterogeneity within tumor specimen - aneuploidy = may modify mutation abundance - need to r/o germline mutations = issue of informed consent and pre/post GC - demonstration of clinical utility including cost effectiveness

Electrocardiogram (ECG/EKG) v Echocardiogram

ECG records electrical activity of heart using electrodes attached to skin Echo (typically transthoracic echocardiogram (TTE)) uses ultrasound to visualize heart

Polar positively charged (basic) amino acids

Each of their side chains contains an additional amino group beyond what is found in the core structure of the amino acid. They therefore function as basic molecules that can accept a hydrogen atom from the existing environment. A BASIC man needs LYSine for HIS ARG-ggravated cold sore lysine, histidine, arginine

NBS for CF was endorsed by the CDC in 2004. The benefits to children with CF diagnosed by this method include all but the following Improved nutritional status Improved cognition Stable pulmonary status Earlier colonization with pseudomonas

Earlier colonization with pseudomonas *Pseudomonas is one of the major pulmonary pathogens in patients with CF

BAC array

Early methodology of comparative genomic hybridization (CGH) - bacterial artificial chromosomes Replaced with oligonucleotide arrays because oligo has higher resolution than BAC (50-60 bp segments, compared to 75,000 - 150,000 bp segments)

UPD associated with a robertsonian translocation Ex: maternal t(14:22), paternal normal

Either end up with monosomy 14 or trisomy 14 - monosomy rescue = paternal uniparental isodisomy 14 + robertsonian translocation - trisomy rescue = maternal uinparental heterodisomy 14 + robertsonian translocation

Which neuromuscular condition typically has this triad of features: contractures in childhood, progressive muscle weakness/wasting, and cardiac features?

Emery-Dreifuss Muscular Dystrophy. Contractures occur early in the elbows, heels, and neck, slowly progressive muscle weakness/wasting, and cardiac conduction and arrhythmias that most often occur after the 2nd decade

Monosomy rescue

Endoduplication (monosomic chromatid replicates itself) - ALWYAS results in uniparental isodisomy

Which of these is not one of the phases of transcription? Elongation Excision Initiation Termination

Excision

If the distribution of SNP risk alleles is very similar in people with the disease and people without the disease, you will have good discriminatory accuracy. True or false?

False

Fragile X syndrome: underlying mechanism

Expansion of the CGG repeat leads to methylation of CpG islands in promoter Methylation shuts off transcription Findings in Fragile X syndrome are due to loss of FMRP

HD genetics

Expansions of CAG triplet within HTT gene (Huntingtin protein) encoding glutamine in the coding sequence of the gene Like Fra(X), HD shows anticipation, but UNLIKE Fra(X), HD alleles generally expand when inherited PATERNALLY Fully penetrant = 40 or more repeats, age of onset has some correlation with repeat length (juvenile >60)

Pre-test counseling for CMA

Explain chromosomes Detection rate (~15-20%) depending on indication Limitations: - no structural info (inversions, balanced trans) - not validated for low-level mosaicism, but may detect it - CNVs of unknown significance Possible results: - positive --> diagnostic - negative --> uninformative - CNV of unclear sig and potential of parental testing Possibility of unexpected results: - medically important CNV unrelated to testing indication - carrier status - consanguinity or non-paternity

What is the significance of the Lyon hypothesis?

Explains manifestations of XL disorders Explains variability of clinical manifestations in females Explains difficulty in biochemical carrier detection in female carriers

The dx of antiphospholipid antibody syndrome can be made on the following: A. One episode of venous thromboembolism B. Laboratory analysis C. 2 fetal losses >10 weeks D. One stillbirth >34 weeks GA E. Recurrent miscarriage F. All of the above G. None of the above

F. All of the above But requires lab criteria and one clinical criteria together to make dx

Radial ray defects, such as a missing or extra thumb, is present in about 50% of patients with which syndrome? Prader-willi Beckwith-wiedemann Fanconi anemia Stickler

FA

In the absence of any other family history of a complex disorder, the chance of recurrence for which kind of relative approximates the square root of the frequency of the condition in the general population? FDR SDR TDR Fourth degree relative

FDR

Current 22q del detection methods include...?

FISH and MLPA

True or False: All patients have the same size 22q deletion

False

Validated SNP associations have been demonstrated to account for 30% of the known heritability of disease X and 5% of the heritability of disease Y. It is likely that many more SNP associations have been identified for disease X as compared to disease Y. True or false?

False

personalized genomic medicine will likely be implemented on a broad population scale in the very near future: true or false

False

A GWAS identified a SNP-disease association with a significance of px10^-3. This SNP should be included in performing genomic risk profiling for this disease. True or false?

False Need p less than or equal to 5x10^-8 because testing multiple associations simultaneously "multiplies" FPR

When we use the term "personalized medicine." what we really mean is the application of genomics to medicine. True or false.

False Personalized medicine DOES NOT = genomic medicine. Personalized medicine includes other factors as well (e.g., environment, genomics, fam hx, response to treatment)

NIPT analyzes exclusively fetal DNA fragments residing in the maternal plasma: True or False

False The fragments are a combination of maternal and placental (trophoblast)

True or False: Most 22q deletions are familial

False - 93% de novo - 7% inherited

True or False: 22q11.2 deletion is the most common cause of CHD

False T21 is

True or false: 45,X is typically due to loss of maternal X chromosome

False, maternal X present in 80% of cases

True or False: CAA repeats interrupting CAG repeats in the ATXN2 gene causing Spinocerebellar ataxia reduces severity of the condition

False, this also results in glutamine. However the more CAA repeats the more stable the expansion making it less likely to expand in the next generation.

True or False: Myotonic Dystrophy Type 1 is caused by CAG repeats in the DMPK gene

False. CTG repeats in DMPK gene

True or False: There has been no documented cases of an HD phenotype appearing in a person with an "intermediate allele".

False. Case of man with movement symptoms with intermediate allele

What neuromuscular condition is caused by a contraction of a repeat that leads to features similar to Limb Girdle?

Fascioscapulohumeral muscular dystrophy (FSHD) AD, 70-90% of people inherit the D4Z4 deletion from a parent. Typically repeat is 11-100 but symptoms occur when the repeat is 1-10.

Zygote comparison: Meiosis I v II error after trisomy rescue

Focus on the centromere! H comes before I Heterodisomy at the centromere = Meiosis I error Isodisomy at the centromere = Meiosis II error

Selection against XL-R mutations

For an XL-R condition with f = 1, 1/3 of the mutant alleles are in males and 2/3 are in females. For XL-R conditions in which f = 0 (e.g., DMD), 1/3 of mutant alleles are lost with each generation through selection and therefore must be replaced through new mutation. Therefore, 1/3 of isolated males (i.e., no family history) with a genetically lethal (f = 0) X-linked disorder are the result of a new mutation, while for 2/3 of isolated affected males, their mothers are unaffected carriers.

First cousin mating

For any allele that the father passes on to his child, the likelihood that the mother inherited the same allele from their common ancestors is 1/8 (i.e., first cousins share on average 1/8 of their genes in common, this is known as the coefficient of relationship (R)). Then, the likelihood that the mother would pass on this allele to her child is 1/2, and F (the coefficient of inbreeding of the child) equals: 1/8 x 1/2 = 1/16 for the child of a first cousin mating

Mutation rate for AR conditions

For autosomal recessive conditions, the probability that both parents are carriers is much greater than the probability that one or both mutant alleles inherited by the affected child were the result of a new mutation. Since such a large number of mutant alleles exist in the population (in carrier heterozygotes), the parents of an affected child are considered to be obligate heterozygote carriers.

Genome wide UPD (uniparental diploidy)

Four scenarios (ALL MOSAIC paternal genome-wide uniparental isodisomy): 1. Fertilization of an egg by a normal sperm followed by replication without division and haploidy rescue 2. Fertilization of an egg by a diploid sperm followed by triploidy rescue and haploidy rescue 3. Fertilization of an egg by two sperms followed triploidy rescue and haploidy rescue 4. Fertilization of an empty egg by normal sperm + zygote fusion followed by haploidy rescue

What is the most common known cause of ASD?

Fragile X ~1/20 children with ASD have FXS

Individuals with a history of ID or autism in the family should be offered...?

Fragile X carrier screening

Which three ethnic groups has a 1/30 carrier rate for Tay Sachs?

French canadian, Cajun, and AJ

What is the only autosomal recessive trinucleotide repeat disorder?

Friedreich ataxia

The family in the previous question comes from a region in Greece where the incidence of this condition is 5%. What is the carrier frequency in males and in females?

G6P is XL-R. So, the male HWE = p + q = 1. So male carrier frequency = q = 0.05. Female HWE = p2 + 2pq + q2 so female carrier frequency = 2pq = 2(95/100)(5/100) = 9.5/100 = ~0.10

Methylation

Gene silencing An "imprinted gene" is a gene that is silenced due to parent-of-origin - A "paternally imprinted gene" is paternally silenced - A "maternally imprinted gene" is maternally silenced

Exception to HW "population is infinitely large" - genetic drift

Genetic drift- when the pool of gametes is formed for the next generation it represents a random sample of alleles from the population. Variation observed in the frequencies of the alleles from one generation to the next is due to this random sampling. The effects in a large population are negligible but can be very influential in a small population.

5 tenets of genetic counseling

Genetic information is key Relationship is integral to genetic counseling Patient autonomy must be supported Patients are resilient Patient emotions make a difference

Silencing through imprinting

Genomic imprinting results in differential expression of alleles at a single locus, depending on whether the allele is inherited from the father or from the mother. Imprinted genes are silenced by modifications to chromatin, and are not a consequence of sequence alteration. - Triggered by METHYLATION For most genes (but not imprinted genes), DNA methylation is removed around the time of fertilization - Allows early embryonic cells to be totipotent New methyl groups begin to be added at the time of implantation - Initiates regulation, restricting fate of cells

Linkage studies are most powerful when applied to detect genetic differences that Have a very large effect on the phenotype of interest Have a smaller effect on the phenotype of interest Are influenced by environmental factors Are very common in the population

Have a very large effect on the phenotype of interest

When measuring the eyes of a person who has telecanthus, you would see...? Greater than average inner canthal distance, normal interpupillary distance Normal inner canthal distance, greater than average interpupillary distance Greater than average inner canthal distance, greater than average interpupillary distance Normal inner canthal distance, normal interpupillary distance

Greater than average inner canthal distance, normal interpupillary distance Telecanthus is the SAME thing as dystopia canthorum (term used in Waardenburg lecture) = lateral displacement of inner canthi, making inner canthal distance greater and giving the appearance of pupils wide nasal bridge + medial pupils

Polar (hydrophilic), neutral amino acids

Have a partial positive or negative charge in the -R groups; form hydrogen bonds as proton donors or acceptors (typically contain electronegative Atoms: S, O, N) Serine, threonine, tyrosine, asparagine, cysteine, glutamine, and glycine

Traditional association studies are useful in detecting genetic differences that Have a very large effect on the phenotype of interest Have a smaller effect on the phenotype of interest Are influenced by environmental factors Are very common in the population

Have a smaller effect on the phenotype of interest

What does MELAS stand for?

Mitochondrial encphalomyopathy Lactic Acidosis Stroke-like episodes

On a level II ultrasound, a baby is found to have pleural and pericardial effusion along with generalized edema. What condition should you suspect?

Hb Barts

Kidney cancer case: Patient presented with metastatic kidney cancer at age 35. He had been worked up and treated at an out-of-state hospital. - Tumor pathology was read as a mixed, possibly papillary tumor. Attempts to have tumor re-reviewed were unsuccessful. - No fam hx of cancer, skin leiomyomas, retinal angiomas or lung blebs/pneumothorax. - Pt's mother and maternal aunt had hyst @ 40, but pt did not know why. Differential and conclusion?

Hereditary Leiomyomatosis (FH) - Renal Cell Cancer - Given aggressive tumor & fam hx of hyst, very possible Hereditary Papillary Renal Carcinoma (MET) - Due to lack of clear pathology or hx of skin leiomyomas Birt Hoge Dube (FLCN) - Should be considered whenever path is complex VHL (VHL) - Most common RCC gene, but usually clear cell Sporadic kidney cancer - Many pts w/ early onset RCC end up with neg testing Conclusion: HLRCC (FH mutation)

What condition should first be of first concern when we detect transposition of the great arteries?

Heterotaxy

Heterozygote advantage

Heterozygotes for the sickle cell allele are more fit due to a lesser susceptibility to malaria than either homozygote normals or affecteds. This is called balancing selection, where selective forces are acting both for and against a particular mutant allele. This results in a stable polymorphism, with relatively high allele frequency.

Homologous recombination

Homologous recombination system can be used to repair damage - Occurs during late S/G2 of the cell cycle (i.e., after DNA replication). - Uses sister chromatid as template for repair of damaged DNA strands Genetic recombination involves the following processes: ◦ strand displacement ◦ ligation ◦ branch migration ◦ duplex separation

HD underlying mechanism

Huntingtin appears to play a critical role in nerve function. Unlike Fragile X, the trinucleotide repeat in HD occurs within the coding region of the gene - The CAG repeat increases the number of glutamine residues in the protein. - Mechanism of toxicity is not completely understood, but may be due to aggregation

Your patient is a 2yo girl with a mutation in the MECP2 gene. Which clinical feature would you not expect to see? Developmental regression Hand wringing Microcephaly Hypertelorism

Hyperteolorism Classic Rett syndrome: - Normal development first 6-18mo - Developmental stagnation → regression (loss of language and purposeful hand movements) - Stereotypic hand movements (like hand washing) - Seizures in up to 90% - ID/acquired microcephaly

A genetic disorder resulting from mutations in the IKBKG (previously NEMO) gene is characterized by blistering of the skin (birth to age ~4mo), followed by a wart-like rash (for several months), swirling hyperpigmentation (age ~6mo into adulthood), and linear hypopigmentation. What condition is this? Incontinentia pigmenti Epidermolysis bullosa Cutis laxa Neurofibromatosis

IP

There are five classes of CF mutations. Some mutations in the CFTR gene retain partial function of the protein and are associated with a milder phenotype including pancreatic sufficiency. These mutations are classified as type I and II II and III III and IV IV and V

IV and V

Coping behaviors: plan

Identifies next steps and follows through on them

Specificity

If a person does not have a disease, how often will the test be negative? Highly specific tests mean a positive result is very likely to be real Specificity = true negatives / (true negatives + false positives)

Sensitivity

If a person has a disease, how often will the test be positive? Highly sensitive tests mean a negative result is very likely to be real Sensitivity = true positives / (true positive + false negative)

Selection against AD mutations

If f = 0 (complete selection against the mutant allele), then almost all affected individuals represent new mutations. (e.g., thanatophoric dysplasia ). Mutation is eliminated in only one generation. If f = 1 (no deleterious effect on reproduction for carriers of the mutant gene) then s = 0 and there is no selection against the mutant allele. Virtually all affected individuals inherited the gene from an affected parent (e.g , Huntington disease). If 0 < f < 1, a proportion of affected individuals have an affected parent, while some are the result of a new mutation.

Positive predictive value

If the test result is positive, how likely is the person to have the disease True positive / (true + false positives)

General principles of TORCH infections

In general, the earlier the TORCH infection occurs during pregnancy, the more severe the complications. Primary infection much worse than secondary infection Woman infected during pregnancy --> production of IgM antibodies followed by IgG antibodies - IgM antibodies against TORCH organisms usually persist for about 3 months - IgG antibodies remain for a lifetime, provide immunity and prevent or reduce the severity of reinfection - If IgM antibodies are present in a pregnant woman, a current or recent infection with the organism is predicted

Detecting consanguinity through CMA

In long ROH, % IBD can be estimated by the sum of the sizes of the homozygous segments divided by the total autosomal genomic length - ~2,881 Mb for GRCh37/hg19 Labs can include the % of the genome that is homozygous - Automated calculation (typically inflated by small regions of homozygosity that are more likely representative of regions of suppressed recombination) Limiting this calculation to segments >2-5 Mb is more likely to result in the inclusion of segments that are truly IBD

These exceptions to random mating tend to...?

Increase the proportion of homozygotes while decreasing the proportion of heterozygotes in a population Can have significant effects on Hardy-Weinberg equilibrium

ACMG 2016 statement on NIPT

Inform all pregnant women: - NIPS is the most sensitive screening option for traditionally screened aneuploidies (ie. Patau, Edwards, and Down syndrome) - of availability of expanded use for sex chromosome anueploidies - of availability of expanded screening for clinically relevant CNVs IF...PPV can be provided Does NOT recommend - Screening for autosomal aneuploidies other than 13, 18, 21 - Screening for genome wide CNVs - Repeating no-call NIPS results --> rather offer diagnostic testing Professional societies don't recommend MSS or NIPT over one another at this time --> reason why screening options is always a discussion with the patient

Uniparental disomy

Inheritance of both homologues of a chromosome from only one parent

Long QT syndrome (LQTS)

Inherited or acquired Prolonged QT interval Characteristic polymorphic ventricular tachycardia - Torsades de pointes = "twisting of points" - May result in syncope or ventricular fibrillation and SCD

Protein synthesis: three stages?

Initiation: recognition of the start site for translation Elongation: polymerization of the polypeptide chain Termination: end of protein synthesis

Reciprocal translocation

Interchange of genetic material between NONHOMOLOGOUS chromosomes Balanced translocations are very common (1/500) - Carriers at risk for multiple SAB and abnormal offspring See study guide document for detail

tRNAs

Intermediary molecules which transfer amino acids to the growing peptide chain during translation tRNA structure: - tRNAs are short: 75-90 nucleotides long. - A specific amino acid is covalently bound at the 3' end of each tRNA. - tRNAs recognize mRNA through complementary base pairing of the anticodon loop (codon:anticodon).

Physical attending

Involve the nonverbal ways in which you use your body to communicate with patients

Autism spectrum disorder Affects ~1/680 children in the U.S. Is diagnosed using MRI Can have its onset at any time during a person's lifetime Is only rarely associated with ID Is characterized by deficits in communication, repetitive behavior, and social interaction

Is characterized by deficits in communication, repetitive behavior, and social interaction Affects 1/54 children (more prevalent) Onset during development (childhood) More often associated with ID

Isodicentric chromosome example

Isodicentric 15, also called idic(15), partial tetrasomy 15q, or inverted duplication 15 (inv dup 15) People with idic(15) are typically born with 47 chromosomes in their body cells, instead of the normal 46.

Defense mechanism: rationalization

Justifying objectionable information with plausible statements "everyone has some abnormal genes... I'm probably not any more at risk than anyone else" "But I take my prenatal vitamins every day"

In which of the following syndromes is long palpebral fissures an associated feature? Treacher collins Kabuki SLOS Cornelia de Lange

Kabuki

There are about 28,000 people with CF living in the U.S. and ~50% are over the age of 18. Survival has consistently improved with improved therapy. The cause of death in patients with CF can be attributed to all of the following complications except Cardio/pulmonary disease Kidney disease Liver disease Transplant complications/rejection

Kidney disease

A male baby was born with adducted thumbs (on his palms) and hydrocephalus. A mutation was found in the L1CAM gene. What is this condition? What is the chance of recurrence in a future pregnancy.

L1 syndrome or X-linked hydrocephalus with stenosis of aqueduct of sylvius (HSAS): X-linked recessive, so if mom is a carrier; 50% of her daughters will be carriers and 50% of her son's will be affected.

DCM with conduction system disease

LMNA: DCM with arrhythmias like atrial fibrillation, risk of sudden cardiac death, often require a pacemaker - Mutations in LMNA also cause Emery-Dreifuss muscular dystrophy - Can present as cardiomyopathy alone, skeletal myopathy alone, or both SCN5A

On a level II ultrasound the stenographer notes a fetal goiter, club feet, meningomyelocele, and the follow up Echo found Ebstein's anomaly. Genetic testing was negative and you want to make sure all targeted questions and medications for teratogens have been addressed. What teratogen is on the top of your list to rule out?

Lithium Association not as supported now as it was historically

Which of the following is typically lethal in males? MECP2-related disorders Fragile X LMNA-related dilated cardiomyopathy Aarskog syndrome

MECP2-related disorders

Ribosomes are the...?

Machines that translate mRNA Ribosomes are made of ribosomal proteins and ribosomal RNAs (rRNAs). Two subunits: - In eukaryotes, have 60S (large) and 40S (small). ◦ The 60S subunit contains 3 rRNAs (28Ss, 5.8S, 5S) and 49 proteins. ◦ The 40S subunit contains one rRNA (18S) and 33 proteins. The rRNAs provide for all the important catalytic functions associated with translation.

An infant with T21 has a karyotype of 46,XY,der(14;21)(q10;q10),+21. What is most likely to be found for the parental karyotypes?

Majority of T21 due to new mutations, so parents most likely normal. If we were considering a balanced translocation carrier, it is more likely to be maternal

A genetic disorder resulting from mutations in the IKBKG (previously NEMO) gene is characterized by blistering of the skin (birth to age ~4mo), followed by a wart-like rash (for several months), swirling hyperpigmentation (age ~6mo into adulthood), and linear hypopigmentation. What feature of this condition may influence the observed pattern of inheritance? Reduced penetrance Anticipation Male lethality Germline mosaicism

Male lethality

When a person has a CHD, that is most likely a...? Deformation Disruption Dysplasia Malformation

Malformation

RNA interference (RNAi)

Many genes are regulated in part by short noncoding RNAs: - microRNAs (miRNA) are transcribed in the cell as short RNAs with a hairpin structure - Small interfering RNAs (siRNAs) are derived from longer double stranded RNAs

Epidemiology of UPD

Mat/pat UPD has been described for all chromosomes except 19 and Y Total frequencies of all UPDs unkown - No systematic studies to determine prevalence - UPD w/o clinical consequence is not identified - UPD mosaicism is often missed with standard dx algorithms Total estimated incidence of 1/3500 (0.03%) Incidence by chromosome is not known - ascertainment bias - higher incidence of mosaic UPD in chromosomes w/ high trisomy rate

A person with very short stature, where the shortening is particularly noticeable in the forearms and lower legs is most likely to have...? Kyphomelic dysplasia Mesomelic dysplasia Micromelic dysplasia Rhizomelic dysplasia

Mesomelic dysplasia *Rhizo has an R in it for uppeR arm

Microsatellite genotyping

Microsatellites: - Di-, tri-, or tetra- nucleotide tandem repeats in DNA sequences - Number of repeats at a given position is variable within the population - Numerous alleles at any given position (15+) Microsatellite typing: - PCR amplification of these highly polymorphic repeats - Followed by high-resolution gel electrophoresis - Parallel analysis of proband and parents

Fetal Valproate Syndrome

Minor facial features: Short anteverted nose, broad nasal bridge, and small mouth with thin upper lip and everted lower lip. 1-2% risk of neural tube defects, particularly open lumbosacral myelocele Recently, concern has been raised about possible increased risk of neurodevelopmental effects of exposure.

What are the five most common neuromuscular disorders not detectable by exome sequencing?

Most dystrophinopathies (60% DMD, 90% BMD) - exon level del/dups - 1/3500 males (DMD), 1/30,000 males (BMD) CMT1A (and HNPP) - 1.3 Mb duplication at 17p12 (reciprocal deletion for HNPP) - 1/10,000 (CMT1A) Myotonic dystrophy (DM1 and DM2) - CTG trinucleotide repeat (DM1) and CCTG tetranucleotide repeat (DM2) - 1/10,000-20,000 (DM1) Gacio-Scapulo-Humeral muscular dystrophy - contraction of D4z4 repeat at 4q with permissive 4qA haplotype - 1/10,000-25,000 SMA (95%) - Homozygous deletion of SMN1 - 1/10,000

>35 repeats is considered abnormal for the trinucleotide repeat in the DMPK gene and expansion is most likely to occur through the mother or the father?

Mother FYI repeat number for full penetrance in triplet repeat expansion conditions - HD: 40 or more - Fragile X: 200 or more - Friedrich's ataxia: 66 or more

What does MERRF stand for?

Myoclonic Epilepsy with Ragged Red Fibers

A 30 year old male has cataracts and while taking the family history the genetic counselor notes that his sister recently had a pregnancy complicated with polyhydramnios, low fetal movement, and club feet. What condition should be ruled out in your male patient?

Myotonic dystrophy Could have a premutation in DMPK leading to his only feature-cataracts.

Acetylcholinesterase levels is used for what diagnosis in prenatal clinics?

NTD Diagnostic test as part of the amnio AFP performed 14-25wks gestation

A CCTG repeat in the CNBP gene is most likely to expand in the mother or father?

Neither, no clear pattern has been established Tetranucleotide CCTG repeat expansion of CNBP occurs in Myotonic dystrophy type 2

Structures located at the ends of chromosomes are called

Telomeres

De novo 22q deletions occur due to

Non-allelic homologous recombination AND the presence of low copy repeats

Post-fertilization errors (mosaic UPD)

Non-disjunction during MITOSIS, not meiosis Leads to non-disjunction of sister chromatids --> trisomic cell, monosomic cell, and normal cell line --> trisomy rescue occurs resulting in uniparental isodisomy, monosomic cell line lost, and normal cell line End result: mosaicism involving UPD and normal cell line

If an intracardia echogenic focus in an otherwise low risk pregnancy is noted, what follow up testing, if any, should be offered?

None There is no increased risk for heart defects. If the pregnancy is already high risk then consider NIPT and level II ultrasound

Which of the following statements about the etiology of complex disorders is true All complex disorders are etiologically homogenous The etiology of complex disorders involves only SNPs but not CNVs The etiology of complex disorders involves only CNVs but not SNPs None of the above

None of the above Involves both SNPs and CNVs and are etiologically heterogeneous

Robertsonian translocation

Nonreciprocal chromosomal transloccation that commonly involves chromosome pairs 13, 14, 15, 21, and 22 (acrocentric). One of the most common types of translocation. Occurs when the long arms of two acrocentric chromosomes fuse at the centromere and the 2 short arms (stalks) are lost.

If you see a mixture of pectus carinautm and excavatum?

Noonan

Phenotype similar to Turner syndrome? (e.g., "male Turner")

Noonan Dysmorphic facial features Pulmonic stenosis Broad/webbed neck Short stature Lymphadema Cardiac involvement (80-90%)

When you hear pulmonic stenosis, think...?

Noonan syndrome

Which of the following features does not rule out a diagnosis of CF? Meconium ileus Positive sweat test Normal NBS result Pancreatic insufficiency

Normal NBS result

Consanguinity among individuals related as third cousins or more apart is...?

Not considered to be genetically significant.

90% of children with OTC deficiency have a parent who is a carrier. Why is this surprising? What could be an explanation?

OTC is X-linked, so one expects 2/3 of children to have a mother who is a carrier. As the question states, it's actually 90% of children with a carrier mother. This is because the mutation rate is much higher in the male germline than the female germline. So, there are more than expected de novo mutations from grandfathers to mothers of affected boys.

Trisomy rescue

Occurs after meiosis I or II error AND ferilization Leads to UPD only if normal parent's chromatid is kicked out - Due to crossing over, resulting zygote typically has a combination of uniparental heterodisomy (homologues, but not identical) and uniparental isodisomy (identical)

Psychological attending

Occurs when you sense experiences, to the extent possible, through the patient's eyes rather than your own

Muir Torre is most often associated with mutations in what gene? What are the main clinical features of this condition?

Of all MMR genes, MSH2 is enriched for Muir Torre Dermatologic variant of Lynch - occurrence of any LS cancers in addition to sebaceous neoplasms + keratocanthomas

What is the most common CHD in 22q?

Of conotruncal defects, ToF

A pregnant woman who is 12-weeks along receives an inconclusive result from NIPT. She is morbidly obese and is asking what to do now. Do you reorder NIPT? Do you offer diagnostic testing?

Offer diagnostic testing Fetal fraction only increases 0.1% per week on average so if she failed once it is not worth ordering it again. ACMG recommendation

What is pleiotropy? Several genes affect many traits One gene affects many traits One gene affects one trait Several proteins affect many traits

One gene affects many traits

Benefits of miRNAs

One miRNA can regulate many mRNAs (common sequence in UTR) Multiple miRNAs can bind to the same mRNA (increases control of expression) Now known to regulate over 30% of mRNAs (involved in development, differentiation, cell proliferation, apoptosis)

An inborn error of metabolism is detected in fraternal twins (one boy and one girl), but does not affect their older brother. There is no fam hx of the condition. If untreated, this condition is associated with microcephaly, epilepsy, severe ID, musty body odor, eczema, and decreased hair and skin pigmentation. Metachromatic leukodystrophy Homocystinuria PKU Zellweger

PKU

Other HCM genes

PRKAG2 and LAMP2 • Result in metabolic storage disease of the myocardium • Mutations in PRKAG2 may result in Wolff- Parkinson-White (WPW) syndrome with or without HCM • Mutations in LAMP2 result in Danon disease, an XL disorder with cardiomyopathy, muscle weakness, variable ID GLA, RAS/MAPK pathway genes - GLA results in Fabry disease (XL), associated with left ventricular hypertophy - RAS/MAPK --> ~20% of those with Noonan develop HCM

What clinical feature of 22q11.2 deletion syndrome is responsible for the development of polyhydramnios?

Palatal anomalies

What are the tetrasomy syndromes?

Pallister Killian - Tetrasomy 12p, mosaic - Isochromosome 12p or, rarely, trisomy 12p Cat Eye: Microduplication 22q11, partial tetrasomy (33% mosaic) - Small supernumerary chromosome, frequently has 2 centromeres, bisatellited, and represents an inv dup(22)(q11)

miRNAs

Part of RNA-induced silencing complex (RISC) Initial transcripts are procesed into short double stranded RNAs One strand is loaded into complex with Argonaute and other proteins - miRNA component binds to one or more regions in 3' UTR

De novo CNVs tend to be maternal or paternal in origin?

Paternal

Primary empathy response types

Patient response: I'm so upset about this dx. I feel like crying all the time. I can't eat. I can't sleep. I haven't been able to keep my mind on my work. - Minimal encourager - "Mhmm" - Paraphrasing - You've been so upset that you're having difficulty eating, sleeping, and working - Summary - This dx has really upset you. Not only do you feel like crying all the time, it's getting in the way of your daily activities - Content response - This dx seems to have turned your world upside down - Affect response - You seem very distressed - Content + Affect - You seem very distressed by this dx

Examples of heterozygote disadvantage

Pericentric inversion carriers have more frequent miscarriages. Rh incompatibility-a (Rh-positive) heterozygous fetus is at risk of Rh disease if the mother is Rh-negative.

Which cancer syndrome that has an increased risk of colon cancer can also be seen with a family history of cervical cancer?

Peutz Jegher syndrome (STK11) Many associated cancer risks - Also ones that are typically sporadic (cervix and lung) Major - Breast (45-50%) - Colon (39%) - Stomach (39%) Female - Ovarian (18-21) - Uterus (9%) - Cervix (10%) Also - sm intestine, pancreas, lung

Other triplet repeat disorders

Polyglutamine repeat disorders: - DRPLA, SBMA, SCA1, SCA2, SCA3, SCA6, SCA7, SCA17 Myotonic dystrophy: - repeat is in 3'UTR - may sequester RNA processing factors Friedreich ataxia: - Repeat is in intron - Change in DNA structure leads to gene shutoff - Only AR trinucleotide repeat disorder!

A baby with a cleft lip/palate could have what other finding due to difficulty in swallowing?

Polyhydramnios

Chimerism

Post-zygotic fusion of two regularly fertilized normal eggs - Results in the presence of two genomes in one individual

For each of the following genes, would genetic testing of an asymptomatic individual be considered predictive? Or, presymptomatic? a. APC b. RB1 null allele c. MSH2 d. RET

Presymptomatic: APC, RB1 null allele, and RET (MEN2) because cancer risk is ~100% - APC: CRC - RB1: retinoblastoma - RET: MTC Predictive: MSH2 because CRC risk is ~70%, not fully penetrant

Protein structure

Primary: amino acid sequence Secondary: - alpha helix: coiled backbone, right handed (stabilized by H bonds between NH and CO groups of peptide backbone) - beta pleated sheet: interactions between two or more adjacent polypeptide strands (H bonds between CO and NH groups on backbones of adjacent strands) Tertiary: 3D arrangement of one protein chain Quaternary: assembly of more than one protein chain into a complex (ex: hemoglobin = 2 alpha + 2 beta globin chains)

Population-based screening

Screening for common genetic diseases offered to the general population

The proband in the example below is a 1yo male with epilepsy and DD with the following WES results. Proband: - SCN1A VUS + - HEXA VUS + - HEXA mutation + Mom: - SCN1A VUS neg - HEXA VUS + - HEXA mutation + Dad: - SCN1A neg - HEXA VUS neg - HEXA mutation neg What is the most likely interpretation?

Proband has a de novo variant in SCN1A that is likely causative of Dravet syndrome and he is a carrier for Tay Sachs disease

All of the following are later onset complications in MFS except Progressive hearing loss Cardiomyopathy Scoliosis progression Dural ectasia

Progressive HL

Southern blot information

Provide information about size of fragments ◦ May detect deletions, insertions, trinucleotide expansions May be possible to identify heterozygotes Will not see point mutations, small deletions Requires a lot of DNA, time consuming, labor intensive Still used for assessing size of trinucleotide repeats in Fragile X syndrome, Friedreich ataxia, etc.

What are the main features of nemaline myopathy?

Proximal muscle weakness (neck, face, shoulders), difficulties eating, nemaline rods on muscle biopsy, and hypotonia with depressed deep tendon reflexes

Informed consent process for carrier screening should include?

Purpose, voluntary nature of screening Range of symptoms and severity of disorders in question Test performance - Sensitivity/Specificity - Residual carrier risk - Residual risk to have an affected child - Mode of inheritance/recurrence risk Meaning of positive and negative results

Defense mechanism: repression

Putting intolerable thoughts and feelings out of one's mind "I don't remember how many miscarriages I had"

The telomerase uses ___________ as a template?

RNA

Rare diseases as a public health concern (consideration of expanded carrier screening)

Rare diseases (also known as "Orphan diseases"): - Have an incidence of < 1/2000 live births - 5000-7000 different types - Affect 6-8% of the world's population (80% of these are genetic in origin; ~54 million affected people in Europe and North America) - Are increasingly viewed as a legitimate health concern Screening may be considered as part of a continuum of health improvement strategies

Medical records verify you were right about the previous question. The couple are both African-American. The incidence of this condition in this population is 1 in 15,000. What is the chance that the woman is a carrier?

Real answer = 1 in 62 Estimate with shorthand math Incidence = 15,000 = q^2 for AR condition sq root (1/15,000) approximate to sq root (1/10,000) = q = ~ > 1/100 2pq = ~>1/50 = 1/62

4 tenants of genetic counseling that ensure that a psychological milieu is achieved? MEMORY: 1. Start with GC relationship... 2. Patients are... A R E

Relationship is integral to GC Patient autonomy must be supported Patients are resilient Patient emotions make a difference

Clinical guidelines for demonstrating UPD

Required to demonstrate uniparental inheritance of two fully informative markers for the chromosome in question Options: - Microarray SNP-based genotyping of trios - Microsatellite genotyping

A female child has apparently normal development during the first year, followed by stagnation, and rapid regression in language and motor skills. She displays stereotypic hand movements and inconsolable crying. The diagnosis is? Cornelia de lange Rett Bardet-Biedl Sanfilippo syndrome (MPS III)

Rett

MECP2

Rett syndrome (XL dominant)

Defense mechanism: regression

Reverting to developmentally less mature behavior A well-educated and articulate couple, upon hearing abnormal test results, suddenly were unable to process any further information. They kept saying "what do you mean?"

Phenotypes of LQTS

Roman-Ward syndrome - usually AD - 4% risk of SCD in 3 most common subtypes from birth-40y Jervell and Lange-Nielsen syndrome - AR - LQTS and SNHL - KLQT1 (KCNQ1) and KCNE1

In which of the following syndromes is maternal UPD a potential etiology? Cornelia de Lange Russell-Silver Beckwith-Wiedemann Angelman

Russell-Silver (because need paternal expression, whereas BWS and Angelman need maternal expression)

In which of the following syndromes is Y-shaped 2,3 toe syndactyly an associated clinical features? Treacher collins Kabuki SLOS Cornelia de Lange

SLOS - Y-shaped 2-3 toe syndactyly - Microcephaly (80-84%) - Pre- and post-natal growth retardation/short stature - Mod-severe intellectual disability (ID) - Congenital cataracts (20%) - Postaxial polydactyly (25-50%) - Hypospadias in males (50%) - Photosensitivity Treacher collins - lower eyelid coloboma Kabuki - persistent fetal fingertip pads, eversion of lateral proportion of lower eyelid, long palpebral fissures, postnatal growth deficiency, skeletal abnormalities (e.g, hemivertebrae) Cornelia de Lange - Hirsutism, synophrys, microcephaly, growth failure, limb anomalies

Which of the following disorders does NOT have an AD inheritance pattern? NF1 Kabuki SLOS Treacher Collins

SLOS (AR) NF1 - AD Kabuki - majority AD (KMT2D), 3-5% KDM6A (XL-R) Treacher collins - AD

What is a modifier for disease severity in SMA?

SMN2 copy number >3 copies reduces the severity and is not present in SMA 0

Which of these findings can impact pulmonary function in people with Marfan? Dural ectasia Scoliosis Renal cysts Ectopia lentis

Scoliosis

Selection against AR mutations

Selection is less effective (i.e., takes many generations to decrease the mutant gene frequency) since the majority of mutant alleles are "hidden" among the unaffected carriers of the mutant gene where fitness is normal. Unlike AD conditions, increasing fitness by improved medical treatment of individuals affected by AR conditions like cystic fibrosis does little to the frequency of the mutant CF gene.

Defense mechanism: displacement

Shifting response from the original aim to a vulnerable target "I won't get any useful information from you and your incompetent lab"

Mesomelic dysplasia

Shortening of distal limb

Rhizomelic dysplasia

Shortening of proximal limb E.g., achondroplasia

An African American couple in clinic are seeing you for preconception genetic counseling. They know they are both carriers of sickle cell disease and the mother is 40 years old. Does the couple have a higher risk of Down syndrome or sickle cell disease?

Sickle cell disease At age 40 her risk for Down syndrome is 1 in 100, risk for SCD is 25%

Known mutations - testing process

Sickle cell disease, Tay-Sachs Many techniques use sets of oligos ◦ One matches typical sequence ◦ One matches known mutation Bind to sample Measure binding somehow ◦ Directly (ASOH) ◦ Indirectly (extension, ligation) Ex: OLA - uses three oligos in one rxn - one pair is a set of ASOs, hybridizes to typical or to mutant sequence. - third oligo hybridizes adjacent to the first, to sequence in common to typical and mutant - after hybridization, ligase added - if typical allele probe is added to sample containing mutation, last base won't hybridize, ligation doesn't occur - However, you need some method of detection (electrophoresis, MALDI-ROF, etc.)

Familial hypercholesterolemia

Significant elevations in total serum chholesterol and LDL cholesterol early in life - elevated risk for CAD and MI LDLR (AD), homozygotes have earlier onset and more severe disease +more genes, genetic heterogeneity Genetic testing allows for early screening of cholesterol levels

In prenatal clinic you have a couple of African descent and a couple of Southeast Asian descent. Which couple has most risk for having a baby affected with Hb Barts?

Southeast Asian Hb Bart = loss of all four alpha chain alleles Southeast Asian couple, most likely to carry a deletion in both alpha loci in cis (aa/- -). Africans are more likely to have a single mutation or carry them in trans (a-/aa or a-/a-).

How to detect del/dups

Southern blots RT-PCR ◦ Real time polymerase chain reaction MLPA ◦ Multiplex ligation-dependent probe amplification Microarray ◦ Originally best for larger deletions and duplications ◦ Newer methods allow better detection

Case example: Patient with eye tumor - 12mo male noted to have a "lazy eye". - Pediatrician identified a unilateral retinal mass and transferred child to Cancer Center - Subsequent work-up including ophthalmic exams and MRI of the eyes/brain. - Child had normal developmental milestones. - Fam hx unremarkable. Differential dx: Eye tumor was thought to be either: ◦ Unilateral retinoblastoma ◦ Retinal hamartoma STAT genetic testing requested to determine line of treatment - Which gene tests would you order? - Would you order a broad based panel or specific gene test(s)?

Specific genes: ◦ RB1 - Hereditary Retinoblastoma ◦ TSC1/TSC2 genes - Tuberous Sclerosis Since the differential dx (retinoblastoma vs retinal hamartoma) was so specific - cardinal features of their respective syndromes, it made sense to order gene specific testing

SMART goals

Specific, Measurable, Attainable, Realistic, Timely

AGG interruptions in FMR1 Predict the severity of ID in affected males Are only present in unaffected males and females with premutations Stabilize the CGG repeat sequence and make it less likely to expand Increase in number with maternal transmission Are the primary cause of developmental symptoms in Fragile X syndrome

Stabilize the CGG repeat sequence and make it less likely to expand

What trisomy is most commonly associated with choroid plexus cysts in addition to other features?

T18, rarely T21

In which conditions is molecular testing not the optimal screening approach?

Tay Sachs (enzyme first, then DNA) Hemoglobinopathies (electrophoresis, then DNA if needed)

Clinical validity

The extent to which a test result is predictive for a disease - If a mutation is found by a test, can you predict a phenotype or syndrome? ◦ Includes positive and negative predictive value In genetics, limited by: ◦ Genetic heterogeneity ◦ Incomplete penetrance

Which of the following statements regarding Manhattan plots is true The highest peaks represent the genetic variants that very strongly influence the phenotype It derives its name from the researcher who developed it The highest peaks represent the genetic variants that are more strongly associated with the phenotype Are very common in the population

The highest peaks represent the genetic variants that are more strongly associated with the phenotype

Which of the following is true? Marfan is not a variable disorder Marfan is an XL dominant disorder The life span of individuals with Marfan has increased over time Marfan is not a good example of pleiotropy

The life span of individuals with Marfan has increased over time Marfan is variable, AD, and a great example o f pleiotropy Pleiotropy - One gene affecting many body systems

Imprinting for offspring

The newly imprinted alleles inherited by the offspring reflect the parent of origin in the imprinting patterns. ◦ The offspring may differ in which allele is expressed since imprinting is reprogrammed each generation. When these offspring manufacture gametes, the imprinting will once again be erased and reset to reflect the sex.

Prevalence

The number or proportion of cases of a particular disease or condition present in a population at a given time.

Incidence

The number or rate of new cases of a particular condition during a specific time.

Exceptions to random mating: population stratification

The population is divided such that matings across the divisions are less common to those within the divisions (e.g., U.S. Caucasians and African-Americans); Or subgroups within a population tend not to inter-marry with the general population (e.g., Amish, Ashkenazi Jews, or Cajuns). Can result in specific disease alleles occurring at much higher frequency in certain ethnic groups (e.g. BRCA1/2 in Ashkenazi Jews, different deletion mutations in alpha-thalassemia in Mediterranean and Southeast Asian groups)

A couple has a son diagnosed with CF. You counsel them that their RR is 1/4. Which of the following is the most common misconception made by a couple in this situation? All future children will be affected This condition can only occur in males Their next 3 children will be unaffected The parents will develop CF too

Their next 3 children will be unaffected

Robertsonian translocations: theoretical versus empiric risks - rob(14;21)(q10;q10) - rob(13;14)(q10;q10)

Theoretical risk of abnormal live birth for both: 33% Empiric risk for 14;21 - female carrier: 10-15% - male carrier: 2% Empiric risk for 13;14 - female carrier: 1% male carrier: <1%

A couple is seen for GC during the 12th week of pregnancy. The woman is identified to be a CF carrier. She has the F508del mutation. Her partner was negative for a panel screening that tests for the most common CF mutations. During the GC session, it is most important that the couple understands The detection rate for CF mutation screening is extremely high There is a residual risk to be a carrier of a CF mutation since panels do not test for all CF mutations There are over 1400 known CF mutations Geno/phenotype correlations of CF mutations

There is a residual risk to be a carrier of a CF mutation since panels do not test for all CF mutations

In the absence of any other family history of a complex disorder, the chance of recurrence for which kind of relative is approximately the same as the chance of the condition in the general population?

Third degree relative

Ethnicity based carrier screening

This includes screening for the conditions for which you have the highest chance to be a carrier, usually determined by your reported ethnicity and family history as some conditions are more common in certain ethnic populations compared to others.

What does a genetic counselor need to consider for recurrence risk when a new case of Spinal Muscular Atrophy is diagnosed in a child?

This is an autosomal recessive condition but in ~2% of cases one of the two mutation is de novo in the child. - Therefore parental testing should be performed in order to establish an accurate risk to siblings. Parent could also be a 2:0 carrier or have germline mosaicism

In a patient at low risk for trisomy (maternal age under 35y with no other risk factors), the risk for pathogenic CNVs is twice as high as the trisomy risk: True or False

True

Which of the following enzymes is NOT needed for base excision repair? DNA glycosylase AP endonuclease Ligase Topoisomerase

Topoisomerase

TORCH

Toxoplasmosis Other: (Syphillis, varicella, parvovirus) Rubella CMV Herpes

In a screening test, sensitivity, and false positive rate are related to each other such that by increasing the sensitivity, you also increase the FPR: True or False

True

In which of the following syndromes is eyelid coloboma an associated clinical feature? Treacher collins Kabuki SLOS Cornelia de Lange

Treacher collins

Coping behaviors: confrontative

Tries to change the opinions of the person who is in charge (e.g., GC or physician)

Coping behaviors: positive reappraisal

Tries to see any possible positive results or outcomes (e.g., BRCA+, now I can do something to lower my risks)

This cytogenetic abnormality makes up 20% of all chromosomally abnormal miscarriages and rarely survive long enough to be a recognized pregnancy.

Triploidy

Proposed mechanisms of UPD

Trisomy rescue - Occurs when a chromosome does not split during fertilization, and the zygote has 3 copies of the same chromosome. The zygote then tries to correct 3 copies by deleting one. There is a possibility that the deleted chromosome is not a double copy. Monosomy rescue - Monosomy rescue occurs when a chromosome is lost and the zygote corrects by duplicating whichever copy is still present. Gamete complementation - Gamete complementation occurs when a gamete that has 2 copies of the chromosome fertilizes (or is fertilized with) a gamete with 0 copies of the chromosome. The abnormalities in both gametes result in UPD even though no "rescue" was required

At any maternal age, the PPV for Down syndrome is higher than it is for T13: True or False?

True Why? Because DS is more prevalence than T13

True or False: Male balanced robertsonian translocation carriers are less likely to transmit their balanced translocation to their offspring when compared to females

True. Meitoic division favors maternal transmission of balanced rob translocations. Her children will have a 60% chance of also being balanced translocation carriers.

Which phenotypic feature is associated with Klinefelter syndrome? a. gynecomastia b. holoprosencephaly c. overlapping fingers d. short stature e. webbed neck

a. gynecomastia

Uniparental heterodisomy

Two chromosomes from the one parent are not identical (homologous)

Double strand break repair

Two distinct mechanisms exist: ◦ Non-homologous recombinational repair (end joining) ◦ Homologous recombinational repair

MLPA method

Two reactions: ligation and amplification ◦ MLPA uses just one primer pair for amplification Design probe pairs for different parts of the gene of interest

Which of the following is NOT bound to a mature mRNA as it is exported to the cytoplasm? CAP binding proteins PolyA binding protein Exon junction complexes U1 snRNP

U1 snRNP

A long run of homozygosity (ROH) that is detected on a single chromosome is most likely due to:

UPD

Mutations in the TTN gene causing foot drop and distal muscle weakness, especially of the anterior tibia, are features of what condition?

Udd distal myopathy

Specificity

Unaffected with negative test / all unaffected 1 - specificity = false positive rate

Reciprocal engagement model - four factors

Understanding and appreciation Support and guidance Facilitative decision making Patient-centered education 17 REM goals come out of these 4 factors

What type of UPD do SNP arrays potentially detect and why?

Uniparental isodisomy (not heterodisomy) Why? - SNP array detects the absence of heterozygosity which could represent uniparental isodisomy - Doesn't detect heterodisomy because they are non-identical chromosomes HOWEVER, threshold for AOH calls is usually 5-10 Mb in clinical analysis; thus, suspected UPD on SNP array must BE CONFIRMED!

Incidental/secondary findings

Unrelated to the indication for ordering testing, but of medical value or utility to the ordering physician and patient - medically actionable condition (childhood or adult onset) - carrier status - pharmacogenetic variants (e.g., warfarin)

The function of DNA helicase is?

Unwinding the DNA double helix and separating the strands

All of these are risk factors for aortic dissection except Diameter of aortic root Fam hx of dissection Use of beta blockers Rate of chance of diameter of aorta

Use of beta blockers

Women who are exposed to Diethylstilbestrol (DES) in utero have what features of this teratogen?

Uterine abnormalities, vaginal adenocarcinomas, and increased rates of infertility.

You are taking a family history before a level II ultrasound and the father mentions he had a cleft palate and you see lip pits on his lower lip. What condition should you suspect and counsel the family on?

Van der woude

Cystic fibrosis - CFTR splicing

Variations in the length of the polypyrimidine tract (T tract) (i.e., alternative splicing) do not cause disease on their own - Homozygotes for 5T may have male infertility - Compound heterozygotes that have one allele with 5T and one with a severe mutation may have symptoms - An allele that has 5T as well as a mild mutation may function as a severe mutation *~10% of the population carries a 5T allele

Gamete complementation

When a gamete with two copies of a chromosome (should have only one), and it gets fertilized with a gamete that happens to have no copies of that chromosome Result is disomic uniparental heterodisomy/isodisomy *if no crossing over had occurred, an MI error would result in heterodisomy and an MII error would result in isodisomy (same for trisomy rescue)

Robertsonian translocations

Whole q arm exchanges of acrocentric chromosomes (13, 14, 15, 21, 22); short arms lost - carriers are balanced - 90% are dicentric - common recurring chromosomal rearrangement (~1/1000 individuals) - rob(13q14q) and rob(14q21q) are most common - often associated with trisomy of chromosomes involved - increased risk for UPD

Supravalvular aortic stenosis is seen in what condition?

Williams

Which of the following syndromes is associated with supravalvular aortic stenosis? Williams Noonan T21 Velocardiofacial syndrome

Williams

What type of tumor is associated with WAGR syndrome?

Wilms tumor WAGR stands for= wilms tumor, aniridia, genital abnormalities, and retardation (ID). Associated with 11p13 deletions

Most mutations are found?

Within the protein coding region

A pedigree shows vertical transmission and about twice as many affected females than males. What pattern of inheritance is suspected? Multifactorial Mito XL-R XL-D

XL-D

Which of the following conditions is caused by defects in nucleotide excision repair? HBOC Lynch MUTYH polyposis Xeroderma pigmentosum

XP

Which of the following are risk factors for colon cancer? [select as many as apply] a. Ashkenazi Jewish ancestry b. insulin-dependent diabetes c. ulcerative colitis d. hypertension

a, b, c

Which conditions are NOT XL-D (male lethal)? a. Aarskog syndrome b. Aicardi syndrome c. Coffin Lowry d. F8-related disease e. incontinentia pigmenti f. Rett syndrome

a, c, d Aarskog and Hemophilia A (F8 related disease) are XL-R without neonatal male lethality Coffin Lowry is XL-D but males survive (just more affected than females)

Increased cancer risk is NOT associated with which of these chromosomal anomalies? a. 1p36 deletion syndrome b. Down syndrome c. Klinefelter syndrome d. Turner syndrome

a. 1p36 deletion syndrome - Significant overlap with PWS (hypotonia, DD/ID, growth retardation, obestiry) - Seizures/ straight eyebrows - Cardiomyopathy/minor CHDs- - Vision defects/Deep set eyes B. Down syndrome - yes, leukemia C. Klinefelter - yes, breast, mediastinal germ cell tumors (1% risk), no screening D. Turner syndrome, wilms tumor, leukemia, gonadoblastoma if mosaic with Y

A prophylactic oophorectomy reduces a premenopausal woman's risk of breast cancer by__ and reduces her risk of ovarian cancer by __ a. 50% and 80-90% b. 50% and 100% c. 25% and 100% d. 15% and 80-90%

a. 50% and 80-90%

For each pair, pick which is more frequent: a. 69,XXX or 69,XXY b. 69,XXY or 46,XX/69,XXY mixoploidy c. 47,XX,+8 or 46,XX/47,XX,+8 (at livebirth) d. 47,XX,+9 or 46,XX/47,XX,+9 (at livebirth)

a. 69,XXY (XXY more common than triple X) b. 69,XXY (XXY or XXX more common than mixoploidy c. 46,XX/47,XX,+8 (mosaic more viable than full trisomy) d. 46,XX/47,XX,+9 (mosaic more viable than full trisomy)

Which risk model includes the woman's personal risk information, her first degree relatives breast cancer history to estimate her risk of invasive breast cancer? a. Gail b. BRCAPRO c. Claus d. IBIS

a. Gail used for tamoxifen effectiveness, takes into account: age, first degree relatives, number and outcome of biopsies - atypical hyperplasia, age of menarche, age of first liveborn. Limitations: no personal history of cancer, does not account for AJ, does not account for paternal aunts having cancer or ovarian cancer, lacks 2nd degree, and does not account for high numbers of cancer or age of onset

IHC testing is negative for MSH2 and MSH6. What is the most plausible cause of this test result? a. MSH2/EPCAM germline mutations, rarely MSH6 b. MSH2 germline mutation c. EPCAM germline mutation d. MSH6 germline mutation

a. MSH2/EPCAM germline mutations, rarely MSH6

Name the 3 main areas of concern for a person with a single pathogenic mutation in the FLCN gene. a. Skin, lung, renal b. Renal, colon, lung c. Skin, renal, pancreatic d. What the heck

a. Skin, lung, renal FCLN --> Birt Hogg Dube --> chromophobe or oncocytoma RCC, lung blebs/risk for spontaneous pneuomothorax, skin lesions (fibrofolliculomas and trichodiscomas) Multiple fibrofolliculomas are pathognomonic

A first-trimester ultrasound shows a nuchal translucency at the 98th percentile. What is the most likely outcome? a. a healthy, euploid pregnancy b. Down syndrome c. congenital heart defect d. Turner syndrome

a. a healthy, euploid pregnancy

Isolated clubfoot is noted on prenatal ultrasound. What should be offered to the patient to address this finding? a. comprehensive ultrasound and chromosome analysis by amniocentesis b. comprehensive ultrasound and NIPT c. comprehensive ultrasound only d. no further testing is indicated

a. comprehensive ultrasound and chromosome analysis by amniocentesis

Learning of inherited cancer risk can lead patients to use denial as a coping mechanism. Match the types of denial: disbelief, dismissal, distraction, deferral a. challenges the credibility of the info b. shifts the topic of conversation c. hears the info but does not accept it d. accepts the info but does not seem to absorb all its implications

a. dismissal b. distraction c. disbelief d. deferral MEMORY: - dismissal (dismissing what you are saying) - deferral (deferring it to another time)

A family history of Noonan syndrome could provide an explanation for which of the following findings on prenatal ultrasound? a. increased NT or second trimester cystic hygroma b. echogenic intracardiac foci c. hypoplastic nasal bone d. ventriculomegaly

a. increased NT or second trimester cystic hygroma

Isolated hyperechoic bowel is noted on prenatal ultrasound. Which of the following is the most likely outcome? a. normal, live-born infant b. gastrointestinal malformation c. Down syndrome d. IUGR

a. normal, live-born infant

A patient has a personal and family history of breast cancer. Genetic testing comes back for a BARD1 variant. To check the most recent data on whether there is an association between BARD1 variants and breast cancer, you start looking on PubMed. You find a case-control study. What type of quantification is the study most likely to report? a. odds ratio = (frequency of BARD1 mutations in affected individuals) / (frequency of BARD1 mutations in unaffected individuals) b. odds ratio = (incidence of breast cancer in BARD1 carriers) / (incidence of breast cancer in non - BARD1 carriers) c. relative risk = (incidence of breast cancer in BARD1 carriers) / (incidence of breast cancer in non - BARD1 carriers) d. relative risk = (frequency of BARD1 mutations in affected individuals) / (frequency of BARD1 mutations in unaffected individuals)

a. odds ratio = (frequency of BARD1 mutations in affected individuals) / (frequency of BARD1 mutations in unaffected individuals) Odds ratio used in case control (start with disease and see who has mutation) - odds of exposure in outcome A / odds of exposure in outcome B - exposure = genotype - outcome A = cancer - outcome B = no cancer Memory: cohort has an r in it for relative risk - Relative risk used in cohort (prospective, start with unaffected, genotype, and then see who gets disease) - Answer C would be what you would see in a cohort study - risk of outcome in exposed / risk of outcome in unexposed

When clubfoot is noted on prenatal ultrasound, what possibility is the greatest concern? a. presence of additional congenital abnormalities b. congenital myotonic dystrophy c. chromosome abnormalities d. neurodevelopmental delay

a. presence of additional congenital abnormalities

For the couple in #8-10, what genetic testing or screening would you offer? a. take a careful family history alert to possible genetic conditions, and then offer standard testing based on that family history and ethnic background b. expanded carrier screening is highly recommended for both parents c. prenatal WES to address the increased risk for autosomal recessive disease d. refer for a Level II fetal anatomy ultrasound to address the increased risk of birth defects

a. take a careful family history alert to possible genetic conditions, and then offer standard testing based on that family history and ethnic background

The prenatal ultrasound finding of shortened long bones would most raise suspicion for a skeletal dysplasia such as achondroplasia if: a. the shortening was more severe (less than the 3rd percentile as opposed to the 5th) b. onset of shortening was earlier in the pregnancy (1st or 2nd trimester as opposed to the 3rd) c. only femur length was shortened d. only humerus length was shortened

a. the shortening was more severe (less than the 3rd percentile as opposed to the 5th)

Match the UPD to the clinical significance: a. upd(6)pat b. upd(7)mat c. upd(11p15)pat d. upd(14)mat e. upd(14)pat f. upd(15)mat g. upd(15)pat

a. transient neonatal diabetes mellitus b. Russell Silver syndrome: growth retardation, triangular facies c. Beckwith-Wiedemann: overgrowth, omphalocele, Wilms tumor d. short stature, mild DD, hypotonia, scoliosis, hypermobile joints (Temple syndrome) - MEMORY: M in Temple, need maternal expression like Angelman and BWS e. ID, short-limb dwarfism, respiratory problems, scoliosis (Kagami-Ogata syndrome) f. Angelman syndrome g. Prader-Willi syndrome

A prenatal patient listens to your offer of maternal serum screening for neural tube defects and asks if there would be a reason to have the screening since she would not terminate. What do you answer? a. yes, the results can aid pregnancy management independent of the decision to carry or terminate b. yes, if positive, the results would provide information on the severity of the defect c. yes, if positive, the results are diagnostic d. no, NT or nuchal fold measurement by ultrasound is a reasonable substitute

a. yes, the results can aid pregnancy management independent of the decision to carry or terminate There are some clinics that use US instead (but not by NT or nuchal fold; those are unrelated)

Analytic validity

ability of a genetic test to accurately and reliably measure the genotype of interest - If a lab receives a sample with a particular mutation, how likely is it that it will report a positive result? - Sensitivity and specificity for the mutations that are being looked for

Telecanthus

abnormally increased distance between the medial canthi of the eyelids (pupils appear more medial due to inner canthi lateral placement)

According to ACMG guidelines, carrier screening for Jewish genetic disorders should include testing for which of the following conditions? [select as many as apply] a. Pompe disease b. Gauchet disease c. Fanconi anemia group A d. Fanconi anemia group C e. ataxia-telengiectasia f. cystic fibrosis g. Glycogen Storage Disease Type I h. Niemann-Pick type A i. Bloom syndrome j. Leber's Hereditary Optic Neuropathy k. Mucolipidosis type IV l. Familial dysautonomia, Tay-Sachs, Canavan disease

b, d, f, h, i, k, l ACMG: Gaucher, Fanc C, CF, Niemann-Pick type A, Bloom, Mucolipidosis type IV, familial dysautonomia, tay sachs, and canavan disease ACOG: cystic fibrosis, familial dysautonomia, tay sachs, and canavan disease

The general population risk for ovarian cancer is___ a. 0.5% b. 1.5% c. 3% d. 6%

b. 1.5%

Four couples, all unaffected. Which couple has the highest chance to have a child with achondroplasia? a. 30-year-old female + 30-year-old male whose brother has achondroplasia b. 30-year-old female + 40-year-old male c. 30-year-old female whose brother has achondroplasia + 30-year-old male d. 40-year-old female + 30-year-old male

b. 30-year-old female + 40-year-old male New mutations associated with APA AD condition so sibling with achondroplasia doesn't increase risk to pregnancy Not A or C because 100% penetrant AD, so those siblings are wild-type. 80% of cases are de novo; de novo mutations for FGFR3 are almost exclusively paternal, and there is an age effect.

A woman is diagnosed with breast cancer at age 27. There is no other family history of cancer. What is the chance of a TP53 mutation? a. less than 5% b. 5-10% c. 11-20% d. more than 20%

b. 5-10% Unilateral breast cancer <30y without any other core cancers --> only 7% with TP53 mutations

Which prior probability risk model calculates the risk of a woman having a BRCA mutation? a. Gail b. BRCAPRO c. Claus d. IBIS

b. BRCAPRO BRCAPRO: prior probability- calculates risk of breast cancer by penetrance of the gene. Includes male breast cancers, three generations of breast cancer, ovarian cancer, ages of relatives, AJ,

Which of the following symptoms are NOT associated with Von Hippel Lindau Syndrome? a. Endolymphatic sac tumors b. Cataracts c. Epididymal cystadenomas d. Hemangioblastomas

b. Cataracts Retinal angiomas do not cause cataracts

You see a couple in clinic, both Ashkenazi Jewish. The woman reports that as a child, her brother bruised easily, was short, had excessive fatigue, bone deformities, and a swollen abdomen that was determined to be hepatosplenomegaly. What was the most likely diagnosis for the brother? a. Canavan b. Gaucher disease c. Mucolipidosis IV d. Niemann-Pick disease

b. Gaucher disease

Mutations in the FH gene cause this autosomal dominant condition that has an increased risk of kidney cancer. Which condition is this? a. Familial Gastrointestinal Stromal Tumor Syndrome b. Hereditary Leiomyomatosis Renal Cell Cancer Syndrome c. Multiple Endocrine Neoplasia type 2 d. Hereditary Papillary Renal Cell Carcinoma

b. Hereditary Leiomyomatosis Renal Cell Cancer Syndrome Increased risk for papillary type 2 RCC

What is the second most common cancer seen in Juvenile Polyposis Syndrome? a. Colon b. Stomach c. Skin d. Pancreatic

b. Stomach Colon: 40-50% risk Stomach: 21% if multiple gastric polyps Small intestine, pancreas, HHT = rare

A patient has an apparently isolated prenatal ultrasound finding of a single umbilical artery (SUA) in the second trimester. She was previously considered to be at low risk for aneuploidy. What do you counsel her? a. There is an increased risk for associated structural anomalies. A Level II ultrasound is indicated and will most likely find other anomalies. b. The association of SUA with aneuploidy is questionable. A Level II ultrasound may be considered. Prenatal testing and screening is not necessarily indicated. c. There is increased risk for aneuploidy, especially trisomy 21. A Level II ultrasound and maternal serum screening or NIPT is indicated. d. There is increased risk for aneuploidy, especially trisomy 18. A Level II ultrasound and maternal serum screening or NIPT is indicated.

b. The association of SUA with aneuploidy is questionable. A Level II ultrasound may be considered. Prenatal testing and screening is not necessarily indicated.

Prenatal ultrasound shows isolated intracardiac echogenic focus for a pregnancy that is otherwise at low risk for aneuploidy. What do you counsel the patient? a. The relative risk for Down syndrome is close to 20. Amniocentesis is indicated. Consider detailed ultrasound evaluation and either maternal serum screening or NIPT. b. The relative risk for Down syndrome is close to 1. Invasive testing is not indicated. Consider detailed ultrasound evaluation and either maternal serum screening or NIPT. c. The risk of a congenital heart defect is increased. Fetal echocardiogram is indicated. Family history should be evaluated for other individuals with congenital heart defects. d. The risk of 22q11.2 deletion syndrome is increased. Chromosome microarray by amniocentesis is indicated. Consider detailed ultrasound examination.

b. The relative risk for Down syndrome is close to 1. Invasive testing is not indicated. Consider detailed ultrasound evaluation and either maternal serum screening or NIPT.

Cleft lip is noted on prenatal ultrasound. What should be offered to the patient to address this finding? a. NIPT b. chromosome analysis and chromosome microarray by amniocentesis c. maternal serum AFP screening d. AFP screening by amniocentesis

b. chromosome analysis and chromosome microarray by amniocentesis

In Peutz-Jeghers syndrome, GI hamartomas are usually ____ and often _____. Colorectal cancer risk is ____. a. found in the small intestine; cause complications such as bleeding; increased mainly due to the hamartomas b. found in the small intestine; cause complications such as bleeding; increased mainly due to adenomatous polyps c. found in the small intestine; cause complications such as bleeding; only slightly increased d. found in the colon and rectum; do not cause complications such as bleeding; only slightly increased

b. found in the small intestine; cause complications such as bleeding; increased mainly due to adenomatous polyps Hamatomatous polyps (which includes PJS polyps) have lower malignant potential than adenomatous polyps Most common polyp location: small intestine (jejunum) · Symptoms: obstruction, intussusception in children especially, abdominal pain, and GI bleeding begin at <20y in 50%

Which of the following ultrasound findings does NOT occur more commonly in males than females? a. hydronephrosis b. single umbilical artery c. cleft lip and/or palate d. talipes equinovarus

b. single umbilical artery (more common in females than in males)

Mild ventriculomegaly is detected on mid-trimester ultrasound. Which of the following is not correct? a. there is an increased risk for developmental delay, especially if the ventriculomegaly is stable or enlarged on later ultrasounds. b. there is an increased risk for seizures, especially if the ventriculomegaly is stable or enlarged on later ultrasounds. c. ventriculomegaly is associated with neural tube defects, Down syndrome, and other genetic syndromes d. ventriculomegaly can be caused by congenital infections such as toxoplasmosis or CMV

b. there is an increased risk for seizures, especially if the ventriculomegaly is stable or enlarged on later ultrasounds.

A 35-year-old Caucasian woman has a prenatal ultrasound at 18 weeks gestation that shows a hypoplastic nasal bone. No other screening has been performed. The risk that the pregnancy is affected by Down syndrome is now closest to: a. 1 in 500 b. 1 in 100 c. 1 in 20 d. 1 in 4

c. 1 in 20

What is the general population versus the Ashkenazi Jewish population carrier frequency for BRCA1 and BRCA2 mutations? a. 1/250 general and 1/30 AJ b. 1/500-1/800 general and 1/25 AJ c. 1/500-1/800 general and 1/40 AJ d. 1/250 general and 1/40 AJ

c. 1/500-1/800 general and 1/40 AJ

Assessment for LFS is warranted by a breast cancer diagnosis before what age? a. 20 b. 25 c. 30 d. 35

c. 30 But I say 35

What tumor/cancer finding is the most suggestive of Li Fraumeni syndrome? a. Breast cancer b. Adrenocortical carcinoma c. Choroid plexus tumor d. Sarcoma

c. Choroid plexus tumor Basically pathognomonic for LFS in children

Which risk model calculates risk of breast cancer in an unaffected woman that incorporates age of onset of cancer in first and second degree relatives on both the maternal and paternal sides? a. Gail b. BRCAPRO c. Claus d. IBIS

c. Claus Claus Model: better at incorporating family histories than the Gail Model. However it still does not include male breast cancers, and not personal risk factors,

This cancer syndrome has mucosal trichilemmomas and acral keratosis in addition to hamartoma polyps and follicular thyroid cancer. Which condition is it? a. PJS b. JPS c. Cowden d. Birt-Hogg-Dube

c. Cowden

Choroid plexus cysts are noted on prenatal ultrasound. When you counsel the 30-year-old patient, what is the most important point? a. this finding increases the risk of aneuploidy b. this finding increases the risk of aneuploidy only if the pregnancy was already at high risk for aneuploidy c. a detailed anatomy survey by ultrasound is necessary d. chromosome microarray by amniocentesis should be considered

c. a detailed anatomy survey by ultrasound is necessary

Hyperechoic bowel is noted on prenatal ultrasound. Which of the following could provide an explanation for this finding? a. both parents are carriers for Bardet-Biedl syndrome b. both parents are carriers for beta-thalassemia c. both parents are carriers for cystic fibrosis d. both parents are carriers for Joubert syndrome

c. both parents are carriers for cystic fibrosis

A patient has a family history of breast cancer as well as several cases of leukemia. A cousin with breast cancer carries a BRCA1 mutation. The patient asks if the BRCA mutation could explain the leukemia cases. You go to PubMed, and find a prospective cohort study on this question. What type of quantification is the study most likely to report? Same choices as #19. a. odds ratio = (frequency of BRCA1 mutations in affected individuals) / (frequency of BRCA1 mutations in unaffected individuals) b. odds ratio = (incidence of breast cancer in BRCA1 carriers) / (incidence of breast cancer in non - BRCA1 carriers) c. relative risk = (incidence of breast cancer in BRCA1 carriers) / (incidence of breast cancer in non - BRCA1 carriers) d. relative risk = (frequency of BRCA1 mutations in affected individuals) / (frequency of BRCA1 mutations in unaffected individuals)

c. relative risk = (incidence of breast cancer in BRCA1 carriers) / (incidence of breast cancer in non - BRCA1 carriers) Start with unaffected individuals --> genotype --> see who gets disease

A couple - a 30-year-old woman and a 40-year-old man - present to prenatal clinic in the first-trimester. There is no family history of chromosome abnormalities, genetic conditions, congenital malformations, or recurrent miscarriage. According to ACMG guidelines, how should the couple be counseled regarding risk of chromosomal abnormalities? a. there is a lower than average risk of Down syndrome b. there is an average risk of Down syndrome: ~ 1 in 1000 at live birth c. there is a potentially increased risk of Down syndrome d. they qualify for high-risk screening options including NIPS

c. there is a potentially increased risk of Down syndrome because of advanced paternal age (at most 1.5X maternal age-related risk). Also increased risk of schizophrenia, autism, and some AD diseases.

Shortened long bones are most accurate at predicting risk of Down syndrome in which trimester? a. first trimester b. second trimester c. third trimester d. both second and third trimester

c. third trimester

Hydrocephalus is a severe form of _______. a. hydrops b. choroid plexus cyst c. ventriculomegaly d. neural tube defect

c. ventriculomegaly

Coping behaviors: accept responsibility

criticizes or blame's oneself ("it's my fault this has happened")

Screening using MSAFP levels can detect __% of open neural tube defects and __% of anencephaly with a screen positive rate of 5% or less. a. 40-50%; 50% b. 40-50%; 95% c. 75-90%; 50% d. 75-90%; 95%

d. 75-90%; 95%

Put the following steps involved with NGS in order: A. gNDA is fragmented into a library of small segments that are each sequenced in parallel. B. DNA is extracted. C. The whole genome sequence is derived from the consensus of aligned reads. D. Individual sequence reads are reassembled through a bio-informatic process by aligning to a reference genome. a. C, A, B, D b. A, B, D, C c. B, A, C, D d. B, A, D, C

d. B, A, D, C

The patient then completes first trimester maternal serum screening, and the results come back as negative. She returns at 18w1d, and says that she is not entirely reassured by the negative result and would like more screening. What should be considered? [select as many as apply] a. quad screen b. penta screen c. NIPS d. fetal anatomy ultrasound

d. Do NOT use quad screen or NIPS after 1st tri serum screening.

When prenatal ultrasound shows absence of the nasal bone, which conditions become more likely? a. Down syndrome b. Down syndrome and trisomy 18 c. Down syndrome, trisomy 18, and trisomy 13 d. Down syndrome, trisomy 18, trisomy 13, and Turner syndrome

d. Down syndrome, trisomy 18, trisomy 13, and Turner syndrome

Which of the following statements are FALSE? a. DCIS is noninvasive b. DCIS Comedo type has an increased risk of being invasive c. LCIS has a 25-30% risk of invasive cancer d. LCIS is a premalignant lesion

d. LCIS is a premalignant lesion LCIS is NOT a premalignant lesion. It is only a marker for breast cancer. If LCIS is found on a biopsy the other breast is just as likely to get breast cancer. This is not the case in DCIS, which is premalignant and has a ~30% chance of breast cancer if left alone LCIS does have a 25-30% chance of invasive cancer, but not considered premalignant

Hamartomas are usually most abundant in what body part in Peutz-Jeghers Syndrome? a. R sided colon b. L sided colon c. Stomach d. Small intestine

d. Small intestine R sided adenomas more common in LS L sided more common in FAP

When using second trimester ultrasound findings to adjust the risk for Down syndrome, which of the following is NOT true? a. One major malformation or multiple soft markers increase the risk for aneuploidy, and invasive testing should be offered b. The most comprehensive risk assessment will use positive likelihood ratios to adjust for the presence of certain markers and use negative likelihood ratios to adjust for the absence of other markers. c. The likelihood ratios are highest for ventriculomegaly, increased nuchal fold, and absent or hypoplastic nasal bone. d. The likelihood ratios are highest for echogenic bowel, hydronephrosis, and absent or hypoplastic nasal bone.

d. The likelihood ratios are highest for echogenic bowel, hydronephrosis, and absent or hypoplastic nasal bone. Likelihood ratios highest for ventriculomegaly (25), thickened nuchal fold (11-18.6), and absent/hypoplastic nasal bone (23-83)

Prenatal ultrasound in the second trimester detects cystic hygroma. What is the most likely outcome? a. a healthy, euploid pregnancy b. Down syndrome c. congenital heart defect d. Turner syndrome

d. Turner syndrome 2nd trimester --> turner syndrome becomes more likely than euploid fetus

Which of the following is NOT correct regarding the relative frequency of ultrasound findings in different ethnic groups? a. intracardiac echogenic focus is more common in Asian women b. short femur length is more common in Asian women and less common in African American women c. hypoplastic nasal bone is more common in Afro-Caribbean women d. cleft lip and/or palate is most common in Caucasian women and African American women

d. cleft lip and/or palate is most common in Caucasian women and African American women

In Cowden syndrome, GI hamartomas are usually ____ and often _____. Colorectal cancer risk is ____. a. found in the small intestine; cause complications such as bleeding; increased mainly due to the hamartomas b. found in the small intestine; cause complications such as bleeding; increased mainly due to adenomatous polyps c. found in the small intestine; cause complications such as bleeding; only slightly increased d. found in the colon and rectum; do not cause complications such as bleeding; only slightly increased

d. found in the colon and rectum; do not cause complications such as bleeding; only slightly increased 9% risk of CRC

A previously healthy 5-year-old boy presents with abdominal pain followed by fever, rapid heart rate, rapid breathing, and lack of responsiveness. Laboratory tests showed no obvious toxins but massive nonimmune intravascular hemolysis and hemoglobinuria. The boy was of Greek ancestry. What's the most likely diagnosis? a. alpha thalassemia b. beta thalassemia c. Familial Mediterranean fever d. glucose-6-phosphate dehydrogenase deficiency

d. glucose-6-phosphate dehydrogenase deficiency

A woman is diagnosed with breast cancer at age 50. She has macrocephaly (98th percentile). She meets criteria for consideration of Cowden syndrome ___ a. with no additional personal/family history b. if her brother has autism c. if her mother had endometrial cancer d. if she has multiple mucocutaneous lesions

d. if she has multiple mucocutaneous lesion Clinical dx with 3 major criteria or 2 major and 1 minor - has to be in one individual only - Breast cancer and macrocephaly = 2 major Mucocutaneous lesions in HER = major criteria and clinical dx FYI: - endometrial cancer = major feature but would need to be in her, not mom - ASD is minor criteria

Which of the following teratogens is associated with an increased risk for cleft lip and palate? a. maternal diabetes b. warfarin c. thalidomide d. maternal smoking

d. maternal smoking

The positive likelihood ratio of the isolated prenatal ultrasound finding of a single umbilical artery (SUA) for Down syndrome is closest to: a. 5 b. 20 c. not applicable; the risk is for trisomy 18 d. not applicable; the association with Down syndrome is questionable

d. not applicable; the association with Down syndrome is questionable

You are working as a lab GC. You are reviewing a test order for sequencing of the FH gene. What would be an appropriate indication for that testing? a. personal history of dyslipidemia; family history of early-onset heart attacks b. clear cell renal carcinoma c. chromophobe clear cell renal carcinoma d. papillary type 2 renal carcinoma

d. papillary type 2 renal carcinoma FH is gene for Hereditary Leiomyomatosis and Renal Cell Cancer (HLRCC) --> papillary type 2 RCC VHL is the gene for VHL --> clear cell RCC FLCN is gene for Birt Hogg Dube --> chromophobe and oncocytoma MET (proto-oncogene) is gene for Hereditary Papillary Renal Cancer (HRPC) --> papillary type 1 RCC

A female child has apparently normal development during the first year, followed by stagnation, and rapid regression in language and motor skills. She displays stereotypic hand movements and inconsolable crying. The genetic counselor explains that this is an XL condition that is __________________ the majority of the time, but may also ____________________. de novo; be inherited from a mother with skewed X-inactivation inherited; be de novo caused by gene deletions; be the result of missense mutations de novo; display anticipation

de novo; be inherited from a mother with skewed X-inactivation

Which of the following APOE genotypes has the greatest risk of Alzheimer disease? a. E2/E2 b. E2/E3 c. E3/E3 d. E3/E4 e. E4/E4

e. E4/E4

If all four major proteins in Lynch syndrome are present on IHC testing and MSI is low, what additional testing for Lynch syndrome do you perform? a. BRAF methylation studies b. Germline genetic testing c. Consider EPCAM deletion d. All of the above e. None of the above

e. None of the above

A 25-year-old woman at low-risk for fetal aneuploidy presents for prenatal care. She is at 8w2d. She has no concerning personal or family history. At this visit, the provider should offer her which of the following? a. options for maternal serum screening for chromosome aneuploidy b. options for invasive diagnostic testing for chromosome aneuploidy c. fetal anatomy ultrasound at 18w0d - 20w0d d. MSAFP in the second trimester e. all of the above

e. all of the above MSAFP in second trimester if patient opts for FTS, not Quad

Benign or malignant? a. lipoma b. hemangioma c. pheochromocytoma d. colon adenoma e. none of the above

e. none of the above All are benign

When tumor NGS identifies a TP53 mutation the person is likely to have Li Fraumeni: true or false?

false

Brachycephaly

fused coronal suture

Dolichocephaly

fused sagittal suture

Clinical utility

how likely the test is to significantly improve patient outcomes - if you know the genotype, can you positively impact a patient's health? - how useful is this test for my patient? Summary: - analytic validity: identify mutation - clinical validity: predict disease - clinical utility: improve patient outcomes

IBD

identical by descent

ACMG secondary findings

includes 59 medically actionable genes - Many cancer predisposition (child and adult onset) - Some cardiac/connective tissue disorders (EDS - vascular type, Marfan, Loeys-Dietz, hypertrophic and dilated cardiomyopathy) - Others: OTC deficiency, Wilson disease, familial hypercholesterolemia, long QT syndromes, etc. - REMOVAL of familial thoracic aortic aneurysms and dissections (MYLK) due to rarity of variants and lack of clinical utility Last revised in 2016 Patients/parents have to OPT OUT: carrier status, pharmacogenetic (lab-specific) Not covered: - common, multifactorial disease - adult onset with no known treatment (ALS, Alzheimer's, Parkinson's) - Limited info on parents' genetic status

Fragile X syndrome... Occurs only in males Is the most common known inherited cause of autism Is caused by a de novo mutation in 10% of cases Results from mRNA toxicity secondary to FMR1 full mutations

is the most common known inherited cause of autism

Coping behaviors: self-controlling

keeps feelings to oneself (e.g., "I'm fine")

A genetic disorder affects a great-grandfather, grandfather, mother, and daughter in the same bloodline. The great-grandfather, grandfather, and mother displayed muscle weakness, cataracts, cardiac conduction abnormalities, and difficulty relaxing their grip during day-to-day activities, like gripping a doorknob. At birth, the daughter has severe hypotonia and difficulty breathing. What condition best fits this phenotype? Limb-girdle muscular dystrophy DMD Fragile X Myotonic dystrophy type 1

myotonic dystrophy type 1

75% of cases of Marfan syndrome are inherited. The incidence is 1 in 5,000. What is the new mutation rate for FBN1?

new mutation rate = mu = q * s s = coefficient of selection against mutant allele. s = 1 - fitness s = 1 - 0.75 = 0.25 (ie the de novo rate) this is an AD condition, so 2pq = incidence 2pq = 1 / 5000 q = 1 / 10,000 mu = q * s mu = (1 / 10,000) * (1/4) = 1 / 40,000 or 2.5 x 10-5

Exception to HW "no migration" - gene flow (population admixture)

non-random slow diffusion of a gene across a reproductive (e.g. ethnic or geographical) barrier. The genes of migrant populations, with their own gene frequencies, are gradually merged into the gene pool of the population into which they have migrated. This can be used to map migration patterns of certain groups. Unlike genetic drift which occurs by chance, gene flow occurs by migration.

Incidence of triploidy is [associated / not associated] with maternal age and is {associated /not associated] with paternal age. RR is [not increased / 1% / 5%]

not associated not associated not increased

The measurement from the inside of one eye to the outside of the same eye is the

palpebral fissure length

Fitness

probability of transmitting one's genes to the next generation compared to average probability for the population. Coefficient of selection (s) = 1 - f (a measure of the loss of fitness)

Ribosomal components (rRNA)

rRNAs are the most abundant RNAs in the cell, comprising ~80% of the total RNA. 45S RNA is a large precursor RNA which is processed to give all smaller individual rRNAs except 5S. - Human cells contain about 200 copies of the 45S rDNA ◦ 45S rDNA is found in clusters on five chromosomes: 13, 14, 15, 21, 22 - 5S rDNA is present in about 2000 copies found in a tandem array on chromosome 1

Dural ectasia - anatomic description

refers to ballooning or widening of the dural sac which can result in posterior vertebral scalloping and is associated with herniation of nerve root sleeves

Defense mechanism: denial

rejecting the possibility that an event happened "nobody every told me I was at risk" "he'll grow out of it" "she looks just like my mother, and my mother is okay"

Brachydactyly is...?

short fingers

Micromelic dysplasia

shortening of proximal and distal limbs

Disruption

structural defect resulting from DESTRUCTION of a body part that has differentiated normally (amniotic band)

Malformation

structural difference arising from a primary localized error in morphogenesis (CHD, polydactyly, CP) Malformed from meiosis (e.g., developmental/genetic cause)

Lyon hypothesis

the proposal that dosage compensation in mammalian females is accomplished by partially and randomly inactivating one of the two X chromosomes Exceptions: - inactivation is not always random (structurally abnormal X non-random inactivation observed) - inactivation not complete (maybe up to 15% of all XL genes escape inactivation) - inactivation is reversible in development of germ cells

What differentiates Costello from other rasopathies?

well-defined tumor risk - 15% lifetime risk - rhabdomyomas most common

Exception to HW "population is infinitely large" - founder effect/population bottleneck (a form of genetic drift)

when, by chance, a genetic isolate is formed. Either by a small number of "founders' split away from the main population, or the population is reduced to a very small number. This isolate may have a higher frequency of particular mutant alleles, especially if one or more of the founders of the small group carried a rare mutant allele (e.g. HD in Venezuelan community, Ellis van Crevald in Old Order Amish, tyrosinemia in French Canadians.)

Uniparental diploidy (genome-wide UPD)

whole chromosome complement is derived from one parent

The frequency of the APOE allele e2 is 10%, e3 is 75%, and e4 is 15%. What proportion of the population will be heterozygous?

~ 0.4 or 40%. Homozygotes v heterozygotes Heterozygotes = 1 - ( (1/10)^2 + (3/4)^2 + (15/100)^2 ) = ~40%

Mutations affecting splicing, transcription, and other REGULATORY processes account for how much of single gene mutations?

~10-20%

General diagnostic yield of WES

~30%

Measurement of mutation rate (μ) is dependent on selection

μ = s x q For AD conditions: e.g., achondroplasia, fitness (f) equals 0.2, therefore, s = 0.8 (i.e., 80% of all mutant alleles (q) are the result of new mutations) Incidence = 1/10,000 = 2pq; therefore, q (the frequency of the mutant allele) is 1/20,000. μ = 0.8 x 1/20,000 = 4 x 10-5 for achondroplasia.

Considerations when picking conditions for ACMG secondary findings

• Clinical Utility of Secondary Findings: treatment, screening, or potential for medical consequences in an asymptomatic person • Limitations and Interpretation of Secondary Findings • Patient Preferences and Secondary Findings • Secondary Findings in Children

Types of anomalies by trimester

• First Trimester - All aneuploids, 45,X, triploids • Second Trimester - All aneuploids, 45,X, triploids - Increase in deletions, duplications, homozygosity • Third Trimester - Common "more viable" aneuploids, +18, +21, 45,X - Increase in deletions, duplications, homozygosity

(ACOG committee opinion 690) Given the multitude of conditions that can be included in expanded carrier screening panels, the disorders selected for inclusion should meet several of the following criteria:

• Have a carrier frequency of 1 in 100 or greater • have a well-defined phenotype • have a detrimental effect on quality of life, cause cognitive or physical impairment, require surgical or medical intervention, or have an onset early in life • should be able to be diagnosed prenatally and may afford opportunities for antenatal intervention to improve perinatal outcomes, changes to delivery management to optimize newborn and infant outcomes • carrier screening panels should not include conditions primarily associated with a disease of adult onset

Detection of genomic changes: various methodology

• Unbanded Chromosomes: 20 Mb • Chromosomes - 550 Band Level: 10 Mb • High-Resolution Chromosomes (g-banding): 3-5 Mb • FISH: 150 kb • CMA (BAC, oligo, SNP): 50 - 150 kb

POC studies - why do them?

• ~15% of pregnancies are spontaneously aborted in the first trimester • ~50% of first trimester SABs are chromosomally abnormal • Some POCs fail to grow in culture --> Can reflex from a no growth to an array study to get results (>95% successful)

What is the PWS 1% rule?

•1% have a translocation or inversion resulting in a deletion of bands 15q11.2-q13. •< than 1% have a "balanced" rearrangement breaking within 15q11.2-q13

How do SNP arrays work?

•SNP positions are determined •335 million SNPs known in the human genome, 1 million used in array •Most common allele at each site = "A", least common = "B" •Two probes created for each SNP site •Patient DNA amplified and hybridized •Different color or light intensity signals determine allele at SNP site •Relative allele distribution provided by either allele difference plots or B-allele frequency plots

SNP array

•SNP-based microarrays detect CNVs as well as genotype information at multiple polymorphic loci throughout the genome In essence SNP data helps: - Confirm CNV calls made by oligo array - Increase sensitivity for mosaicism detection - Allow for detection of excessive homozygosity If homozygous at all SNPs --> suspicious for UPD; If mix of homozygous and heterozygous at SNPs --> less suspicious

Oligo array

•The original technology utilized by CMA •SNP array able to be added in 2012 •Oligo arrays determine copy number only •Utilize oligomers across the genome (not just at SNP sites) •Can determine copy number variants •Report CNVs with Log R ratio plots

Consequences of nonsense mutations (premature stop codon)

◦ Could have nearly complete protein with reduced activity ◦ Could have dramatically shortened protein with no activity ◦ Could have shortened protein that interferes with activity Ex: - beta-thalassemia usually caused by biallelic (AR) mutations which reduce the amount of available beta-globin --> mild to moderate anemia - Rare cases with nonsense mutations in the third (last) exon which result in production of a shorter, non-functional protein that interferes with assembly of full-length β-globin and α-globin chains --> severe, AD anemia


Conjuntos de estudio relacionados

Are the effects of privation reversible?

View Set

Iggy Chapter 56: Care of Patients with Noninflammatory Intestinal Disorders

View Set

INFS2608 Lecture 1 - Introduction

View Set

Midterm Exam Exp. 1-9 Monday night

View Set

Health Care System/Organizations

View Set

Ch. 3: Financial Instruments, Financial Markets, and Financial Institutions - Money and Banking

View Set